Acca Financial Reporting f7 LSBF Revision Kit [PDF]

  • 0 0 0
  • Gefällt Ihnen dieses papier und der download? Sie können Ihre eigene PDF-Datei in wenigen Minuten kostenlos online veröffentlichen! Anmelden
Datei wird geladen, bitte warten...
Zitiervorschau

lOMoARcPSD|4664814

ACCA Financial Reporting F7 LSBF Revision Kit Accounting (Hanoi University)

StuDocu is not sponsored or endorsed by any college or university Downloaded by isavic Alsina ([email protected])

lOMoARcPSD|4664814

LSB_F7_Rev Kit:297mm x 210mm

28/10/09

13:36

Page 1

F7 Financial Reporting (INT)

ACCA Downloaded by isavic Alsina ([email protected])

lOMoARcPSD|4664814

LSB_F7_Rev Kit:297mm x 210mm

28/10/09

13:36

Page 2

FINANCIAL REPORTING (INTERNATIONAL)

British Library Cataloguing-in-Publication Data A catalogue record for this book is available from the British Library Published by InterActive World Wide Limited Westgate House, 8-9 Holborn London EC1N 2LL www.iaww.com www.studyinteractive.org ISBN 978-1-907217-21-0 First Edition 2009 Printed in Romania

© 2009 InterActive World Wide Limited. London School of Business & Finance and the LSBF logo are trademarks or registered trademarks of London School of Business & Finance (UK) Limited in the UK and in other countries and are used under license. All used brand names or typeface names are trademarks or registered trademarks of their respective holders. We are grateful to the Association of Chartered Certified Accountants (ACCA), the Chartered Institute of Management Accountants (CIMA) and the Institute of Chartered Accountants of England and Wales (ICAEW) for their permission to reproduce past examination questions. All the solutions to these questions have been prepared by InterActive World Wide Limited. All our rights reserved. No part of this publication may be reproduced, stored in a retrieval system, or transmitted, in any form or by any means, electronic, mechanical, photocopying, recording or otherwise, without the prior written permission of InterActive World Wide.

2

Downloaded by isavic Alsina ([email protected])

lOMoARcPSD|4664814

LSB_F7_Rev Kit:297mm x 210mm

28/10/09

13:36

Page 3

FOREWORD

Foreword

Thank you for choosing to study with the London School of Business and Finance (LSBF). A dynamic, quality-oriented and innovative educational institution, the London School of Business and Finance offers specialised programmes, designed with students and employers in mind. We are always at the frontline driving the latest professional developments and trends. LSBF attracts the highest quality candidates from over 140 countries worldwide. We work in partnership with leading accountancy firms, banks and best-practice organisations – enabling thousands of students to realise their full potential in accountancy, finance and the business world. With an international perspective, LSBF has developed a rich portfolio of professional qualifications and executive education programmes. To complement our face-to-face and cutting-edge online learning products, LSBF is now pleased to offer tailored study materials to support students in their preparation for exams. The exam focused content in this manual will provide you with a comprehensive and up-to-date understanding of the ACCA syllabus. We have an award-winning team of tutors, who are highly experienced in helping students through their professional exams and have received consistently excellent feedback. I hope that you will find this manual helpful and wish you the best of luck in your studies. Aaron Etingen ACCA, BA, Founder and CEO

3

Downloaded by isavic Alsina ([email protected])

lOMoARcPSD|4664814

LSB_F7_Rev Kit:297mm x 210mm

28/10/09

13:36

Page 4

FINANCIAL REPORTING (INTERNATIONAL)

4

Downloaded by isavic Alsina ([email protected])

lOMoARcPSD|4664814

LSB_F7_Rev Kit:297mm x 210mm

28/10/09

13:36

Page 5

CONTENTS

Contents

Foreword

3

Contents

5

How to use this LSBF Revision Kit

9

About ACCA Paper F6 - Taxation (UK)

13

Questions

17

Answers

71

Feedback and Review Form

161

5

Downloaded by isavic Alsina ([email protected])

lOMoARcPSD|4664814

LSB_F7_Rev Kit:297mm x 210mm

28/10/09

13:36

Page 6

FINANCIAL REPORTING (INTERNATIONAL)

6

Downloaded by isavic Alsina ([email protected])

lOMoARcPSD|4664814

LSB_F7_Rev Kit:297mm x 210mm

28/10/09

13:36

Page 7

CONTENTS Number

Name of Question

1

Hepburn

Pilot Paper 200x

2

Highveldt

June 2005

3

Hydan

June 2006

4

Holdrite

December 2004

5

Hedra

December 2005

6

Hosterling

December 2006

7

Horsefield

June 2002

8

Winger

Pilot Paper 200x

9

Petra

December 2005

10

Allgone

June 2003

11

Tadeon

December 2006

12

Kala

Pilot Paper 200x

13

Telenorth

December 2001

14

Tourmalet

December 2003

15

Wellmay

June 2007

16

Peterlee

June 2006

17

Derringdo

June 2003

18

Broadoak

December 2001

19

Wilderness

December 2005

20

Elite Leisure

December 2005

21

Linnet

June 2004

22

Torrent

June 2006

23

Bowtock

December 2003

24

Triangle

June 2005

25

Atkins

December 2002

26

CB

May 2005

27

QRS

Pilot Paper 200x

28

Harper

December 2002

29

FW

May 2005

30

LMN

xxxx

31

EFG

Pilot Paper 200x

32

Breadline

June 2002

33

Comparator

December 2003

34

Bigwood

December 2004

35

Tabba

December 2005

36

Nedberg

December 2002

37

Minster

December 2006

38

Rytetrend

June 2003

39

Update

June 2003

40

Appraisal

F7 Pilot Paper

7

Downloaded by isavic Alsina ([email protected])

lOMoARcPSD|4664814

LSB_F7_Rev Kit:297mm x 210mm

28/10/09

13:36

Page 8

FINANCIAL REPORTING (INTERNATIONAL)

8

Downloaded by isavic Alsina ([email protected])

lOMoARcPSD|4664814

LSB_F7_Rev Kit:297mm x 210mm

28/10/09

13:36

Page 9

F7 How to use this LSBF Revision Kit

9

Downloaded by isavic Alsina ([email protected])

lOMoARcPSD|4664814

LSB_F7_Rev Kit:297mm x 210mm

28/10/09

13:36

Page 10

FINANCIAL REPORTING (INTERNATIONAL)

10

Downloaded by isavic Alsina ([email protected])

lOMoARcPSD|4664814

LSB_F7_Rev Kit:297mm x 210mm

28/10/09

13:36

Page 11

HOW TO USE THIS LSBF REVISION KIT

How to use this LSBF Revision Kit So, you have a nice big revision kit packed with questions, but do not know where to start? And when to start? And how to start?! Where and When to Start This rather depends on where you have reached in your studies: •

If you have not yet started your class, online program, or self-study plan, then it is a bit early to be attempting exam-standard questions! However, it would be useful to pick out some questions at random and read them through. Get a feel for the requirements, as these tend to be examined over and over again – and this will help you when you start your studying. Your brain will link what you read and hear with the exam question requirements you have already seen, and this should help to strengthen your overall understanding of how the knowledge gets tested.



If you have started your class, online program, or self-study plan, you will probably have a tutor, or study planner, which directs you to practise specific questions, based on what you have covered so far.



Try to keep up with the suggested questions – as the real exam approaches there are likely to be plenty more exam questions that you are told to attempt, so do not fall behind! The more questions you have practised, the more likely you are to be successful.



As the real exam approaches, do not be afraid to repeat questions you have practised before. Any good question is worth doing at least twice!



Don’t fall into the trap of starting with Q1, then Q2, Q3 etc. as this tends to result in lots of practice of the first syllabus area and virtually no practice of anything else! Pick questions at random from throughout the book, and keep a record of what you have attempted so far, by ticking the Question List after every attempt.

How to Start – Attempting Questions There are in fact several ways to attempt a question. Some are better methods than others, but it all rather depends on how close you are to the exams and how your brain works. THE FULL QUESTION ATTEMPT •

To start, read ONLY the requirements: o Break each requirement down as much as you can – for example, the following requirement has three separate things to do to earn marks: “Discuss the ethical issues in the scenario and how the directors and auditors should manage them.” Your answer would need to discuss issues, explain what the directors should do, and then explain what the auditors should do – three separate tasks, each of them carrying part of the total mark. o Note how many marks are available. It varies by exam, but the most common allocation is one mark for each point you adequately make in your answer. So, if you are not sure, a 9-mark requirement probably wants you to cover 9 separate points. o Re-read the requirements to make sure you fully understand them – it is these that drive the format of the answer.



Before you read the story/scenario (if there is one), PLAN what might go into your answer. It might be the case that once you read the story and have a better idea of what is happening, some of your early thoughts can be discarded. However, if you were asked to produce a tax computation, or an audit plan, or a set of financial statements, you should have a good idea of the structure and content of your answer without reading the detailed story.

11

Downloaded by isavic Alsina ([email protected])

lOMoARcPSD|4664814

LSB_F7_Rev Kit:297mm x 210mm

28/10/09

13:36

Page 12

FINANCIAL REPORTING (INTERNATIONAL) •

If there is a story/scenario supporting the requirements, then clearly you should read it. But: o Simply copying parts of the story into your answer is unlikely to earn marks – unless you ADD something yourself. Typically this means using the story to illustrate the technical point that the requirement has forced you to discuss. o Try to put any relevant information straight into your answer plan – it is easy to highlight dozens of interesting things, then have to read the whole story again to remember why you highlighted them!



When writing your full answer, use the requirement to structure your answer and provide any headings you may need.



Keep your presentation neat. Most exam questions requiring written answers do NOT want a long discursive essay. Most answers need a series of relatively short paragraphs explaining a series of points briefly, but with enough detail to make it obvious what you are trying to explain. For more guidance on this, look at the Answers in the back of this book.



When you have finished your attempt, take a couple of minutes to rest – then review the suggested solution in the Revision Kit. Make a note of those elements of the answer that you got wrong or missed out. The brain tends to be a logical thing – if you miss a point or make a mistake this time, there is every chance you will make the same mistake again!

THE QUICK QUESTION ATTEMPT Of course, in the real exam you have to provide full answers – so you MUST make plenty of full attempts, as described above. However, especially with written questions, it is often possible to spend only 10-15 minutes on a question and still get some real benefit. o Simply read the requirements, and plan an answer as described above in “The Full Question Attempt”. Now go and look at the answer, especially the headings and layout. If your plan is similarly structured, then you were clearly on the right path and understood the tasks set by the examiner. o Now go back and read the scenario. You know your plan is ok, so try to put things from the scenario into your planned structure, wherever they seem to be most relevant. Just expand your plan – but do not write out a full answer. This is a useful exercise as you get closer to the real exam, as it allows you to gain confidence in the examiner’s requirements and also see a large number of different questions relatively quickly. However, for questions with numbers and calculations, only a full attempt is likely to work.

The Last 4 Exam Papers The most recent exam papers, together with the examiner’s own suggested solutions, can be found, for free, on the ACCA’s website at www.accaglobal.com/students/acca/exams In our Revision Kits, additional questions have been included to ensure that you can practise things similar to recent exam questions. Examiners tend to be very repetitive in style, but clearly they are not going to set exactly the same story and numbers again!

12

Downloaded by isavic Alsina ([email protected])

lOMoARcPSD|4664814

LSB_F7_Rev Kit:297mm x 210mm

28/10/09

13:36

Page 13

F7 About ACCA Paper F7 Financial Reporting (INT)

Downloaded by isavic Alsina ([email protected])

lOMoARcPSD|4664814

LSB_F7_Rev Kit:297mm x 210mm

28/10/09

13:36

Page 14

FINANCIAL REPORTING (INTERNATIONAL)

14

Downloaded by isavic Alsina ([email protected])

lOMoARcPSD|4664814

LSB_F7_Rev Kit:297mm x 210mm

28/10/09

13:36

Page 15

ABOUT ACCA PAPER F6 - TAXATION

Aim of the Paper The aim of Paper F7, Financial Reporting is to develop knowledge and skills in understanding and applying accounting standards and the theoretical framework in the preparation of financial statements of entities, including groups, and how to analyse and interpret those financial statements. The paper also forms the basis of the assumed knowledge required in Paper P2, Corporate Reporting.

Outline of the Syllabus • • • • •

Conceptual framework. Regulatory framework. Financial Statements (11 areas). Business Combinations. Analysing & interpreting financial statements.

Format of the Exam Paper Exam Paper (All Compulsory) Q1 Consolidation including small discussion element; computations designed to test understanding of principles (25 marks) Q2 Preparing / Restating Financial Statements (Published A/cs) (25 marks) Q3 Performance appraisal/interpretation and / or cash flows (25 marks) Q4 Conceptual / regulatory framework – Standards (15 marks) Q5 Conceptual / regulatory framework – Standards (10 marks) Remember Paper F7 is ACCA’s SECOND Level Financial Accounting: it is very different to Paper F3/1.1 (and will be examined in greater depth!)

Getting the most from your studies Candidates need to understand the theory and concepts underlying the preparation and regulation of an entity's financial reports, to apply their knowledge of accounting standards to prepare financial statements of both single entities and groups, and finally, to demonstrate their analytical skills to assess the performance of entities based on the information provided by those financial statements.

15

Downloaded by isavic Alsina ([email protected])

lOMoARcPSD|4664814

LSB_F7_Rev Kit:297mm x 210mm

28/10/09

13:36

Page 16

FINANCIAL REPORTING (INTERNATIONAL)

16

Downloaded by isavic Alsina ([email protected])

lOMoARcPSD|4664814

LSB_F7_Rev Kit:297mm x 210mm

28/10/09

13:36

Page 17

F7 Questions

Downloaded by isavic Alsina ([email protected])

lOMoARcPSD|4664814

LSB_F7_Rev Kit:297mm x 210mm

28/10/09

13:36

Page 18

FINANCIAL REPORTING (INTERNATIONAL)

18

Downloaded by isavic Alsina ([email protected])

lOMoARcPSD|4664814

LSB_F7_Rev Kit:297mm x 210mm

28/10/09

13:36

Page 19

QUESTIONS

1.

Hepburn

(a)

On 1 October 2008 Hepburn acquired 80% of the equity share capital of Salter by way of a share exchange. Hepburn issued five of its own shares for every two shares in Salter. The market value of Hepburn's shares on 1 October 2008 was $3 each. The share issue has not yet been recorded in Hepburn's books. The summarised financial statements of both companies are: INCOME STATEMENTS YEAR TO 31 MARCH 2009 Hepburn $'000 1,200 (650) 550 (120) Nil 430 (100) 330

Sales revenues Cost of sales Gross profit Operating expenses Debenture interest Profit before tax Income tax expense Profit for the year

Salter $'000 1,000 (660) 340 (88) (12) 240 (40) 200

STATEMENTS OF FINANCIAL POSITION AS AT 31 MARCH 2009 Hepburn $'000 Assets Non-current assets Property, plant and equipment Investments Current assets Inventory Accounts receivable Bank

Salter $'000

620 20 640 240 170 20

Total assets Equity and liabilities Equity Equity shares of $1 each Retained earnings Non-current liabilities 8% debentures Current liabilities Trade accounts payable Current tax payable

$'000

660 10 670 280 210 40

430 1,070

530 1,200

400 450 850

150 700 850

nil

150

170 50

155 45 220 1,070

Total equity and liabilities

$'000

200 1,200

The following information is relevant: (i)

The fair values of Salter's assets were equal to their book values with the exception of its land, which had a fair value of $125,000 in excess of its book value at the date of acquisition.

19

Downloaded by isavic Alsina ([email protected])

lOMoARcPSD|4664814

LSB_F7_Rev Kit:297mm x 210mm

28/10/09

13:36

Page 20

FINANCIAL REPORTING (INTERNATIONAL) (ii)

In the post acquisition period Hepburn sold goods to Salter at a price of $100,000, this was calculated to give a mark-up on cost of 25% to Hepburn. Salter had half of these goods in inventory at the year end.

(iii)

Consolidated goodwill is reviewed annually for impairment. At 31 March 2009 its impaired value was $180,000.

(iv)

The current accounts of the two companies disagreed due to a cash remittance of $20,000 to Hepburn on 26 March 2009 not being received until after the year end. Before adjusting for this, Salter's debit balance in Hepburn's books was $56,000.

(v)

The non-controlling interest is measured using the proportion of net assets method.

Required Prepare a consolidated income statement and statement of financial position for Hepburn for the year to 31 March 2009. (20 marks) (b)

At the same date as Hepburn made the share exchange for Salter’s shares, it also acquired 6,000 'A' shares in Woodbridge for a cash payment of $20,000. The share capital of Woodbridge is made up of: Equity voting A shares Equity non-voting B shares

10,000 14,000

All of Woodbridge’s equity shares are entitled to the same dividend rights; however during the year to 31 March 2009 Woodbridge made substantial losses and did not pay any dividends. Hepburn has treated its investment in Woodbridge as an ordinary long-term investment on the basis that: (i) (ii) (iii)

It is only entitled to 25% of any dividends that Woodbridge may pay; It does not have any directors on the board of Woodbridge; It does not exert any influence over the operating policies or management of Woodbridge.

Required Comment on the accounting treatment of Woodbridge by Hepburn’s directors and state how you believe the investment should be accounted for. (5 marks) Note. You are not required to amend your answer to part (a) in respect of the information in part (b). (25 marks)

2.

Highveldt Highveldt, a public listed company, acquired 75% of Samson’s ordinary shares on 1 April 2008. Highveldt paid an immediate $3.50 per share in cash and agreed to pay a further amount of $108 million on 1 April 2009. Highveldt’s cost of capital is 8% per annum. Highveldt has only recorded the cash consideration of $3.50 per share. The summarised statements of financial positions of the two companies at 31 March 2009 are shown below:

Tangible non-current assets (note (i)) Development costs (note (iv)) Investments (note (ii)) Current assets Total assets

Highveldt $m 420 nil 300 720 133 853

20

Downloaded by isavic Alsina ([email protected])

Samson $m 320 40 20 380 91 471

lOMoARcPSD|4664814

LSB_F7_Rev Kit:297mm x 210mm

28/10/09

13:36

Page 21

QUESTIONS Highveldt $m Equity and liabilities Ordinary shares of $1 each Reserves: Share premium Revaluation reserve Retained earnings – 1 April 2008 – year to 31 March 2009

Non-current liabilities 10% inter company loan (note (ii)) Current liabilities Total equity and liabilities

Samson $m

$m

$m

270

80

80 45

40 nil

160 190

134 76 350 745

210 330

nil 108 853

60 81 471

The following information is relevant: (i)

Highveldt has a policy of revaluing land and buildings to fair value. At the date of acquisition Samson’s land and buildings had a fair value $20 million higher than their book value and at 31 March 2009 this had increased by a further $4 million (ignore any additional depreciation).

(ii)

Included in Highveldt’s investments is a loan of $60 million made to Samson at the date of acquisition. Interest is payable annually in arrears. Samson paid the interest due for the year on 31 March 2009, but Highveldt did not receive this until after the year end. Highveldt has not accounted for the accrued interest from Samson.

(iii)

Samson had established a line of products under the brand name of Titanware. Acting on behalf of Highveldt, a firm of specialists, had valued the brand name at a value of $40 million with an estimated life of 10 years as at 1 April 2008. The brand is not included in Samson’s statement of financial position.

(iv)

Samson’s development project was completed on 30 September 2008 at a cost of $50 million. $10 million of this had been amortised by 31 March 2009. Development costs capitalised by Samson at the date of acquisition were $18 million. Highveldt’s directors are of the opinion that Samson’s development costs do not meet the criteria in IAS 38 ‘Intangible Assets’ for recognition as an asset.

(v)

Samson sold goods to Highveldt during the year at a profit of $6 million, one-third of these goods were still in the inventory of Highveldt at 31 March 2009.

(vi)

An impairment test at 31 March 2009 on the consolidated goodwill concluded that it should be written down by $22 million. No other assets were impaired.

(vii)

It is group policy to measure the non-controlling interest using the proportion of net assets method.

Required (a)

Calculate the following figures as they would appear in the consolidated statement of financial position of Highveldt at 31 March 2009: (i) (ii) (iii)

Goodwill; (8 marks) Non-controlling interest; (4 marks) The following consolidated reserves: share premium, revaluation surplus and retained earnings. (8 marks)

Note: Show your workings. (b)

Explain why consolidated financial statements are useful to the users of financial statements (as opposed to just the parent company’s separate (entity) financial statements). (5 marks) (25 marks)

21

Downloaded by isavic Alsina ([email protected])

lOMoARcPSD|4664814

LSB_F7_Rev Kit:297mm x 210mm

28/10/09

13:36

Page 22

FINANCIAL REPORTING (INTERNATIONAL)

3.

Hydan On 1 October 2008 Hydan, a publicly listed company, acquired a 60% controlling interest in Systan paying $9 per share in cash. Prior to the acquisition Hydan had been experiencing difficulties with the supply of components that it used in its manufacturing process. Systan is one of Hydan’s main suppliers and the acquisition was motivated by the need to secure supplies. In order to finance an increase in the production capacity of Systan, Hydan made a non-dated loan at the date of acquisition of $4 million to Systan that carried an actual and effective interest rate of 10% per annum. The interest to 31 March 2009 on this loan has been paid by Systan and accounted for by both companies. The summarised draft financial statements of the companies are: INCOME STATEMENTS FOR THE YEAR ENDED 31 MARCH 2009 Hydan

Revenue Cost of sales Gross profit Operating expenses Interest income Finance costs Profit/(loss) before tax Income tax (expense)/relief Profit/(loss) for the year

Systan preacquisition $’000 24,000 (18,000) 6,000 (1,200) nil nil 4,800 (1,200) 3,600

$’000 98,000 (76,000) 22,000 (11,800) 350 (420) 10,130 (4,200) 5,930

postacquisition $’000 35,200 (31,000) 4,200 (8,000) nil (200) (4,000) 1,000 (3,000)

STATEMENTS OF FINANCIAL POSITIONS AS AT 31 MARCH 2009

Non-current assets Property, plant and equipment Investments (including loan to Systan) Current assets Total assets Equity and liabilities Ordinary shares of $1 each Share premium Retained earnings Non-current liabilities 7% Bank loan 10% loan from Hydan Current liabilities Total equity and liabilities

Hydan $’000

Systan $’000

18,400 16,000 34,400 18,000 52,400

9,500 nil 9,500 7,200 16,700

10,000 5,000 20,000 35,000

2,000 500 6,300 8,800

6,000 nil 11,400 52,400

nil 4,000 3,900 16,700

The following information is relevant: (i)

At the date of acquisition, the fair values of Systan’s property, plant and equipment were $1.2 million in excess of their carrying amounts. This will have the effect of creating an additional depreciation charge (to cost of sales) of $300,000 in the consolidated financial statements for the year ended 31 March 2009. Systan has not adjusted its assets to fair value.

22

Downloaded by isavic Alsina ([email protected])

lOMoARcPSD|4664814

LSB_F7_Rev Kit:297mm x 210mm

28/10/09

13:36

Page 23

QUESTIONS (ii)

In the post acquisition period Systan’s sales to Hydan were $30 million on which Systan had made a consistent profit of 5% of the selling price. Of these goods, $4 million (at selling price to Hydan) were still in the inventory of Hydan at 31 March 2009. Prior to its acquisition Systan made all its sales at a uniform gross profit margin.

(iii)

Included in Hydan’s current liabilities is $1 million owing to Systan. This agreed with Systan’s receivables ledger balance for Hydan at the year end.

(iv)

An impairment review of the consolidated goodwill at 31 March 2009 revealed that its current value was $375,000 less than its carrying amount.

(v)

Neither company paid a dividend in the year to 31 March 2009.

(vi)

It is Hydan Group policy to measure the non-controlling interest at fair value. At the date of acquisition of Systan, the fair value of a 40% holding was $5.5million

Required (a)

Prepare the consolidated income statement for the year ended 31 March 2009 and the consolidated statement of financial position at that date. (20 marks)

(b)

Discuss the effect that the acquisition of Systan appears to have had on Systan’s operating performance. (5 marks) (25 marks)

4.

Holdrite Holdrite purchased 75% of the issued share capital of Staybrite and 40% of the issued share capital of Allbrite on 1 April 2009. Details of the purchase consideration given at the date of purchase are: Staybrite: a share exchange of two shares in Holdrite for every three shares in Staybrite plus an issue to the shareholders of Staybrite 8% loan notes redeemable at par on 30 June 2011 on the basis of $100 loan note for every 250 shares held in Staybrite. Allbrite: a share exchange of three shares in Holdrite for every four shares in Allbrite plus $1 per share acquired in cash. The market price of Holdrite’s shares at 1April 2009 was $6 per share. The summarised income statements for the three companies for the year to 30 September 2009 are: Holdrite $000 75,000 (47,400) 27,600 (10,480) 17,120 (170) 16,950 (4,800) 12,150

Revenue Cost of Sales Gross Profit Operating expenses Operating Profit Interest expense Profit before tax Income tax expense Profit for year

Staybrite $000 40,700 (19,700) 21,000 (9,000) 12,000

Allbrite $000 31,000 (15,300) 15,700 (9,700) 6,000

12,000 (3,000) 9,000

6,000 (2,000) 4,000

The following information is relevant: (i) A fair value exercise was carried out for Staybrite at the date of its acquisition with the following results:

Land Plant

Book value $‘000 20,000 25,000

Fair value $‘000 23,000 30,000 23

Downloaded by isavic Alsina ([email protected])

lOMoARcPSD|4664814

LSB_F7_Rev Kit:297mm x 210mm

28/10/09

13:36

Page 24

FINANCIAL REPORTING (INTERNATIONAL) The fair values have not been reflected in Staybrite’s financial statements. The increase in the fair value of the plant would create additional depreciation of $500,000 in the post acquisition period in the consolidated financial statements to 30 September 2009. Depreciation of plant is charged to cost of sales. (ii)

The details of each company’s share capital and reserves at 1 October 2008 are:

Equity shares of $1 each Share premium Retained earnings

Holdrite $‘000 20,000 5,000 18,000

Staybrite $‘000 10,000 4,000 7,500

Allbrite $‘000 5,000 2,000 6,000

(iii)

In the post acquisition period Holdrite sold goods to Staybrite for $10 million. Holdrite made a profit of $4 million on these sales. One-quarter of these goods were still in the inventory of Staybrite at 30 September 2009.

(iv)

Impairment tests on the goodwill of Staybrite and the investment in Allbrite at 30 September 2009 resulted in the need to write down Staybrite’s goodwill by $750,000.

(v)

Holdrite paid a dividend of $5 million on 20 September 2009. Staybrite and Allbrite did not make any dividend payments.

(vi)

Holdrite Group measure the non-controlling interest using the proportion of net assets method.

Required (a)

Calculate the goodwill arising on the purchase of the shares in Staybrite and the carrying value of Allbrite at 1 April 2009 and 30 September 2009. (8 marks)

(b)

Prepare a consolidated income statement for the Holdrite Group for the year to 30 September 2009. (15 marks) Show the movement on the consolidated retained earnings attributable to Holdrite for the year to 30 September 2009. (2 marks) (25 marks)

(c )

5.

Hedra Hedra, a public listed company, acquired the following investments: (i)

On 1 October 2008, 72 million shares in Salvador for an immediate cash payment of $195 million. Hedra agreed to pay further consideration on 30 September 2009 if the post acquisition profits of Salvador exceeded an agreed figure at that date. Hedra has not accounted for this deferred payment as it did not believe it would be payable. The fair value of the payment at 1 October 2008 was $49million. Salvador also accepted a $50 million 8% loan from Hedra at the date of its acquisition.

(ii)

On 1 April 2009, 40 million shares in Aragon by way of a share exchange of two shares in Hedra for each acquired share in Aragon. The share market value of Hedra ‘s shares at the date of this share exchange was $2.50. Hedra has not yet recorded the acquisition of the investment in Aragon.

The summarised statements of financial position of the three companies as at 30 September 2009 are: Hedra $m Non–current assets Property, plant and equipment Investments – in Salvador – other

$m

$m

Salvador $m

358 245 45 648

240 nil nil 240

Aragon $m

$m 270 nil nil 270

Continued on next page

24

Downloaded by isavic Alsina ([email protected])

lOMoARcPSD|4664814

LSB_F7_Rev Kit:297mm x 210mm

28/10/09

13:36

Page 25

QUESTIONS Hedra $m Current assets Inventories Trade receivables Cash and bank

$m

130 142 nil

Equity and liabilities Ordinary share capital ($1 each) Reserves: Share premium Revaluation Retained earnings

$m

Salvador $m

80 97 4

Current liabilities Trade payables Bank overdraft Current tax payable

200 470

400

120

100

50 nil 60

45

118 12 50

nil nil 300 110 230

50 nil

50

141 nil nil 180 920

Total equity and liabilities

110 70 20 181 421

40 15 240

nil 45

$m

272 920

295 695 Non–current liabilities 8% loan note Deferred tax

Aragon $m

300 400 nil nil

nil

40 nil 30 141 421

70 470

The following information is relevant. (a)

Fair value adjustments and revaluations: (i) Hedra’s accounting policy for land and buildings is that they should be carried at their fair values. The fair value of Salvador’s land at the date of acquisition was $20 million in excess of its carrying value. By 30 September 2009 this excess had increased by a further $5 million. Salvador’s buildings did not require any fair value adjustments. The fair value of Hedra’s own land and buildings at 30 September 2009 was $12 million in excess of its carrying value in the above statement of financial position. (ii) The fair value of some of Salvador’s plant at the date of acquisition was $20 million in excess of its carrying value and had a remaining life of four years (straight–line depreciation is used). (iii)At the date of acquisition Salvador had unrelieved tax losses of $40 million from previous years. Salvador had not accounted for these as a deferred tax asset as its directors did not believe the company would be sufficiently profitable in the near future. However, the directors of Hedra were confident that these losses would be utilised and accordingly they should be recognised as a deferred tax asset. By 30 September 2009 the group had not yet utilised any of these losses. The income tax rate is 25%.

(b)

The retained earnings of Salvador and Aragon at 1 October 2008, as reported in their separate financial statements, were $20 million and $200 million respectively. All profits are deemed to accrue evenly throughout the year.

(c)

Hedra’s policy is to value non-controlling interests at their fair values. The Directors of Hedra assessed the fair value of the non-controlling interest in Salvador at the date of acquisition to be $110 million.

(d)

An impairment test on 30 September 2009 showed that consolidated goodwill should be written down by $20million. Hedra has applied IFRS 3 Business combinations since the acquisition of Salvador.

25

Downloaded by isavic Alsina ([email protected])

lOMoARcPSD|4664814

LSB_F7_Rev Kit:297mm x 210mm

28/10/09

13:36

Page 26

FINANCIAL REPORTING (INTERNATIONAL) (e)

The investment in Aragon has not suffered any impairment.

Required Prepare the consolidated statement of financial position of Hedra as at 30 September 2009.

6.

(25 marks)

Hosterling Hosterling purchased the following equity investments: On 1 October 2008: 80% of the issued share capital of Sunlee. The acquisition was through a share exchange of three shares in Hosterling for every five shares in Sunlee. The market price of Hosterling's shares at 1 October 2008 was $5 per share. On 1 July 2009: 6 million shares in Amber paying $3 per share in cash and issuing to Amber's shareholders 6% (actual and effective rate) loan notes on the basis of $100 loan note for every 100 shares acquired. The summarised income statements for the three companies for the year ended 30 September 2009 are:

Revenue Cost of sales Gross profit/(loss) Other income (note (i)) Distribution costs Administrative expenses Finance costs Profit/(loss) before tax Income tax (expense)/credit Profit/(loss) for the year

Hosterling $'000 105,000 (68,000) 37,000 400 (4,000) (7,500) (1,200) 24,700 (8,700) 16,000

Sunlee $'000 62,000 (36,500) 25,500 nil (2,000) (7,000) (900) 15,600 (2,600) 13,000

Amber $'000 50,000 (61,000) (11,000) nil (4,500) (8,500) nil (24,000) 4,000 (20,000)

The following information is relevant: (i)

The other income is a dividend received from Sunlee on 31 March 2009.

(ii)

The details of Sunlee's and Amber's share capital and reserves at 1 October 2008 were:

Equity shares of $1 each Retained earnings (iii)

Sunlee $'000 20,000 18,000

Amber $'000 15,000 35,000

A fair value exercise was carried out at the date of acquisition of Sunlee with the following results:

Intellectual property Land Plant

Carrying amount $'000 18,000 17,000 30,000

Fair value $'000 22,000 20,000 35,000

The intellectual property is still in development. The fair values have not been reflected in Sunlee's financial statements. Plant depreciation is included in cost of sales. No fair value adjustments were required on the acquisition of Amber.

26

Downloaded by isavic Alsina ([email protected])

Remaining life (straight line) see below not applicable five years

lOMoARcPSD|4664814

LSB_F7_Rev Kit:297mm x 210mm

28/10/09

13:36

Page 27

QUESTIONS (iv)

In the year ended 30 September 2009 Hosterling sold goods to Sunlee at a selling price of $18 million. Hosterling made a profit of cost plus 25% on these sales. $7.5 million (at cost to Sunlee) of these goods were still in the inventories of Sunlee at 30 September 2009.

(v)

Impairment tests for both Sunlee and Amber were conducted on 30 September 2009. They concluded that the goodwill of Sunlee should be written down by $1.6 million and, due to its losses since acquisition, the investment in Amber was worth $21.5 million.

(vi)

All trading profits and losses are deemed to accrue evenly throughout the year.

(vii)

The non-controlling interest is measured using the proportion of net assets method.

Required (a)

Calculate the goodwill arising on the acquisition of Sunlee at 1 October 2008. (5 marks)

(b )

Calculate the carrying amount of the investment in Amber at 30 September 2009 under the equity method prior to the impairment test. (4 marks)

(c)

Prepare the consolidated income statement for the Hosterling Group for the year ended 30 September 2009. (16 marks) (25 marks)

7.

Horsefield Horsefield, a public company, acquired 90% of Sandfly's $1 ordinary shares on 1 April 2007 paying $3.00 per share. The balance on Sandfly's retained earnings at this date was $800,000, and the fair value of a 10% shareholding was $250,000. On 1 October 2008, Horsefield acquired 30% of Anthill's $1 ordinary shares for $3.50 per share. The statements of financial position of the three companies at 31 March 2009 are shown below. Horsefield Sandfly Anthill $'000 $'000 $'000 $'000 $'000 $'000 Non-current assets Property, plant and equipment 8,050 3,600 1,650 Investments 910 nil 4,000 12,050 4,510 1,650 Current assets Inventory 830 340 250 Accounts receivable 520 290 350 Bank 240 nil 100 630 700 1,590 Total assets 13,640 5,140 2,350 Equity and liabilities Equity Ordinary shares of $1 each 5,000 1,200 600 Reserves: Retained earnings b/f 6,000 1,400 800 Profit year to 31 March 2009 1,500 900 600 7,500 2,300 1,400 12,500 3,500 2,000

27

Downloaded by isavic Alsina ([email protected])

lOMoARcPSD|4664814

LSB_F7_Rev Kit:297mm x 210mm

28/10/09

13:36

Page 28

FINANCIAL REPORTING (INTERNATIONAL) Non-current liabilities 10% loan notes Current liabilities Accounts payable Taxation Overdraft

500

240

420 220 nil

960 250 190 640 13,640

Total equity and liabilities

nil 200 150 nil

1,400 5,140

350 2,350

The following information is relevant. (i)

Fair value adjustments On 1 April 2007 Sandfly owned a property that had a fair value of $120,000 in excess of its book value. The value of this property has not changed since acquisition. Just prior to its acquisition, Sandfly was successful in applying for a six-year licence to dispose of hazardous waste. The licence was granted by the government at no cost. However, Horsefield estimated that the licence was worth $180,000 at the date of acquisition.

(ii )

In January 2009 Horsefield sold goods to Anthill for $65,000. These were transferred at a mark up of 30% on cost. Two thirds of these goods were still in the inventory of Anthill at 31 March 2009.

(iii )

To facilitate the consolidation procedures the group insists that all intragroup current account balances are settled prior to the year-end. However, a cheque for $40,000 from Sandfly to Horsefield was not received until early April 2009. Intragroup balances are included in accounts receivable and payable as appropriate.

(iv)

There are no indications that goodwill has been impaired.

(v)

Anthill is to be treated as an associate of Horsefield.

(vi)

The full goodwill method is used to measure the non-controlling interest.

Required (a)

Prepare the consolidated statement of financial position of Horsefield as at 31 March 2009 in accordance with IFRS 3 Business combinations. (20 marks)

(b)

Discuss the matters to consider in determining whether an investment in another company constitutes associate status. (5 marks) (25 marks)

8.

Winger The following list of account balances relates to Winger at 31 March 2009. $'000 Sales revenue (note a) Cost of sales Distribution costs Administration expenses Lease rentals (note b) Loan interest paid Dividend paid Property at cost (note c) Plant and equipment cost Depreciation 1 April 2008 - plant and equipment

$'000 358,450

185,050 28,700 15,000 20,000 2,000 12,000 200,000 154,800

28

Downloaded by isavic Alsina ([email protected])

34,800

lOMoARcPSD|4664814

LSB_F7_Rev Kit:297mm x 210mm

28/10/09

13:36

Page 29

QUESTIONS

Development expenditure (note d) Profit on disposal of non-current assets Trade accounts receivable Inventories: 31 March 2009 Cash and bank Trade accounts payable Taxation: over provision in year to 31 March 2008 Equity shares of 25c each 8% loan notes (issued in 2006) Retained earnings 1 April 2008

$'000 30,000

$'000 45,000

55,000 28,240 10,660

741,450

29,400 2,200 150,000 50,000 71,600 741,450

The following notes are relevant. (a)

Included in sales revenue is $27 million, which relates to sales made to customers under sale or return agreements. The expiry date for the return of these goods is 30 April 2009. Winger has charged a mark-up of 20% on cost for these sales.

(b)

A lease rental of $20 million was paid on 1 April 2008. It is the first of five annual payments in advance for the rental of an item of equipment that has a cash purchase price of $80 million. The auditors have advised that this is a finance lease and have calculated the implicit interest rate in the lease as 12% per annum. Leased assets should be depreciated on a straight-line basis over the life of the lease.

(c)

On 1 April 2008 Winger acquired a new property at a cost of $200 million. For the purpose of calculating depreciation only, the asset has been separated into the following elements. Separate asset Land Heating system Lifts Building

Cost $'000 50,000 20,000 30,000 100,000

Life freehold 10 years 15 years 50 years

The depreciation of the elements of the property should be calculated on a straight-line basis. The new property replaced an existing one that was sold on the same date for $95 million. It had cost $50 million and had a carrying value of $80 million at the date of sale. The profit on this property has been calculated on the original cost. It had not been depreciated on the basis that the depreciation charge would not be material. Plant and machinery is depreciated at 20% on the reducing balance basis. (d)

The figure for development expenditure in the list of account balances represents the amounts deferred in previous years in respect of the development of a new product. Unfortunately, during the current year, the government has introduced legislation which effectively bans this type of product. As a consequence of this the project has been abandoned. The directors of Winger are of the opinion that writing off the development expenditure, as opposed to its previous deferment, represents a change of accounting policy and therefore wish to treat the write off as a prior period adjustment.

(e)

A provision for income tax for the year to 31 March 2009 of $15 million is required.

Required (a)

Prepare Winger’s income statement for the year to 31 March 2009, along with the changes in retained earnings from the statement of changes in equity. (9 marks)

(b)

Prepare a statement of financial position as at 31 March 2009 in accordance with International Financial Reporting Standards as far as the information permits. (11 marks)

(c )

Discuss the acceptability of the company's previous policy in respect of non-depreciation of property. (5 marks) (25 marks) 29

Downloaded by isavic Alsina ([email protected])

lOMoARcPSD|4664814

LSB_F7_Rev Kit:297mm x 210mm

28/10/09

13:36

Page 30

FINANCIAL REPORTING (INTERNATIONAL)

9.

Petra The following trial balance relates to Petra, a public listed company, at 30 September 2009: $'000 Revenue (note (i)) Cost of sales (note (i)) Distribution costs Administration expenses Loan interest paid Ordinary shares of 25 cents each fully paid Share premium Retained earnings 1 October 2008 6% Redeemable loan note (issued in 2007) Land and buildings at cost (land element $40 million) note (ii)) Plant and equipment at cost (note (iii)) Deferred development expenditure (note (iv)) Accumulated depreciation at 1 October 2008 – buildings – plant and equipment Accumulated amortisation of development expenditure at 1 October 2008 Income tax (note (v)) Deferred tax (note (v)) Trade receivables Inventories – 30 September 2009 Cash and bank Trade payables

$'000 197,800

114,000 17,000 18,000 1,500 40,000 12,000 34,000 50,000 100,000 66,000 40,000 16,000 26,000 8,000 1,000 15,000 24,000 21,300 11,000 413,800

15,000 413,800

The following notes are relevant. (i)

Included in revenue is $12 million for receipts that the company’s auditors have advised are commission sales. The costs of these sales, paid for by Petra, were $8 million. $3million of the profit of $4 million was attributable to and remitted to Sharma (the auditors have advised that Sharma is the principal for the transactions). Both the $8 million cost of sales and the $3 million paid to Sharma have been included in cost of sales.

(ii)

The buildings had an estimated life of 30 years when they were acquired and are being depreciated on the straight–line basis.

(iii)

Included in the trial balance figures for plant and equipment is plant that had cost $16 million and had accumulated depreciation of $6 million at 1 October 2008. Following a review of the company’s operations this plant was made available for sale at the beginning of the year. Negotiations with a broker have concluded that a realistic selling price of this plant will be $7.5 million and the broker will charge a commission of 8% of the selling price. The plant had not been sold by the year end. Plant is depreciated at 20% per annum using the reducing balance method. Depreciation of buildings and plant is charged to cost of sales.

(iv)

The development expenditure relates to the capitalised cost of developing a product called the Topaz. It had an original estimated life of five years. Production and sales of the Topaz started in October 2007. A review of the sales of the Topaz in late September 2009, showed them to be below forecast and an impairment test concluded that the fair value of the development costs at 30 September 2009 was only $18 million and the expected period of further sales (from this date) was only a further two years.

(v)

The balance on the income tax account in the trial balance is the under–provision in respect of the income tax liability for the year ended 30 September 2008. The directors have estimated the provision for income tax for the year ended 30 September 2009 to be $4 million and the required provision for deferred tax at 30 September 2009 is $17.6 million.

30

Downloaded by isavic Alsina ([email protected])

lOMoARcPSD|4664814

LSB_F7_Rev Kit:297mm x 210mm

28/10/09

13:36

Page 31

QUESTIONS Required Prepare for Petra: (a)

An income statement for the year ended 30 September 2009; and (10 marks)

(b)

A statement of financial position as at 30 September 2009 (10 marks) Note. A statement of changes in equity is NOT required. Disclosure notes are NOT required.

(c)

The directors hold options to purchase 24 million shares for a total of $7.2 million. The options were granted two years ago and have been correctly accounted for. The options do not affect your answer to (a) and (b) above. The average stock market value of Petra’s shares for the year ended 30 September 2009 can be taken as 90 cents per share.

Required A calculation of the basic and diluted earnings per share for the year ended 30 September 2009 (comparatives are not required) (5 marks) (25 marks)

10. Allgone The following trial balance relates to Allgone at 31 March 2009: $'000 Sales revenue (note (i)) Purchases Operating expenses Loan interest paid Preference dividend Land and buildings – at valuation (note (ii)) Plant and equipment – cost Software – cost 1 April 2006 Available-for-sale investment – value at 1 April 2008 (note (iii)) Depreciation 1 April 2008 – plant and equipment Depreciation 1 April 2008 – software Costs of fraud (note (iv)) Trade receivables Inventory – 1 April 2008 Bank Trade payables Ordinary shares of 25c each 10% Preference shares (redeemable) 12% Loan note (issued 1 July 2008) Deferred tax Revaluation reserve (relating to land and buildings and the investments) Retained earnings – 2008

$'000 236,200

127,850 12,400 2,400 1,000 130,000 84,300 10,000 12,000 24,300 6,000 32,000 23,000 19,450

454,400

350 15,200 60,000 20,000 40,000 3,000 45,000 4,350 454,400

The following notes are relevant: (i)

Sales include $8 million for goods sold in March 2009 for cash to Funders, a merchant bank. The cost of these goods was $6 million. Funders has the option to require Allgone to repurchase these goods within one month of the year-end at their original selling price plus a facilitating fee of $250,000.

31

Downloaded by isavic Alsina ([email protected])

lOMoARcPSD|4664814

LSB_F7_Rev Kit:297mm x 210mm

28/10/09

13:36

Page 32

FINANCIAL REPORTING (INTERNATIONAL) The inventory at 31 March 2009 was counted at a cost value of $8·5 million. This includes $500,000 of slow moving inventory that is expected to be sold for a net $300,000. (ii)

Non-current assets On 1 April 2008 Allgone revalued its land and buildings. The details are: Cost 1 April 2003 $'000 20,000 80,000

Land Building

Valuation 1 April 2008 $'000 25,000 105,000

The building had an estimated life of 40 years when it was acquired and this has not changed as a result of the revaluation. Depreciation is on a straight-line basis. The surplus on the revaluation has been added to the revaluation reserve, but no other movements on the revaluation reserve have been recorded. Plant and equipment is depreciated at 20% per annum on the reducing balance basis. Software is depreciated by the sum of the digits method over a 5-year life. (iii)

The investment represents 7.5% of the ordinary share capital of Wondaworld. Changes in the fair value of the investment are recognised in accordance with IAS 39. Prior to 1 April 2008, an increase of $5million in the fair value had been recognised. The stock market price of Wondaworld’s ordinary shares was $2.50 each on 1 April 2008 and by 31 March 2009 this had fallen to $2.25.

(iv)

A $32m fraud was discovered during this financial year. A senior employee of the company, who left in January 2008, had diverted investment funds into his private bank account. The fraud was discovered by the employee’s replacement in April 2008. It is unlikely that any of the funds will be recovered. Allgone has now implemented tighter procedures to prevent such a fraud recurring. The company has been advised that this loss will not qualify for any tax relief. The directors have asked if this loss can be treated as an extraordinary item.

(v)

The directors have estimated the provision for income tax for the year to 31 March 2009 at $11.3 million. The deferred tax liability at 31 March 2009 is to be adjusted to reflect the tax base of the company’s net assets being $16 million less than their carrying values. The rate of income tax is 30%. The movement on deferred tax should be charged to the income statement.

(vi)

The finance charge relating to the preference shares is $2,000,000 per annum.

Required In accordance with International Accounting Standards and International Financial Reporting Standards as far as the information permits, prepare: (a)

The income statement and statement of comprehensive income of Allgone for the year to 31 March 2009 (7 marks)

(b)

The statement of Changes in Equity for the year to 31 March 2009 (5 marks)

(c)

A statement of financial position as at 31 March 2009 (13 marks) Notes to the financial statements are not required.

(25 marks)

11. Tadeon The following trial balance relates to Tadeon, a publicly listed company, at 30 September 2009: $'000 Revenue Cost of sales Operating expenses Loan interest paid (note (i)) Rental of vehicles (note (ii)) 32

Downloaded by isavic Alsina ([email protected])

118,000 40,000 1,000 6,200

$'000 277,800

lOMoARcPSD|4664814

LSB_F7_Rev Kit:297mm x 210mm

28/10/09

13:36

Page 33

QUESTIONS $'000 Investment income 25 year leasehold property at cost (note (iii)) Plant and equipment at cost Investments at amortised cost Accumulated depreciation at 1 October 2008

$'000 2,000

225,000 181,000 42,000 – leasehold property – plant and equipment

36,000 85,000 150,000 18,600 50,000 12,000

Equity shares of 20 cents each fully paid Retained earnings at 1 October 2008 2% Loan note (note (i)) Deferred tax balance 1 October 2008 (note (iv)) Trade receivables Inventories at 30 September 2009 Bank Trade payables Suspense account (note (v))

53,500 33,300

700,000

1,900 18,700 48,000 700,000

The following notes are relevant: (i)

The loan note was issued on 1 October 2008. It is redeemable on 30 September 2013 at a large premium (in order to compensate for the low nominal interest rate). The finance department has calculated that the effective interest rate on the loan is 5.5% per annum.

(ii)

The rental of the vehicles relates to two separate contracts. These have been scrutinised by the finance department and they have come to the conclusion that $5 million of the rentals relate to a finance lease. The finance lease was entered into on 1 October 2008 (the date the $5 million was paid) for a four year period. The vehicles had a fair value of $20 million (straight-line depreciation should be used) at 1 October 2008 and the lease agreement requires three further annual payments of $6 million each on the anniversary of the lease. The interest rate implicit in the lease is to be taken as 10% per annum. (Note: you are not required to calculate the present value of the minimum lease payments.) The other contract is an operating lease and should be charged to operating expenses. Other plant and equipment is depreciated at 121/2% per annum on the reducing balance basis. All depreciation of property, plant and equipment is charged to cost of sales.

(iii)

On 30 September 2009 the leasehold property was revalued to $200 million. The directors wish to incorporate this valuation into the financial statements.

(iv)

The directors have estimated the provision for income tax for the year ended 30 September 2009 at $38 million. At 30 September 2009 there were $74 million of taxable temporary differences, of which $20 million related to the revaluation of the leasehold property (see (iii) above). The income tax rate is 20%.

(v)

The suspense account balance can be reconciled from the following transactions: The payment of a dividend in October 2008. This was calculated to give a 5% yield on the company's share price of 80 cents as at 30 September 2008. The net receipt in March 2009 of a fully subscribed rights issue of one new share for every three held at a price of 32 cents each. The expenses of the share issue were $2 million and should be charged to share premium. Note. The cash entries for these transactions have been correctly accounted for.

33

Downloaded by isavic Alsina ([email protected])

lOMoARcPSD|4664814

LSB_F7_Rev Kit:297mm x 210mm

28/10/09

13:36

Page 34

FINANCIAL REPORTING (INTERNATIONAL) Required Prepare for Tadeon: (a)

A statement of comprehensive income for the year ended 30 September 2009; and (10 marks)

(b)

A statement of financial position as at 30 September 2009. (15 marks)

Note. A statement of changes in equity is not required. Disclosure notes are not required.

(25 marks)

12. Kala The following trial balance relates to Kala, a publicly listed company, at 31 March 2009:

Land and buildings at cost (note (i)) Plant – at cost (note (i)) Investment properties – valuation at 1 April 2008 (note (i)) Purchases Operating expenses Loan interest paid Rental of leased plant (note (ii)) Dividends paid Inventory at 1 April 2008 Trade receivables Revenue Income from investment property Equity shares of $1 each fully paid Retained earnings at 1 April 2008 8% (actual and effective) loan note (note (iii)) Accumulated depreciation at 1 April 2008 – buildings – plant Trade payables Deferred tax Bank

$'000 270,000 156,000 90,000 78,200 15,500 2,000 22,000 15,000 37,800 53,200 278,400

739,700

$'000

4,500 150,000 119,500 50,000 60,000 26,000 33,400 12,500 5,400 739,700

The following notes are relevant: (i)

The land and buildings were purchased on 1 April 1993. The cost of the land was $70 million. No land and buildings have been purchased by Kala since that date. 1 April 2008 Kala had its land and buildings professionally valued at $80 million and $175 million respectively. The directors wish to incorporate these values into the financial statements. The estimated life of the buildings was originally 50 years and the remaining life has not changed as a result of the valuation. Later, the valuers informed Kala that investment properties of the type Kala owned had increased in value by 7% in the year to 31 March 2009. Plant, other than leased plant (see below), is depreciated at 15% per annum using the reducing balance method. Depreciation of buildings and plant is charged to cost of sales.

(ii)

1 April 2008 Kala entered into a lease for an item of plant which had an estimated life of five years. The lease period is also five years with annual rentals of $22 million payable in advance from 1 April 2008. The plant is expected to have a nil residual value at the end of its life. If purchased this plant would have a cost of $92 million and be depreciated on a straight-line basis. The lessor includes a finance cost of 10% per annum when calculating annual rentals. (Note: you are not required to calculate the present value of the minimum lease payments.)

34

Downloaded by isavic Alsina ([email protected])

lOMoARcPSD|4664814

LSB_F7_Rev Kit:297mm x 210mm

28/10/09

13:36

Page 35

QUESTIONS (iii)

The loan note was issued on 1 July 2008 with interest payable six monthly in arrears.

(iv)

The provision for income tax for the year to 31 March 2009 has been estimated at $28.3 million. The deferred tax provision at 31 March 2009 is to be adjusted to a credit balance of $14.1 million.

(v )

The inventory at 31 March 2009 was valued at $43.2 million.

Required: Prepare for Kala: (a)

A statement of comprehensive income for Kala for the year ended 31 March 2009. (10 marks)

(b)

A statement of changes in equity for the year ended 31 March 2009. (4 marks)

(c)

A statement of financial position as at 31 March 2009. (11 marks) (25 marks)

13. Telenorth The following trial balance relates to Telenorth at 30 September 2009. $'000 Sales revenue Inventory 1 October 2008 Purchases Distribution expenses Administration expenses Loan note interest paid Interim dividends: ordinary preference Investment income 25 year leasehold building – cost Plant and equipment – cost Computer system – cost Investments at valuation Depreciation 1 October 2008 (note (b)) Leasehold building Plant and equipment Computer system Trade accounts receivable (note (c)) Bank overdraft Trade accounts payable Deferred tax (note (d)) Ordinary shares of $1 each Suspense account (note (e)) 6% loan notes (issued 1 October 2008) 8% preference shares (redeemable) Revaluation surplus (note (d)) Retained earnings 1 October 2008

$'000 283,460

12,400 147,200 22,300 34,440 300 2,000 480 1,500 56,250 55,000 35,000 34,500 18,000 12,800 9,600 35,700

435,570

1,680 17,770 5,200 20,000 26,000 10,000 12,000 3,400 14,160 435,570

35

Downloaded by isavic Alsina ([email protected])

lOMoARcPSD|4664814

LSB_F7_Rev Kit:297mm x 210mm

28/10/09

13:36

Page 36

FINANCIAL REPORTING (INTERNATIONAL) The following notes are relevant. (a)

An inventory count was not conducted by Telenorth until 4 October 2009 due to operational reasons. The value of the inventory on the premises at this date was $16 million at cost. Between the year-end and the inventory count the following transactions have been identified. $ Normal sales at a mark up on cost of 40% 1,400,000 Sales on a sale or return basis at a mark up on cost of 30% 650,000 Goods received at cost 820,000 All sales and purchases had been correctly recorded in the period in which they occurred.

(b)

Telenorth has the following depreciation policy. Leasehold building – straight-line Plant and equipment – five years straight line with residual values estimated at $5,000,000 Computer system – 40% per annum reducing balance Depreciation of the leasehold building and plant is treated as cost of sales; depreciation of the computer system is an administration cost.

(c)

The outstanding account receivable of a major customer amounting to $12 million was factored to Kwikfinance on 1 September 2009. The terms of the factoring were as follows. (i)

Kwikfinance paid 80% of the outstanding account to Telenorth immediately

(ii)

The balance will be paid (less the charges below) when the account is collected in full. Any amount of the account outstanding after four months will be transferred back to Telenorth at its full book value.

(iii)

Kwikfinance will charge 1.0% per month of the net amount owing from Telenorth at the beginning of each month. Kwikfinance had not collected any of the amounts receivable by the year end. Telenorth debited the cash from Kwikfinance to its bank account and removed the account receivable from its sales ledger. It has prudently charged the difference as an administration cost.

(d)

A provision for income tax of $23.4 million for the year to 30 September 2009 is required. The deferred tax liability is to be increased by $2.2 million, of which $1 million is to be charged direct to the revaluation surplus.

(e)

The suspense account contains the proceeds of two share issues.

(f)

(i)

The exercise of all the outstanding directors' share options of four million shares on 1 October 2008 at $2 each

(ii)

A fully subscribed rights issue on 1 July 2009 of 1 for 4 held at a price of $3 each. The stock market price of Telenorth's shares immediately before the rights issue was $4.

The finance charge relating to the preference shares is equal to the dividend payable.

Required (a) (i) The income statement of Telenorth for the year to 30 September 2009. (8 marks) (ii) A statement of financial position as at 30 September 2009 in accordance with International Financial Reporting Standards as far as the information permits. (12 marks) Notes to the financial statements are not required. (b)

Calculate the earnings per share in accordance with IAS 33 for the year to 30 September 2009 (ignore comparatives). (5 marks) (25 marks)

36

Downloaded by isavic Alsina ([email protected])

lOMoARcPSD|4664814

LSB_F7_Rev Kit:297mm x 210mm

28/10/09

13:36

Page 37

QUESTIONS

14. Tourmalet The following extracted balances relate to Tourmalet at 30 September 2009: $000 Ordinary shares of 20 cents each Retained earnings at 1 October 2008 Revaluation reserve at 1 October 2008 6% Redeemable preference shares 2011 Trade accounts payable Tax Land and buildings – at valuation (note (iii)) Plant and equipment – cost (note (v)) Investment property – valuation at 1 October 2008 (note (iv)) 1 October 2008 – land and buildings Depreciation 1 October 2008 – plant and equipment Trade accounts receivable Inventory – 1 October 2008 Bank Sales revenue (note (i)) Investment income (from properties) Purchases Distribution expenses Administration expenses Interim preference dividend Ordinary dividend paid

$000 50,000 47,800 18,500 30,000 35,300 2,100

150,000 98,600 10,000 9,000 24,600 31,200 26,550 3,700 313,000 1,200 158,450 26,400 23,200 900 2,500 531,500

531,500

The following notes are relevant: (i)

Sales revenue includes $50 million for an item of plant sold on 1 June 2009. The plant had a book value of $40 million at the date of its sale, which was charged to cost of sales. On the same date,Tourmalet entered into an agreement to lease back the plant for the next five years (being the estimated remaining life of the plant) at a cost of $14 million per annum payable annually in arrears. An arrangement of this type is deemed to have a financing cost of 12% per annum. No depreciation has been charged on the item of plant in the current year.

(ii)

The inventory at 30 September 2009 was valued at cost of $28·5 million. This includes $4·5 million of slow moving goods. Tourmalet is trying to sell these to another retailer but has not been successful in obtaining a reasonable offer. The best price it has been offered is $2 million.

(iii)

On 1 October 2005 Tourmalet had its land and buildings revalued by a firm of surveyors at $150 million, with $30 million of this attributed to the land. At that date the remaining life of the building was estimated to be 40 years. These figures were incorporated into the company’s books. There has been no significant change in property values since the revaluation. $500,000 of the revaluation reserve will be realised in the current year as a result of the depreciation of the buildings and should be transferred to retained earnings.

(iv)

Details of the investment property are: Value – 1 October 2008 Value – 30 September 2009

$10 million $9·8 million

The company adopts the fair value method in IAS 40 ‘Investment Property’ of valuing its investment property. (v)

Plant and equipment (other than that referred to in note (i) above) is depreciated at 20% per annum on the reducing balance basis. All depreciation is to be charged to cost of sales. 37

Downloaded by isavic Alsina ([email protected])

lOMoARcPSD|4664814

LSB_F7_Rev Kit:297mm x 210mm

28/10/09

13:36

Page 38

FINANCIAL REPORTING (INTERNATIONAL) (vi)

The above balances contain the results of Tourmalet’s car retailing operations which ceased on 31 December 2008 due to mounting losses. The results of the car retailing operation, which is to be treated as a discontinued operation, for the year to 30 September 2009 are:

Sales Cost of sales Operating expenses (4,000 less 800 tax repayment due)

$000 15,200 16,000 3,200

The operating expenses are included in administration expenses in the trial balance. Tourmalet is still paying rentals for the lease of its car showrooms. The rentals are included in operating expenses. Tourmalet is hoping to use the premises as an expansion of its administration offices. This is dependent on obtaining planning permission from the local authority for the change of use, however this is very difficult to obtain. Failing this, the best option would be early termination of the lease which will cost $1.5 million in penalties. This amount has not been provided for. (vii)

The balance on the taxation account in the trial balance is the result of the settlement of the previous year’s tax charge. The directors have estimated the provision for income tax for the year to 30 September 2009 at $9.2 million.

(viii)

The preference shares will be redeemed at par. The finance cost is equivalent to the annual dividend.

Required (a)

Comment on the substance of the sale of the plant and the directors’ treatment of it. (5 marks)

(b)

Prepare the income statement. (17 marks)

(c)

A statement of changes in equity for Tourmalet for the year to 30 September 2009 in accordance with current International Accounting Standards. (3 marks) Note: A statement of financial position is NOT required. Disclosure notes are NOT required. (25 marks)

15. Wellmay The summarised draft financial statements of Wellmay are shown below. INCOME STATEMENT YEAR ENDED 31 MARCH 2009 $’000 4,200 (2,700) 1,500 (470 20 (55) 995 (360) 635

Revenue (note (i)) Cost of sales (note (ii)) Gross profit Operating expenses Investment property rental income Finance costs Profit before tax Income tax Profit for the year STATEMENT OF FINANCIAL POSITION AS AT 31 MARCH 2009 $’000 Assets Non-current assets Property, plant and equipment (note (iii)) Investment property (note (iii))

38

Downloaded by isavic Alsina ([email protected])

$’000

4,200 400 4,600

lOMoARcPSD|4664814

LSB_F7_Rev Kit:297mm x 210mm

28/10/09

13:36

Page 39

QUESTIONS $’000 Current assets Total assets Equity and liabilities Equity Equity shares of 50 cents each (note (vii)) Reserves: Retained earnings (note (iv)) Revaluation reserve

$’000 1,400 6,000

1,200 2,850 350

Non-current liabilities 8% convertible loan note (2012) (note (v)) Deferred tax (note (vi)) Current liabilities Total equity and liabilities

600 180

3,200 4,400

780 820 6,000

The following information is relevant to the draft financial statements: (i)

Revenue includes $500,000 for the sale on 1 April 2008 of maturing goods to Westwood. The goods had a cost of $200,000 at the date of sale. Wellmay can repurchase the goods on 31 March 2010 for $605,000 (based on achieving a lender’s return of 10% per annum) at which time the goods are estimated to have a value of $750,000.

(ii)

Past experience shows that after the end of the reporting period the company often receives unrecorded invoices for materials relating to the previous year. As a result of this an accrued charge of $75,000 for contingent costs has been included in cost of sales and as a current liability.

(iii)

Non-current assets: Wellmay owns two properties. One is a factory (with office accommodation) used by Wellmay as a production facility and the other is an investment property that is leased to a third party under an operating lease. Wellmay revalues all its properties to current value at the end of each year and uses the fair value model in IAS 40 Investment property. Relevant details of the fair values of the properties are:

Valuation 31 March 2008 Valuation 31 March 2009

Factory $’000 1,200 1,350

Investment property $’000 400 375

The valuations at 31 March 2009 have not yet been incorporated into the financial statements. Factory depreciation for the year ended 31 March 2009 of $40,000 was charged to cost of sales. As the factory includes some office accommodation, 20% of this depreciation should have been charged to operating expenses. (iv)

The balance of retained earnings is made up of:

Balance b/f 1 April 2008 Profit for the year Dividends paid during year ended 31 March 2009

(v)

$’000 2,615 635 (400) 2,850

8% convertible loan note (2012) On 1 April 2008 an 8% convertible loan note with a nominal value of $600,000 was issued at par. It is redeemable on 31 March 2012 at par or it may be converted into equity shares of Wellmay on the basis of 100 new shares for each $200 of loan note. An equivalent loan note without the conversion option would have carried an interest rate of 10%. Interest of $48,000 has been paid on the loan and charged as a finance cost. 39

Downloaded by isavic Alsina ([email protected])

lOMoARcPSD|4664814

LSB_F7_Rev Kit:297mm x 210mm

28/10/09

13:36

Page 40

FINANCIAL REPORTING (INTERNATIONAL) The present value of $1 receivable at the end of each year, based on discount rates of 8% and 10% are:

End of year 1 2 3 4

8% 0·93 0·86 0·79 0·73

10% 0·91 0·83 0·75 0·68

(vi)

The carrying amounts of Wellmay’s net assets at 31 March 2009 are $600,000 higher than their tax base. The rate of taxation is 35%. The income tax charge of $360,000 does not include the adjustment required to the deferred tax provision which should be charged in full to the income statement.

(vii)

Bonus/scrip issue: On 15 March 2009,Wellmay made a bonus issue from retained earnings of one share for every four held. The issue has not been recorded in the draft financial statements.

Required Redraft the financial statements of Wellmay, including a statement of other comprehensive income and a statement of changes in equity, for the year ended 31 March 2009 reflecting the adjustments required by notes (i) to (vii) above. Note: Calculations should be made to the nearest $’000. (25 marks)

16. Peterlee (a)

The IASB’s Framework for the preparation and presentation of financial statements (Framework) sets out the concepts that underlie the preparation and presentation of financial statements that external users are likely to rely on when making economic decisions about an entity. Explain the purpose and authoritative status of the Framework. (5 marks)

(b)

Of particular importance within the Framework are the definitions and recognition criteria for assets and liabilities. Define assets and liabilities and explain the important aspects of their definitions. Explain why these definitions are of particular importance to the preparation of an entity’s statement of financial position and income statement. (8 marks) (13 marks)

40

Downloaded by isavic Alsina ([email protected])

lOMoARcPSD|4664814

LSB_F7_Rev Kit:297mm x 210mm

28/10/09

13:36

Page 41

QUESTIONS

17. Derringdo (a)

Revenue recognition is the process by which companies decide when and how much income should be included in the income statement. It is a topical area of great debate in the accounting profession. The IASB looks at revenue recognition from conceptual and substance points of view. There are occasions where a more traditional approach to revenue recognition does not entirely conform to the IASB guidance; indeed neither do some International Accounting Standards.

Required Explain the implications that the IASB’s Framework for the Preparation and Presentation of Financial Statements (Framework) and the application of substance over form have on the recognition of income. Give examples of how this may conflict with traditional practice and some accounting standards. (6 marks) (b)

Derringdo sells goods supplied by Gungho. The goods are classed as A grade (perfect quality) or B grade, having slight faults. Derringdo sells the A grade goods acting as an agent for Gungho at a fixed price calculated to yield a gross profit margin of 50%. Derringdo receives a commission of 12·5% of the sales it achieves for these goods. The arrangement for B grade goods is that they are sold by Gungho to Derringdo and Derringdo sells them at a gross profit margin of 25%. The following information has been obtained from Derringdo’s financial records:

Inventory held on premises 1 April 2008 Goods from Gungho year to 31 March 2009 Inventory held on premises 31 March 2009

– A grade – B grade – A grade – B grade – A grade – B grade

$'000 2,400 1,000 18,000 8,800 2,000 1,250

Required Prepare the income statement extracts for Derringdo for the year to 31 March 2009 reflecting the above information. (5 marks) (c)

Derringdo acquired an item of plant at a gross cost of $800,000 on 1 October 2008. The plant has an estimated life of 10 years with a residual value equal to 15% of its gross cost. Derringdo uses straight-line depreciation on a time apportioned basis. The company received a government grant of 30% of its cost price at the time of its purchase. The terms of the grant are that if the company retains the asset for four years or more, then no repayment liability will be incurred. If the plant is sold within four years a repayment on a sliding scale would be applicable. The repayment is 75% if sold within the first year of purchase and this amount decreases by 25% per annum. Derringdo has no intention to sell the plant within the first four years. Derringdo’s accounting policy for capital based government grants is to treat them as deferred credits and release them to income over the life of the asset to which they relate.

Required (i)

Discuss whether the company’s policy for the treatment of government grants meets the definition of a liability in the IASB’s Framework; and (3 marks)

(ii)

Prepare extracts of Derringdo’s financial statements for the year to 31 March 2009 in respect of the plant and the related grant: – applying the company’s policy; – in compliance with the definition of a liability in the Framework. Your answer should consider whether the sliding scale repayment should be used in determining the deferred credit for the grant. (6 marks)

(d)

Derringdo sells carpets from several retail outlets. In previous years the company has undertaken responsibility for fitting the carpets in customers’ premises. Customers pay for the carpets at the time they are ordered. The average length of time from a customer ordering a carpet to its fitting is 14 days. 41

Downloaded by isavic Alsina ([email protected])

lOMoARcPSD|4664814

LSB_F7_Rev Kit:297mm x 210mm

28/10/09

13:36

Page 42

FINANCIAL REPORTING (INTERNATIONAL) In previous years, Derringdo had not recognised a sale in income until the carpet had been successfully fitted as the rectification costs of any fitting error would be expensive. From 1 April 2008 Derringdo changed its method of trading by sub-contracting the fitting to approved contractors. Under this policy the sub-contractors are paid by Derringdo and they (the subcontractors) are liable for any errors made in the fitting. Because of this Derringdo is proposing to recognise sales when customers order and pay for the goods, rather than when they have been fitted. Details of the relevant sales figures are:

Sales made in retail outlets for the year to 31 March 2009 Sales value of carpets fitted in the 14 days to 14 April 2008 Sales value of carpets fitted in the 14 days to 14 April 2009

$'000 23,000 1,200 1,600

Note: the sales value of carpets fitted in the 14 days to 14 April 2008 are not included in the annual sales figure of $23 million, but those for the 14 days to 14 April 2009 are included. Required Discuss whether the above represents a change of accounting policy, and, based on your discussion, calculate the amount that you would include in sales revenue for carpets in the year to 31 March 2009. (5 marks) (25 marks)

18. Broadoak The broad principles of accounting for property, plant and equipment involve distinguishing between capital and revenue expenditure, measuring the cost of assets, determining how they should be depreciated and dealing with the problems of subsequent measurement and subsequent expenditure. IAS 16 Property, plant and equipment has the intention of improving consistency in these areas. Required (a)

Explain: (i) How the initial cost of property, plant and equipment should be measured (4 marks) (ii) The circumstances in which subsequent expenditure on those assets should be capitalised. (3 marks)

(b)

Explain IAS 16's requirements regarding the revaluation of non-current assets and the accounting treatment of surpluses and deficits on revaluation and gains and losses on disposal. (8 marks)

(c)

(i) Broadoak has recently purchased an item of plant from Plantco, the details of this are: $ Basic list price of plant Trade discount applicable to Broadoak Ancillary costs: Shipping and handling costs Estimated pre-production testing Maintenance contract for three years Site preparation costs: electrical cable installation concrete reinforcement own labour costs

$ 240,000 12.5% on list price 2,750 12,500 24,000

14,000 4,500 7,500 26,000

Broadoak paid for the plant (excluding the ancillary costs) within four weeks of order, thereby obtaining an early settlement discount of 3%.

42

Downloaded by isavic Alsina ([email protected])

lOMoARcPSD|4664814

LSB_F7_Rev Kit:297mm x 210mm

28/10/09

13:37

Page 43

QUESTIONS Broadoak had incorrectly specified the power loading of the original electrical cable to be installed by the contractor. The cost of correcting this error of $6,000 is included in the above figure of $14,000. The plant is expected to last for 10 years. At the end of this period there will be compulsory costs of $15,000 to dismantle the plant and $3,000 to restore the site to its original use condition. Required Calculate the amount at which the plant will be measured at recognition. (Ignore discounting.) (5 marks) (ii) Broadoak acquired a 12 year lease on a property on 1 October 2007 at a cost of $240,000. The company policy is to revalue its properties to their market values at the end of each year. Accumulated amortisation is eliminated and the property is restated to the revalued amount. Annual amortisation is calculated on the carrying values at the beginning of the year. The market values of the property on 30 September 2008 and 2009 were $231,000 and $175,000 respectively. The existing balance on the revaluation surplus at 1 October 2007 was $50,000. This related to some non-depreciable land whose value had not changed significantly since 1 October 2007. Required Prepare extracts of the financial statements of Broadoak for the years to 30 September 2008 and 2009 in respect of the leasehold property. (5 marks) (25 marks)

19. Wilderness (a)

IAS 36 Impairment of assets was issued in June 1998 and subsequently amended in March 2004. Its main objective is to prescribe the procedures that should ensure that an entity’s assets are included in its balance sheet at no more than their recoverable amounts. Where an asset is carried at an amount in excess of its recoverable amount, it is said to be impaired and IAS 36 requires an impairment loss to be recognised.

Required (i) Define an impairment loss explaining the relevance of fair value less costs to sell and value in use; and state how frequently assets should be tested for impairment; (6 marks) Note: your answer should NOT describe the possible indicators of an impairment. (ii) Explain how an impairment loss is accounted for after it has been calculated. (5 marks) (b)

The assistant financial controller of the Wilderness group, a public listed company, has identified the matters below which she believes may indicate an impairment to one or more assets: (i) Wilderness owns and operates an item of plant that cost $640,000 and had accumulated depreciation of $400,000 at 1 October 2008. It is being depreciated at 12% on cost. On 1 April 2009 (exactly half way through the year) the plant was damaged when a factory vehicle collided into it. Due to the unavailability of replacement parts, it is not possible to repair the plant, but it still operates, albeit at a reduced capacity. Also it is expected that as a result of the damage the remaining life of the plant from the date of the damage will be only two years. Based on its reduced capacity, the estimated present value of the plant in use if $150,000. The plant has a current disposal value of $20,000 (which will be nil in two years’ time), but Wilderness has been offered a trade–in value of $180,000 against a replacement machine which has a cost of $1 million (there would be no disposal costs for the replaced plant). Wilderness is reluctant to replace the plant as it is worried about the long–term demand for the produce produced by the plant. The trade–in value is only available if the plant is replaced.

Required Prepare extracts from the statement of financial position and income statement of Wilderness in respect of the plant for the year ended 30 September 2009. Your answer should explain how you arrived at your figures. (7 marks) 43

Downloaded by isavic Alsina ([email protected])

lOMoARcPSD|4664814

LSB_F7_Rev Kit:297mm x 210mm

28/10/09

13:37

Page 44

FINANCIAL REPORTING (INTERNATIONAL) (ii) On 1 April 2008 Wilderness acquired 100% of the share capital of Mossel, whose only activity is the extraction and sale of spa water. Mossel had been profitable since its acquisition, but bad publicity resulting from several consumers becoming ill due to a contamination of the spa water supply in April 2009 has led to unexpected losses in the last six months. The carrying amounts of Mossel’s assets at 30 September 2009 are:

Brand (Quencher – see below) Land containing spa Purifying and bottling plant Inventories

$’000 7,000 12,000 8,000 5,000 32,000

The source of the contamination was found and it has now ceased. The company originally sold the bottled water under the brand name of ‘Quencher’, but because of the contamination it has rebranded its bottled water as ‘Phoenix’. After a large advertising campaign, sales are now starting to recover and are approaching previous levels. The value of the brand in the balance sheet is the depreciated amount of the original brand name of ‘Quencher’. The directors have acknowledged that $1.5 million will have to be spent in the first three months of the next accounting period to upgrade the purifying and bottling plant. Inventories contain some old ‘Quencher’ bottled water at a cost of $2 million; the remaining inventories are labeled with the new brand ‘Phoenix’. Samples of all the bottled water have been tested by the health authority and have been passed as fit to sell. The old bottled water will have to be relabelled at a cost of $250,000, but is then expected to be sold at the normal selling price of (normal) cost plus 50%. Based on the estimated future cash flows, the directors have estimated that the value in use of Mossel at 30 September 2009, calculated according to the guidance in IAS 36, is $20 million. is no reliable estimate of the fair value less costs to sell of Mossel. Required Calculate the amounts at which the assets of Mossel should appear in the consolidated statement of financial position of Wilderness at 30 September 2009. Your answer should explain how you arrive at your figures. (7 marks) (25 marks)

20. Elite Leisure Elite Leisure is a private limited liability company that operates a single cruise ship. The ship was acquired on 1 October 2000. Details of the cost of the ship’s components and their estimated useful lives are: Component Ship’s fabric (hull, decks etc) Cabins and entertainment area fittings Propulsion system

Original cost ($ million) 300 150 100

Deprecation basis 25 years straight–line 12 years straight–line Useful life of 40,000 hours

At 30 September 2008 no further capital expenditure had been incurred on the ship. In the year ended 30 September 2008 the ship had experienced a high level of engine trouble which had cost the company considerable lost revenue and compensation costs. The measured expired life of the propulsion system at 30 September 2008 was 30,000 hours. Due to the unreliability of the engines, a decision was taken in early October 2008 to replace the whole of the propulsion system at a cost of $140 million. The expected life of the new propulsion system was 50,000 hours and in the year ended 30 September 2009 the ship had

44

Downloaded by isavic Alsina ([email protected])

lOMoARcPSD|4664814

LSB_F7_Rev Kit:297mm x 210mm

28/10/09

13:37

Page 45

QUESTIONS used its engines for 5,000 hours. At the same time as the propulsion system replacement, the company took the opportunity to do a limited upgrade to the cabin and entertainment facilities at a cost of $60 million and repaint the ship’s fabric at a cost of $20 million. After the upgrade of the cabin and entertainment area fittings it was estimated that their remaining life was five years (from the date of the upgrade). For the purpose of calculating depreciation, all the work on the ship can be assumed to have been completed on 1 October 2008. All residual values can be taken as nil. Required Calculate the carrying amount of Elite Leisure’s cruise ship at 30 September 2009 and its related expenditure in the income statement for the year ended 30 September 2009. Your answer should explain the treatment of each item. (12 marks)

21. Linnet (a)

Linnet is a large public listed company involved in the construction industry. Revenue on construction contracts is normally recognised by reference to the stage of completion of the contract. However, in certain circumstances, revenue is only recognised to the extent that it does not exceed recoverable contract costs.

Required Discuss the principles that underlie each of the two methods and describe the circumstances in which their use is appropriate. (5 marks) (b)

is part way through a contract to build a new football stadium at a contracted price of $300 million. Details of the progress of this contract at 1 April 2008 are shown below:

Cumulative sales revenue recognised Cumulative cost of sales to date Profit to date

$ million 150 112 38

The following information has been extracted from the accounting records at 31 March 2009: $ million Total progress payment received for work certified at 29 February 2009 180 Total costs incurred to date (excluding rectification costs below) 195 Rectification costs 17 Linnet has received progress payments of 90% of the work certified at 29 February 2009. Linnet’s surveyor has estimated the sales value of the further work completed during March 2009 was $20 million. At 31 March 2009 the estimated remaining costs to complete the contract were $45 million. The rectification costs are the costs incurred in widening access roads to the stadium. This was the result of an error by Linnet’s architect when he made his initial drawings. Linnet calculates the percentage of completion of its contracts as the proportion of sales value earned to date compared to the contract price. All estimates can be taken as being reliable. Required Prepare extracts of the financial statements for Linnet for the above contract for the year to 31 March 2009. (8 marks) (13 marks)

45

Downloaded by isavic Alsina ([email protected])

lOMoARcPSD|4664814

LSB_F7_Rev Kit:297mm x 210mm

28/10/09

13:37

Page 46

FINANCIAL REPORTING (INTERNATIONAL)

22. Torrent Torrent is a large publicly listed company whose main activity involves construction contracts. Details of three of its contracts for the year ended 31 March 2009 are: Contract Date commenced Estimated duration

Alfa 1 April 2007 3 years $m 20 15

Beta 1 October 2008 18 months $m 6 7.5

Ceta 1 October 2008 2 years $m 12 10

5 12.5 3.5 5.4 12.6

Nil 2 5.5 Nil 1.8

Nil 4 6 Nil nil

Fixed contract price Estimated costs at start of contract Cost to date: At 31 March 2008 At 31 March 2009 Estimated costs at 31 March 2009 to complete Progress payments received at 31March 2008 Progress payments received at 31 March 2009 Notes (i)

The company’s normal policy for determining the percentage completion of contracts is based on the value of work invoiced to date compared to the contract price. Progress payments received represent 90% of the work invoiced. However, no progress payments will be invoiced or received from contract Ceta until it is completed, so the percentage completion of this contract is to be based on the cost to date compared to the estimated total contract costs.

(ii)

The cost to date of $12·5 million at 31 March 2009 for contract Alfa includes $1 million relating to unplanned rectification costs incurred during the current year (ended 31 March 2009) due to subsidence occurring on site.

(iii)

Since negotiating the price of contract Beta,Torrent has discovered the land that it purchased for the project is contaminated by toxic pollutants. The estimated cost at the start of the contract and the estimated costs to complete the contract include the unexpected costs of decontaminating the site before construction could commence.

Required Prepare extracts of the income statement and statement of financial position for Torrent in respect of the above construction contracts for the year ended 31 March 2009. (12 marks)

23. Bowtock Bowtock has leased an item of plant under the following terms: Commencement of the lease was 1 January 2008. Term of lease 5 years. Annual payments in advance $12,000. Cash price and fair value of the asset – $52,000 at 1 January 2008 – equivalent to the present value of the minimum lease payments. Implicit interest rate within the lease (as supplied by the lessor) 8% per annum (to be apportioned on a time basis where relevant). The company’s depreciation policy for this type of plant is 20% per annum on cost (apportioned on a time basis where relevant).

46

Downloaded by isavic Alsina ([email protected])

lOMoARcPSD|4664814

LSB_F7_Rev Kit:297mm x 210mm

28/10/09

13:37

Page 47

QUESTIONS Required Prepare extracts of the income statement and statement of financial position for Bowtock for the year to 30 September 2009 for the above lease. (5 marks)

24. Triangle Triangle, a public listed company, is in the process of preparing its draft financial statements for the year to 31 March 2009. The following matters have been brought to your attention: (i)

On 1 April 2008 the company brought into use a new processing plant that had cost $15 million to construct and had an estimated life of ten years. The plant uses hazardous chemicals which are put in containers and shipped abroad for safe disposal after processing. The chemicals have also contaminated the plant itself which occurred as soon as the plant was used. It is a legal requirement that the plant is decontaminated at the end of its life. The estimated present value of this decontamination, using a discount rate of 8% per annum, is $5 million. The financial statements have been charged with $1·5 million ($15 million/10 years) for plant depreciation and a provision of $500,000 ($5 million/10 years) has been made towards the cost of the decontamination. (8 marks)

(ii)

On 15 May 2009 the company’s auditors discovered a fraud in the material requisitions department. A senior member of staff who took up employment with Triangle in August 2008 had been authorising payments for goods that had never been received. The payments were made to a fictitious company that cannot be traced. The member of staff was immediately dismissed. Calculations show that the total amount of the fraud to the date of its discovery was $240,000 of which $210,000 related to the year to 31 March 2009. (Assume the fraud is material). (5 marks)

(iii)

The company has contacted its insurers in respect of the above fraud. Triangle is insured for theft, but the insurance company maintains that this is a commercial fraud and is not covered by the theft clause in the insurance policy. Triangle has not yet had an opinion from its lawyers. (4 marks)

(iv)

On 1 April 2008 Triangle sold maturing inventory that had a carrying value of $3 million (at cost) to Factorall, a finance house, for $5 million. Its estimated market value at this date was in excess of $5 million. The inventory will not be ready for sale until 31 March 2012 and will remain on Triangle’s premises until this date. The sale contract includes a clause allowing Triangle to repurchase the inventory at any time up to 31 March 2012 at a price of $5 million plus interest at 10% per annum compounded from 1 April 2008. The inventory will incur storage costs until maturity. The cost of storage for the current year of $300,000 has been included in trade receivables (in the name of Factorall). If Triangle chooses not to repurchase the inventory, Factorall will pay the accumulated s storage costs on 31 March 2012. The proceeds of the sale have been debited to the bank and the sale has been included in Triangle’s sales revenue. (8 marks)

Required Explain how the items in (i) to (iv) above should be treated in Triangle’s financial statements for the year to 31 March 2009 in accordance with current international accounting standards. Your answer should quantify the amounts where possible. The mark allocation is shown against each of the four matters above.

(25 marks)

25. Atkins The principle of recording the substance or economic reality of transactions rather than their legal form lies at the heart of the Framework for Preparation and Presentation of Financial Statements and several International Financial Reporting Standards. The development of this principle was partly in reaction to a minority of public interest companies entering into certain complex transactions. These transactions sometimes led to accusations that company directors were involved in 'creative accounting'. (i)

Atkins's operations involve selling cars to the public through a chain of retail car showrooms. It buys most of its new vehicles directly from the manufacturer on the following terms: – Atkins will pay the manufacturer for the cars on the date they are sold to a customer or six months 47

Downloaded by isavic Alsina ([email protected])

lOMoARcPSD|4664814

LSB_F7_Rev Kit:297mm x 210mm

28/10/09

13:37

Page 48

FINANCIAL REPORTING (INTERNATIONAL) after they are delivered to its showrooms whichever is the sooner. – The price paid will be 80% of the retail list price as set by the manufacturer at the date that the goods are delivered. – Atkins will pay the manufacturer 1.5% per month (of the cost price to Atkins) as a 'display charge' until the goods are paid for. – Atkins may return the cars to the manufacturer any time up until the date the cars are due to be paid for. Atkins will incur the freight cost of any such returns. Atkins has never taken advantage of this right of return. – The manufacturer can recall the cars or request them to be transferred to another retailer any time up until the time they are paid for by Atkins. Required Discuss which party bears the risks and rewards in the above arrangement and come to a conclusion on how the transactions should be treated by each party. (6 marks) (ii)

Atkins bought five identical plots of development land for $2 million in 2006. On 1 October 2008 Atkins sold three of the plots of land to an investment company, Landbank, for a total of $2.4 million.This price was based on 75% of the fair market value of $3.2 million as determined by an independent surveyor at the date of sale. The terms of the sale contained two clauses: – Atkins can re-purchase the plots of land for the full fair value of $3.2 million (the value determined at the date of sale) any time until 30 September 2011; and – On 1 October 2011, Landbank has the option to require Atkins to re-purchase the properties for $3.2 million. You may assume that Landbank seeks a return on its investments of 10% per annum.

Required Discuss the substance of the above transactions; and (3 marks) Prepare extracts of the income statement and statement of financial position (ignore cash) of Atkins for the year to 30 September 2009: – If the plots of land are considered as sold to Landbank; and (2 marks) – Reflecting the substance of the above transactions. (4 marks) (15 marks)

26. CB On 1 February 2008, CB, a listed entity, had 3 million ordinary shares in issue. On 1 March 2008, CB made a rights issue of 1 for 4 at $6.50 per share. The issue was completely taken up by shareholders. Extracts from CB’s financial statements for the year ended 31 January 2009 are presented below: CB: Extracts from income statement for the year ended 31 January 2009

Operating profit Finance cost Profit before tax Income tax expense Profit for the period

$000 1,380 (400) 980 (255) 725

48

Downloaded by isavic Alsina ([email protected])

lOMoARcPSD|4664814

LSB_F7_Rev Kit:297mm x 210mm

28/10/09

13:37

Page 49

QUESTIONS CB: Extracts from summarised statement of changes in equity for the year ended 31 January 2009

Balance at 1 February 2008 Issue of share capital Total comprehensive income Equity dividends Balance at 31 January 2009

$000 7,860 4,875 1,625 (300) 14,060

Just before the rights issue, CB’s share price was $7.50, rising to $8.25 immediately afterwards. The share price at close of business on 31 January 2009 was $6.25. At the beginning of February 2009, the average price earnings ratio in CB’s business sector was 28.4, and the P/E of its principal competitor was 42.5. Required: (a) Calculate the earnings per share for CB for the year ended 31 January 2009, and its PE ratio at that date. (6 marks) (b) Discuss the significance of PE ratios to investors and CB’s PE ratio relative to those of its competitor and business sector (4 marks) (10 marks)

27. QRS The directors of QRS, a listed entity, have met to discuss the business’ medium to long term financing requirements. Several possibilities were discussed, including the issue of more shares using a rights issue. In many respects, this would be the most desirable option because the entity is already quite highly geared. However, the directors are aware of several recent cases where rights issues have not been successful because share prices are currently quite low and many investors are averse to any kind of investment in shares. Therefore, the directors have turned their attention to other options. The finance director is on sick leave, and so you, her assistant, have been given the task of responding to the following note from the Chief Executive: ‘Now that we’ve had a chance to discuss possible financing arrangements, the directors are in agreement that we should structure our issue of financial instruments in order to be able to classify them as equity rather than debt. Any increase in the gearing ratio would be unacceptable. Therefore, we have provisionally decided to make two issues of financial instruments as follows: 1. An issue of non-redeemable preference shares to raise $4million. These shares will carry a fixed interest rate of 6% and because they are shares they can be classified as equity. 2. An issue of 6% convertible bonds, issued at par value, to raise $6million. These bonds will carry a fixed date for conversion in four years’ time. Each $100 of debt will be convertible at the holder’s option into 130 $1 shares. In our opinion, these bonds can actually be classified as equity immediately, because they are convertible within five years on terms that are favourable to the holder. Please confirm that these instruments will not increase our gearing ratio should they be issued. Note: You determine that the market rate available for similar non-convertible bonds is currently 8%. Required Explain to the directors the accounting treatment in respect of debt/equity classification required by IAS 32 Financial Instruments: Disclosure and Presentation for each of the proposed issues, advising them on the acceptability of classifying the instruments as equity. Your explanation should be accompanied by calculations where appropriate. (10 marks)

49

Downloaded by isavic Alsina ([email protected])

lOMoARcPSD|4664814

LSB_F7_Rev Kit:297mm x 210mm

28/10/09

13:37

Page 50

FINANCIAL REPORTING (INTERNATIONAL)

28. Harper You are a partner in a small audit and accounting practice. You have just completed the audit and finalised the financial statements of a small family owned company in discussion with its managing director Mrs Harper. After the meeting Mrs Harper has asked for your help. She has obtained the published financial statements of several quoted companies in which she is considering buying some shares as a personal investment. She presents you with the following information: (a)

In the year to 30 September 2009, two companies, Gamma and Toga, reported identical profits before tax of $100 million. Information in the Chairmen’s reports said both companies also expected profits from their core activities (to be interpreted as from continuing operations) to grow by 10% in the following year. Mrs Harper has extracted information from the income statements and made the following summary: Gamma $ million 70 nil 30 100

Operating profit: Continuing activities Acquisitions Discontinued activities

Toga $ million 90 50 (40) 100

A note to the financial statements of Toga said that both the discontinuation and acquisition occurred on 1 April 2009 and were part of an overall plan to focus on its traditional core activities after incurring large losses on a new foreign venture. Required (i) Briefly explain to Mrs Harper why information on discontinued operations is useful; (3 marks) (ii) Calculate the expected operating profit for both companies for the year to 30 September 2010 (assuming the Chairmen’s growth forecasts are correct): – – (b)

in the absence of information of the discontinued operations; and based on the information provided above. (4 marks)

Taylor is another company about which Mrs Harper has obtained the following information from its published financial statements: Earnings per share: Year to 30 September Basic earnings per share

2009 25 cents

2008 20 cents

The earnings per share is based on attributable earnings of $50 million ($30 million in 2008) and 200 million ordinary shares in issue throughout the year (150 million weighted average number of ordinary shares in 2008). Statement of financial position extracts: 8% Convertible loan stock

$ million 200

$ million 200

The loan stock is convertible to ordinary shares in 2011 on the basis of 70 new shares for each $100 of loan stock. Note to the financial statements: There are directors’ share options (in issue since 2006) that allow Taylor’s directors to subscribe for a total of 50 million new ordinary shares at a price of $1.50 each. (Assume the current rate of income tax for Taylor is 25% and the market price of its ordinary shares throughout the year has been $2.50).

50

Downloaded by isavic Alsina ([email protected])

lOMoARcPSD|4664814

LSB_F7_Rev Kit:297mm x 210mm

28/10/09

13:37

Page 51

QUESTIONS Mrs Harper has read that the trend of the earnings per share is a reliable measure of a company’s profit trend. She cannot understand why the increase in profits is 67% ($30 million to $50 million), but the increase in the earnings per share is only 25% (20 cents to 25 cents). She is also confused by the company also quoting a diluted earnings per share figure, which is lower than the basic earnings per share. Required

(c)

(i)

Explain why the trend of earnings per share may be different from the trend of the reported profit, and which is the more useful measure of performance; (3 marks)

(ii)

Calculate the diluted earnings per share for Taylor based on the effect of the convertible loan stock and the directors’ share options for the year to 30 September 2009 (ignore comparatives); and (5 marks)

(iii)

Explain the relevance of the diluted earnings per share measure. (4 marks)

Mrs Harper has noticed that the tax charge for a company called Stepper is $5 million on profits before tax of $35 million. This is an effective rate of tax of 14.3%. Another company Jenni has an income tax charge of $10 million on profit before tax of $30 million. This is an effective rate of tax of 33.3% yet both companies state the rate of income tax applicable to them is 25%. Mrs Harper has also noticed that in the statements of cash flow each company has paid the same amount of tax of $8 million.

Required Advise Mrs Harper of the possible reasons why the income tax charge in the financial statements as a percentage of the profit before tax may not be the same as the applicable income tax rate, and why the tax paid in the statement of cash flow may not be the same as the tax charge in the income statement. (6 marks) (25 marks)

29. FW FW is a listed entity involved in the business of oil exploration, drilling and refining in three neighbouring countries – Aye, Bee and Cee. The business has been consistently profitable, creating high returns for its international shareholders. In recent years, however, there has been an increase in environmental lobbying in FW’s three countries of operation. Two years ago, and environmental group based in Cee started lobbying the government to take action against FW for alleged destruction of valuable wildlife habitats in Cee’s protected wetlands and the displacement of the local population. At the time, the directors of FW took legal advice, on the basis of which they assessed the risk of liability at less than 50%. A contingent liability of $500million was noted in the financial statements to cover possible legal costs, compensation to displaced persons and reinstatement of the habitats, as well as fines. FW is currently preparing its financial statements for the year ended 28 February 2009. Recent advice from the entity’s legal advisers has assessed that the risk of a successful action against FW has increased, and must now be regarded as more likely than not to occur. The board of directors has met to discuss the issue. The directors accept that a provision of $500million is required but would like to be informed of the effects of the adjustment on certain key ratios that the entity headlines in its annual report. All of the directors are concerned about the potentially adverse effect on the share price, as FW is actively engaged in a takeover bid that would involve a substantial share exchange. In addition they feel that the public’s image of the entity is likely to be damaged. The draft financial statements for the year ended 28 February 2009 include the following information relevant for the calculation of key ratios. All figures are before taking into account the $500million provision. The provision will be charged to operating expenses.

51

Downloaded by isavic Alsina ([email protected])

lOMoARcPSD|4664814

LSB_F7_Rev Kit:297mm x 210mm

28/10/09

13:37

Page 52

FINANCIAL REPORTING (INTERNATIONAL)

Net assets (before long term loans) at 1 March 2008 Net assets (before long term loans) at 28 February 2009-08-18 Long term loans at 28 February 2009-08-18 Share capital and reserves at 1 March 2008 Share capital and reserves at 28 February 2009-08-18 Revenue Operating profit Profit before tax Profit for the period

$m 9,016 10,066 4,410 4,954 5,656 20,392 2,080 1,670 1,002

The number of ordinary shares in issue throughout the years ended 29 February 2008 and 2009 was 6,000 million shares of 25c each. FW’s key financial ratios for the 2008 financial year (calculated using the financial statements for the year ended 28 February 2008) were -

Return on equity (using average equity) Return on net assets (using average net assets) Gearing (debt as percentage of equity) Operating profit margin Earnings per share

24.7% 17.7% 82% 10.1% 12.2c

Required Prepare a briefing paper that analyses and interprets the effects of making the environmental provision on FW’s key financial ratios.You should take into account the possible effects on the public perception of FW (12 marks)

30. LMN LMN trades in motor vehicles, which are manufactured and supplied by their manufacturer, IJK. Trading between the two entities is subject to a contractual agreements, the principal terms of which are as follows: •

LMN is entitled to hold on its premises at any one time up to 80 vehicles supplied by IJK. LMN is free to specify the ranges and models of vehicle supplied to it. IJK retains legal title to the vehicles until such time as they are sold to a third party by LMN.



While the vehicles remain on its premises, LMN is required to insure them against loss or damage.



The price at which vehicles are supplied is determined at the time of delivery; it is not subject to any subsequent alteration.



When LMN sells a vehicle to a third party, it is required to inform IJK within three working days. IJK submits an invoice to LMN at the originally agreed price; the invoice is payable by LMN within 30 days.



LMN is entitled to use any of the vehicles supplied to it for demonstration purposes and road testing. However, if more than a specified number of kilometres are driven in a vehicle, LMN is required to pay IJK a rental charge.



LMN has the right to return any vehicle to IJK at any time without incurring a penalty, except for any rental charge incurred in respect of excess kilometres driven.

Required Discuss the economic substance of the contractual arrangement between the two entities in respect of the recognition of inventory and of sales. Refer where appropriate to IAS 18 Revenue. (10 marks) 52

Downloaded by isavic Alsina ([email protected])

lOMoARcPSD|4664814

LSB_F7_Rev Kit:297mm x 210mm

28/10/09

13:37

Page 53

QUESTIONS

31. EFG You are an accountant at EFG, an entity that has recently embarked upon an aggressive programme of acquisitions in order to grow its market share as rapidly as possible. EFG has targeted J, a well-established entity operating in the same sector, but with a significant level of export sales. In order to be able to respond to opportunities quickly, EFG has extablished a basic set of four key financial ratios to assess the performance and position of target businesses. If a business’s ratios fall within the set criteria, more detailed analysis will follow prior to the launch of a formal bid. The four key ratios and the criteria are: Gross profit margin Operating profit margin Return on total capital employed Gearing (long-term liabilities/shareholders’ funds)

should exceed 25% should exceed 13% should exceed 25% should not exceed 25%

J’s most recent financial statements are as follows: J Income statement for the year ended 31 January 2009 $000 1,810 (1,381) 429 (236) 193 (9) 184 (50) 134

Revenue Cost of sales Gross profit Operating expenses Profit from operations Finance cost Profit before tax Income tax expense Net profit for the year

J Statement of changes in equity for the year ended 31 January 2009 Share capital $000 Balance at 1 Feb 2008 Transfer Total comprehensive income Dividends Balance at 31 Jan 2009

350

350

Revaluation reserve $000 210 (10)

200

Retained earnings $000 96 10 134 (21) 219

Total $000 656 134 (21) 769

J Statement of financial position at 31 Jan 2009 $000 Non-current assets Property, plant and equipment Current assets Inventories Trade receivables Cash

$000 707

201 247 18 466 1,173 Continued on next page

53

Downloaded by isavic Alsina ([email protected])

lOMoARcPSD|4664814

LSB_F7_Rev Kit:297mm x 210mm

28/10/09

13:37

Page 54

FINANCIAL REPORTING (INTERNATIONAL) $000 Equity and liabilities Share capital Revaluation reserve Retained earnings

$000

350 200 219 769

Non-current liabilities Long term borrowings Current liabilities Trade payables Income tax

248 142 14 156 1,173

J’s directors who each hold a significant percentage of the ordinary share capital in the entity are interested in EFG’s potential bid, and they have co-operated fully in providing information. On a recent visit to the entity, EFG’s finance director has ascertained that , in many respects, the financial and operating policies of the two businesses are very similar. However there are some differences, summarised as follows: •

J has a policy of revaluation of property, but EFG’s key ratios are set on the assumption of valuation at depreciated historic cost. J owns one property, a warehouse building that was revalued five years ago. At that time, the revaluation surplus was $250,000, and the estimated useful life of the property was 25 years assuming a residual value of nil. J depreciated property on the straight line basis.



J employs a highly skilled team of sales representatives who are paid a substantial profit-related bonus at the end of each year. for the year ended 31 January 2009, the total bonus paid was $96,000 included in operating expenses. EFG’s operating policy does not include the payment of bonuses to staff; the directors prefer to reward staff by a fixed salary. The financial controller estimates that EFG’s operating policy would involve payment of an additional $50,000 of fixed salaries instead of the bonus.



The issued share capital of J includes $50,000 of 4% preferred stock. The directors of EFG believe this should be classified as a long-term liability.



J values inventories using an average cost basis, whereas EFG’s valuation policy is first in first out (FIFO). J’s accountants have estimated that the valuation of their opening and closing inventories on a FIFO basis would be: At 1 February 2008 At 31 January 2009-08-16

$208,000 $218,000

J’s opening inventories at average cost were $197,000. Required (a) Calculate the four key financial ratios for J before making any adjustments in respect of changes required by EFG’s financial and operating policies. (2 marks) (b) Calculate the four key financial ratios for J after making adjustments in respect of changes required by EFG’s financial and operating policies (For this purpose, assume that the alteration in respect of the remuneration of sales representatives would take effect from 1 February 2008). Using EFG’s criteria, advise the directors on whether or not they should pursue the potential acquisition of J. (16 marks) (18 marks)

54

Downloaded by isavic Alsina ([email protected])

lOMoARcPSD|4664814

LSB_F7_Rev Kit:297mm x 210mm

28/10/09

13:37

Page 55

QUESTIONS

32. Breadline You are the assistant financial controller of Judicious. One of your company's credit controllers has asked you to consider the account balance of one of your customers, Breadline. He is concerned at the pattern of payments and increasing size and age of the account balance. As part of company policy he has obtained the most recently filed financial statements of Breadline and these are summarised below. A note to the financial statements of Breadline states that it is a wholly owned subsidiary of Wheatmaster, and its main activities are the production and distribution of bakery products to wholesalers. By coincidence your company's Chief Executive has been made aware that Breadline may be available for sale. She has asked for your opinion on whether Breadline would make a suitable addition to the group's portfolio. BREADLINE INCOME STATEMENT YEAR TO:

Sales revenue Cost of sales Gross profit Operating expenses Finance costs: loan note overdraft Profit before tax Taxation Profit for the year

31 December 2009 $'000 $'000 8,500 (5,950) 2,550 (560) 10 10 (20) 1,970 (470) 1,500

31 December 2008 $'000 $'000 6,500 (4,810) 1,690 (660) nil 5 (5) 1,025 (175) 850

31 December 2009 $'000 $'000

31 December 2008 $'000 $'000

STATEMENTS OF FINANCIAL POSITION AS AT:

Non-current assets Freehold premises at valuation Leasehold premises Plant Current assets Inventory Accounts receivable Bank

nil 2,500 1,620 4,120

1,250 nil 750 2,000

370 960 nil

Total assets Equity and liabilities Equity Ordinary shares of $1 each Reserves Share premium Revaluation surplus (re freehold premises) Retained earnings

240 600 250 1,330 5,450

1,090 3,090

500

100

200

nil

nil 3,000

700 1,700 3,200 3,700

2,400 2,500 Continued on next page

55

Downloaded by isavic Alsina ([email protected])

lOMoARcPSD|4664814

LSB_F7_Rev Kit:297mm x 210mm

28/10/09

13:37

Page 56

FINANCIAL REPORTING (INTERNATIONAL) 31 December 2009 $'000 $'000 Non-current liabilities % loan note Current liabilities Accounts payable Overdraft

31 December 2008 $'000 $'000

500 1,030 220

590 nil 1,250 5,450

Total equity and liabilities

nil

590 3,090

From your company's own records you have ascertained that sales to Breadline for the year to 2009 and 2008 were $1,200,000 and $800,000 respectively and the year-end account balances were $340,000 and $100,000 respectively. Normal credit terms, which should apply to Breadline, are that payment is due 30 days after the end of the month of sale. You are also aware that the company has not changed its address and is trading from the same premises. A note to Breadline's financial statements says that the profit on the disposal of its freehold premises has been included in cost of sales as this is where the depreciation on the freehold was charged. Dividends of $900,000 were paid in 2009. No dividends were paid in 2008. Note. A commercial rate of interest on the loan note of Breadline would be 8% per annum. Required (a) Describe the matters that may be relevant when entity financial statements are used to assess the performance of a company that is a wholly owned subsidiary. (5 marks) Note. Your answer should give attention to related party issues. (b) From the information above and with the aid of suitable ratios, prepare a report for your Chief Executive on the overall financial position of Breadline. Your answer should include reference to matters in the financial statements of Breadline that may give you cause for concern or require further investigation. (20 marks) (25 marks)

33. Comparator Comparator assembles computer equipment from bought in components and distributes them to various wholesalers and retailers. It has recently subscribed to an interfirm comparison service. Members submit accounting ratios as specified by the operator of the service, and in return, members receive the average figures for each of the specified ratios taken from all of the companies in the same sector that subscribe to the service. The specified ratios and the average figures for Comparator’s sector are shown below. Ratios of companies reporting a full year’s results for periods ending between 1 July 2009 and 30 September 2009 Return on capital employed Revenue/capital employed Gross profit margin Net profit (before tax) margin Current ratio Quick ratio Inventory holding period Accounts receivable collection period Accounts payable payment period Debt to equity Dividend yield Dividend cover

22·1% 1·8 times 30% 12·5% 1·6:1 0·9:1 46 days 45 days 55 days 40% 6% 3 times

56

Downloaded by isavic Alsina ([email protected])

lOMoARcPSD|4664814

LSB_F7_Rev Kit:297mm x 210mm

28/10/09

13:37

Page 57

QUESTIONS Comparator’s financial statements for the year to 30 September 2009 are set out below: Income statement Sales revenue Cost of sales Gross profit Other operating expenses Interest payable Inventory write off due to obsolescence Profit before taxation Income tax expense Profit for the year Extracts of changes in equity Retained earnings – 1 October 2008 Net profit for the year Dividends paid (interim $60,000; final $30,000) Retained earnings – 30 September 2009 Statement of financial position Non-current assets (note (i)) Current assets Inventory Accounts receivable Bank

$'000 2,425 (1,870) 555 (215) (34) (120) 186 (90) 96

179 96 (90) 185 $'000

275 320 Nil

Equity and liabilities Equity Ordinary shares (25 cents each) Retained earnings Non-current liabilities 8% loan notes Current liabilities Bank overdraft Trade accounts payable

$'000 540

595 1,135

150 185 335 300 65 350 85

500 1,135

Notes (i) The details of the non-current assets are:

At 30 September 2009

Cost $'000 3,600

Accumulated depreciation $'000 3,060

Net book value $'000 540

(ii) The market price of Comparator’s shares throughout the year averaged $6·00 each. Required (a) Explain the problems that are inherent when ratios are used to assess a company’s financial performance. Your answer should consider any additional problems that may be encountered when using interfirm comparison services such as that used by Comparator. (7 marks)

57

Downloaded by isavic Alsina ([email protected])

lOMoARcPSD|4664814

LSB_F7_Rev Kit:297mm x 210mm

28/10/09

13:37

Page 58

FINANCIAL REPORTING (INTERNATIONAL) (b) Calculate the ratios for Comparator equivalent to those provided by the interfirm comparison service. (6 marks) (c) Write a report analysing the financial performance of Comparator based on a comparison with the sector averages. (12 marks) (25 marks)

34. Bigwood Bigwood, a public company, is a high street retailer that sells clothing and food. The managing director is very disappointed with the current year’s results. The company expanded its operations and commissioned a famous designer to restyle its clothing products. This has led to increased sales in both retail lines, yet overall profits are down. Details of the financial statements for the two years to 30 September 2009 are shown below. Year to 30 September 2009 $’000 $’000 16,000 7,000 23,000 14,500 4,750 (19,250) 3,750 (2,750) 1,000 (300) 700 (250) 450

INCOME STATEMENT: Revenue Cost of sales

- clothing - food - clothing - food

Gross profit Other operating expenses Operating profit Interest expense Profit before tax Income tax expense Profit for the year SUMMARISED CHANGES IN EQUITY:

Retained profit b/f Profit for the year Dividends paid Retained profit c/f

Year to 30 September 2009 $’000 1,900 450 (600) 1,750

Year to 30 September 2008 $’000 $’000 15,600 4,000 19,600 12,700 3,000 (15,700) 3,900 (1,900) 2,000 (80) 1,920 (520) 1,400 Year to 30 September 2008 $’000 1,100 1,400 (600) 1,900

STATEMENTS OF FINANCIAL POSITION AS AT:

Property, plant and equipment at cost Accumulated depreciation Current assets Inventory

– clothing – food Trade receivables Bank Total assets

30 September 2009 $’000 $’000 17,000 (5,000) 12,000

30 September 2008 $’000 $’000 9,500 (3,000) 6,500

2,700 200 100 Nil

1,360 140 50 450

3,000 15,000

58

Downloaded by isavic Alsina ([email protected])

2,000 8,500

lOMoARcPSD|4664814

LSB_F7_Rev Kit:297mm x 210mm

28/10/09

13:37

Page 59

QUESTIONS 30 September 2009 $’000 $’000

30 September 2008 $’000 $’000

5,000 1,000 1,750 7,750

3,000 Nil 1,900 4,900

3,000

1,000

Equity and liabilities Issued ordinary capital ($1 shares) Share premium Retained earnings Non-current liabilities Long-term loans Current liabilities Bank overdraft Trade payables Current tax payable

930 3,100 220

4,250 15,000

nil 2,150 450

2,600 8,500

Note. The directors have signalled their intention to maintain annual dividends at $600,000 for the foreseeable future. The following information is relevant: (i)

The increase in property, plant and equipment was due to the acquisition of five new stores and the refurbishment of some existing stores during the year. The carrying value of fixtures scrapped at the refurbished stores was $1.2 million; they had originally cost $3 million. Bigwood received no scrap proceeds from the fixtures, but did incur costs of $50,000 to remove and dispose of them. The losses on the refurbishment have been charged to operating expenses. Depreciation is charged to cost of sales apportioned in relation to floor area (see below).

(ii)

The floor sales areas (in square metres) were: 30 September 2009 48,000 6,000 54,000

Clothing Food

30 September 2008 35,000 5,000 40,000

(iii)

The share price of Bigwood averaged $6.00 during the year to 30 September 2008, but was only $3.00 at 30 September 2009.

(iv)

The following ratios have been calculated: Return on capital employed Net assets turnover Gross profit margin – clothing – food Net profit (after tax) margin Current ratio Inventory holding period – clothing – food Accounts payment period Gearing Interest cover

2009 9·3% 2·1 times

2008 33·9% 3·3 times

9·4% 32·1% 2·0% 0·71:1

18·6% 25% 7·1% 0·77 :1

68 days 15 days 59 days 28% 3·3 times

39 days 17 days 50 days 17% 25 times

59

Downloaded by isavic Alsina ([email protected])

lOMoARcPSD|4664814

LSB_F7_Rev Kit:297mm x 210mm

28/10/09

13:37

Page 60

FINANCIAL REPORTING (INTERNATIONAL) Required (a) Prepare, using the indirect method, a statement of cash flows for Bigwood for the year to 30 September 2009 (12 marks) (b) Write a report analysing the financial performance and financial position of Bigwood for the two years ended 30 September 2009. (13 marks) Your report should utilise the above ratios and the information in your statement of cash flows. It should refer to the relative performance of the clothing and food sales and be supported by any further ratios you consider appropriate. (25 marks)

35. Tabba The following draft financial statements relate to Tabba, a private company. Statements of financial position as at: Property, plant and equipment (note (ii)) Current assets Inventories Trade receivables Insurance claim (note (iii)) Cash and bank Total assets Equity and liabilities Share capital ($1 each) Reserves: Revaluation (note (ii)) Retained earnings

30 September 2009 $'000 $'000 10,600

30 September 2008 $'000 $'000 15,800

2,550 3,100 1,500 850

1,850 2,600 1,200 nil 8,000 18,600

5,650 21,450

6,000

6,000

nil 2,550

1,600 850 2,550 8,550

Non–current liabilities Finance lease obligations (note (ii)) 6% loan notes 10% loan notes Deferred tax Government grants (note (ii))

2,000 800 nil 200 1,400

2,450 8,450 1,700 nil 4,000 500 900

4,400 Current liabilities Bank overdraft Trade payables Government grants (note (ii)) Finance lease obligations (note (ii)) Current tax payable Total equity and liabilities

nil 4,050 600 900 100

7,100 550 2,950 400 800 1,200

5,650 18,600

60

Downloaded by isavic Alsina ([email protected])

5,900 21,450

lOMoARcPSD|4664814

LSB_F7_Rev Kit:297mm x 210mm

28/10/09

13:37

Page 61

QUESTIONS The following additional information is relevant: (i)

Income statement extract for the year ended 30 September 2009: $’000 270 (260) 40 50 50 100

Operating profit before interest and tax Interest expense Interest receivable Profit before tax Net income tax credit Profit for the year

Note. The interest expense includes finance lease interest. (ii)

The details of the property, plant and equipment are:

At 30 September 2008 At 30 September 2009

Cost $’000 20,200 16,000

Acc'd depreciation $’000 4,400 5,400

Carrying value $’000 15,800 10,600

During the year Tabba sold its factory for its fair value $12 million and agreed to rent it back, under an operating lease, for a period of five years at $1 million per annum. At the date of sale it had a carrying value of $7.4 million based on a previous revaluation of $8.6 million less depreciation of $1.2 million since the revaluation. The profit on the sale of the factory has been included in operating profit. The surplus on the revaluation reserve related entirely to the factory. No other disposals of non–current assets were made during the year. Plant acquired under finance leases during the year was $1.5 million. Other purchases of plant during the year qualified for government grants of $950,000. Amortisation of government grants has been credited to cost of sales. (iii)

The insurance claim related to flood damage to the company’s inventories which occurred in September 2008. The original estimate has been revised during the year after negotiations with the insurance company. The claim is expected to be settled in the near future.

Required (a)

Prepare a statement of cash flows using the indirect method for Tabba in accordance with IAS 7 Statement of Cash Flows for the year ended 30 September 2009. (17 marks)

(b)

Using the information in the question and your statement of cash flows, comment on the change in the financial position of Tabba during the year ended 30 September 2009. (8 marks) Note. You are not required to calculate any ratios.

(25 marks)

61

Downloaded by isavic Alsina ([email protected])

lOMoARcPSD|4664814

LSB_F7_Rev Kit:297mm x 210mm

28/10/09

13:37

Page 62

FINANCIAL REPORTING (INTERNATIONAL)

36. Nedberg The financial statements of Nedberg for the year to 30 September 2009, together with the comparative of financial position for the year to 30 September 2008 are shown below: INCOME STATEMENT – YEAR TO 30 SEPTEMBER 2009 $m 3,820 (2,620) 1,200 (300) (30) 870 (270) 600

Sales revenue Cost of sales (note (1)) Gross Profit for the period Operating expenses (note (1)) Interest – Loan note Profit before tax Taxation Profit for the year

STATEMENTS OF FINANCIAL POSITION AS AT 30 SEPTEMBER Non-current assets Property, plant and equipment Intangible assets (note (2)) Current assets Inventory Trade receivables Cash Total assets Equity and liabilities Ordinary Shares of $1 each Reserves: Share premium Revaluation surplus Retained earnings Total equity Non-current liabilities (note(3)) Current liabilities (note(4)) Total equity and liabilities

$m

1,420 990 70

2009 $m 1,890 650 2,540

2,480 5,020

2008 $m

940 680 nil

$m 1,830 300 2,130

1,620 3,750

750

500

350 140 1,890 3,130 870 1,020 5,020

100 nil 1,600 2,200 540 1,010 3,750

Notes to the financial statements (1)

Cost of sales includes depreciation of property, plant and equipment of $320 million and a loss on the sale of plant of $50 million. It also includes a credit for the amortisation of government grants. Operating expenses include a charge of $20 million for the impairment of goodwill.

(2)

Intangible non-current assets

Deferred development expenditure Goodwill

2009 $m 470 180 650

2008 $m 100 200 300

62

Downloaded by isavic Alsina ([email protected])

lOMoARcPSD|4664814

LSB_F7_Rev Kit:297mm x 210mm

28/10/09

13:37

Page 63

QUESTIONS (3)

Non-current liabilities 10% loan note Government grants Deferred tax

(4)

Current liabilities Trade payables Bank overdraft Accrued loan interest Taxation

2009 $m 300 260 310 870

2008 $m 100 300 140 540

2009 $m 875 nil 15 130 1,020

2008 $m 730 115 5 160 1,010

The following additional information is relevant: (i)

Intangible non-current assets The company successfully completed the development of a new product during the current year, capitalising a further $500 million before amortisation charges for the period.

(ii)

Property, plant and equipment/revaluation reserve • The company revalued its buildings by $200 million on 1 October 2008. The surplus was credited to revaluation surplus. • New plant was acquired during the year at a cost of $250 million and a government grant of $50 million was received for this plant. • On 1 October 2008 a bonus issue of 1 new share for every 10 held was made from the revaluation surplus. • $10 million has been transferred from the revaluation surplus to realised profits as a year-end adjustment in respect of the additional depreciation created by the revaluation. • The remaining movement on property, plant and equipment was due to the disposal of obsolete plant.

(iii)

Share issues In addition to the bonus issue referred to above Nedberg made a further issue of ordinary shares for cash.

(iv)

Dividends Dividends paid during the year amounted to $320,000.

Required (a)

A statement of cash flows for Nedberg for the year to 30 September 2009 prepared in accordance with IAS 7 Statement of cash flows. (20 marks)

(b)

Comment briefly on the financial position of Nedberg as portrayed by the information in your statement of cash flows. (5 marks) (25 marks)

63

Downloaded by isavic Alsina ([email protected])

lOMoARcPSD|4664814

LSB_F7_Rev Kit:297mm x 210mm

28/10/09

13:37

Page 64

FINANCIAL REPORTING (INTERNATIONAL)

37. Minster Minster is a publicly listed company. Details of its financial statements for the year ended 30 September 2009, together with a comparative statement of financial position, are: STATEMENT OF FINANCIAL POSITION AT 30 September 2009 $'000 $'000 Non-current assets (note (i)) Property, plant and equipment Software Investments at fair value through profit and loss Current assets Inventories Trade receivables Amounts due from construction contracts Bank Total assets Equity and liabilities Equity shares of 25 cents each Reserves Share premium (note (ii)) Revaluation reserve Retained earnings

30 September 2008 $'000 $'000

1,280 135

940 nil

150 1,565

125 1,065

480 270

510 380

80 nil

55 35 830 2,395

980 2,045

500

300

150 60 950

85 25 965 1,160 1,660

Non-current liabilities 9% loan note Environmental provision Deferred tax

120 162 18

1,075 1,375 nil nil 25

300 Current liabilities Trade payables Bank overdraft Current tax payable Total equity and liabilities

350 25 60

25 555 40 50

435 2,395

64

Downloaded by isavic Alsina ([email protected])

645 2,045

lOMoARcPSD|4664814

LSB_F7_Rev Kit:297mm x 210mm

28/10/09

13:37

Page 65

QUESTIONS STATEMENT OF COMPREHENSIVE INCOME FOR THE YEAR ENDED 30 SEPTEMBER 2009

Revenue Cost of sales Gross profit Operating expenses Finance costs (note (i)) Investment income and gain on investments Profit before tax Income tax expense Profit for the year Other comprehensive income – revaluation surplus on land Total comprehensive income

$'000 1,397 (1,110) 287 (125) 162 (40) 20 142 (57) 85 35 120

The following supporting information is available: (i)

Included in property, plant and equipment is a coal mine and related plant that Minster purchased on 1 October 2008. Legislation requires that in ten years' time (the estimated life of the mine) Minster will have to landscape the area affected by the mining. The future cost of this has been estimated and discounted at a rate of 8% to a present value of $150,000. This cost has been included in the carrying amount of the mine and, together with the unwinding of the discount, has also been treated as a provision. The unwinding of the discount is included within finance costs in the income statement. Depreciation of property, plant and equipment for the year was $255,000. There were no disposals of property, plant and equipment during the year. The software was purchased on 1 April 2009 for $180,000. The market value of the investments had increased during the year by $15,000. There have been no sales of these investments during the year.

(ii)

On 1 April 2009 there was a bonus (scrip) issue of equity shares of one for every four held utilising the share premium reserve. A further cash share issue was made on 1 June 2009. No shares were redeemed during the year.

(iii)

A dividend of 5 cents per share was paid on 1 July 2009.

Required (a)

Prepare a statement of cash flows for Minster for the year to 30 September 2009 in accordance with IAS 7 Statement of cash flows. (15 marks)

(b)

Comment on the financial performance and position of Minster as revealed by the above financial statements and your statement of cash flows. (10 marks) (25 marks)

65

Downloaded by isavic Alsina ([email protected])

lOMoARcPSD|4664814

LSB_F7_Rev Kit:297mm x 210mm

28/10/09

13:37

Page 66

FINANCIAL REPORTING (INTERNATIONAL)

38. Rytetrend Rytetrend is a retailer of electrical goods. Extracts from the company’s financial statements are set out below: INCOME STATEMENT FOR THE YEAR ENDED 31 MARCH 2009 $'000 Sales revenue Cost of sales Gross profit Other operating expense Interest payable – loan notes – overdraft Profit before taxation Income tax expense Profit for the year

2008

$'000 31,800 (22,500) 9,300 (5,440)

(260) (200)

(460) 3,400 (1,000) 2,400

$'000

$'000 23,500 (16,000) 7,500 (4,600)

(500) Nil

(500) 2,400 (800) 1,600

STATEMENTS OF FINANCIAL POSITION AS AT 31 MARCH 2009 Assets Non-current assets (note (i)) Current assts Inventory Receivables Bank Total assets Equity and liabilities Equity Ordinary capital ($1 shares) Share premium Retained earnings Non-current liabilities 10% loan notes 6% loan notes Current liabilities Bank overdraft Trade payables Taxation Warranty provision (note (ii)) Total equity and liabilities

$'000

2,650 1,100 Nil

1,050 2,850 720 500

2008 $'000 24,500

3,750 28,250

$'000

3,270 1,950 400

10,000 Nil 6,160 16,160

Nil 2,000

4,000 Nil

5,120 28,250

Nil 1,980 630 150

(i) The details of the non-current assets are:

At 31 March 2008 At 31 March 2009

5,620 22,920

11,500 1,500 8,130 21,130

Notes

Cost $'000 27,500 37,250

$'000 17,300

Accumulated Depreciation Net book value $'000 $'000 10,200 17,300 12,750 24,500

66

Downloaded by isavic Alsina ([email protected])

2,760 22,920

lOMoARcPSD|4664814

LSB_F7_Rev Kit:297mm x 210mm

28/10/09

13:37

Page 67

QUESTIONS During the year there was a major refurbishment of display equipment. Old equipment that had cost $6 million in September 2004 was replaced with new equipment at a gross cost of $8 million. The equipment manufacturer had allowed Rytetrend a trade in allowance of $500,000 on the old display equipment. In addition to this Rytetrend used its own staff to install the new equipment. The value of staff time spent on the installation has been costed at $300,000, but this has not been included in the cost of the asset. All staff costs have been included in operating expenses. All display equipment held at the end of the financial year is depreciated at 20% on its cost. No equipment is more than five years old. (ii) Operating expenses contain a charge of $580,000 for the cost of warranties on the goods sold by Rytetrend. The company makes a warranty provision when it sells its products and cash payments for warranty claims are deducted from the provision as they are settled. (iii) Dividends paid in the year ended 31 March 2009 were $430,000. Required (a) Prepare a cash flow statement for Rytetrend for the year ended 31 March 2009. (12 marks) (b) Write a report briefly analysing the operating performance and financial position of Rytetrend for the years ended 31 March 2008 and 2009. (13 marks) Your report should be supported by appropriate ratios.

(25 marks)

39. Update Most companies prepare their financial statements under the historical cost convention. In times of rising prices it has been said that without modification such financial statements can be misleading. Required (a) Explain the problems that can be encountered when users rely on financial statements prepared under the historical cost convention for their information needs. (6 marks) Note: Your answer should consider problems with the income statement and the statement of financial position. (b) Update has been considering the effect of alternative methods of preparing their financial statements. As an example they picked an item of plant that they acquired from Suppliers on 1 April 2006 at a cost of $250,000. The following details have been obtained: • •

the company policy is to depreciate plant at 20% per annum on the reducing balance basis. the movement in the retail price index has been: 1 April 2006 1 April 2007 1 April 2008 31 March 2009



180 202 206 216

Suppliers’ price catalogue at 31 March 2009 shows an item of similar plant at a cost of $320,000. On reading the specification it appears that the new model can produce 480 units per hour whereas the model owned by Update can only produce 420 units per hour.

Required Calculate for Update the depreciation charge for the plant for the year to 31 March 2009 (based on year end values) and its carrying value in the statement of financial position on that date using: • • •

the historical cost basis; a current purchasing power basis; and a current cost basis. (6 marks) (12 marks) 67

Downloaded by isavic Alsina ([email protected])

lOMoARcPSD|4664814

LSB_F7_Rev Kit:297mm x 210mm

28/10/09

13:37

Page 68

FINANCIAL REPORTING (INTERNATIONAL)

40. Appraisal Reactive is a publicly listed company that assembles domestic electrical goods which it then sells to both wholesale and retail customers. Reactive’s management were disappointed in the company’s results for the year ended 31 March 2008. In an attempt to improve performance the following measures were taken early in the year ended 31 March 2009: • A national advertising campaign was undertaken • Rebates to all wholesale customers purchasing goods above set quantity levels were introduced • The assembly of certain lines ceased and was replaced by bought in completed products. This allowed Reactive to dispose of surplus plant. Reactive’s summarised financial statements for the year ended 31 March 2009 are set out below: Income statement $million 4,000 (3,450) 550 (370) 180 40 (20) 200 (50) 150

Revenue (25% cash sales) Cost of sales Gross profit Operating expenses Profit on disposal of plant (note 1) Finance charges Profit before tax Income tax expense Profit for the period Statement of financial position Non-current assets Property, plant and equipment (note 1) Current assets Inventory Trade receivables Bank

$million

550 250 360 Nil 610 1,160

Total assets Equity and liabilities Equity shares of 25c each Retained earnings

100 380 480

Non-current liabilities 8% loan notes Current liabilities Bank overdraft Trade payables Current tax payable Total equity and liabilities

$million

200 10 430 40 480 1,160

68

Downloaded by isavic Alsina ([email protected])

lOMoARcPSD|4664814

LSB_F7_Rev Kit:297mm x 210mm

28/10/09

13:37

Page 69

QUESTIONS Below are ratios for the year ended 31 March 2008: Return on capital employed (profit before interest and tax over total assets less current liabilities) Net asset (equal to capital employed) turnover Gross profit margin Net profit (before tax) margin Current ratio Closing inventory holding period Trade receivables’ collection period Trade payables’ payment period Dividend yield Dividend cover

28.1% 4 times 17% 6.3% 1.6:1 46 days 45 days 55 days 3.75% 2 times

Notes: •

Reactive received $120million from the sale of plant that had a carrying amount of $80million at the date of its sale;



The market price of Reactive’s shares throughout the year averaged $3.75 each;



There were no issues of redemption of shares or loans during the year;



Dividends paid during the year ended 31 March 2009 amounted to $90million, maintaining the same dividend paid in the year ended 31 March 2008.

Required (a) Calculate ratios for the year ended 31 March 2009 (showing your workings) for Reactive, equivalent to those provided above (10 marks) (b) Analyse the financial performance and position of Reactive for the year ended 31 March 2009 compared to the previous year (10 marks) (c) Explain in what ways your approach to performance appraisal would differ if you were asked to assess the performance of a not-for-profit organisation (5 marks) (25 marks)

69

Downloaded by isavic Alsina ([email protected])

lOMoARcPSD|4664814

LSB_F7_Rev Kit:297mm x 210mm

28/10/09

13:37

Page 70

FINANCIAL REPORTING (INTERNATIONAL)

70

Downloaded by isavic Alsina ([email protected])

lOMoARcPSD|4664814

LSB_F7_Rev Kit:297mm x 210mm

28/10/09

13:37

Page 71

F7 Answers

Downloaded by isavic Alsina ([email protected])

lOMoARcPSD|4664814

LSB_F7_Rev Kit:297mm x 210mm

28/10/09

13:37

Page 72

FINANCIAL REPORTING (INTERNATIONAL)

72

Downloaded by isavic Alsina ([email protected])

lOMoARcPSD|4664814

LSB_F7_Rev Kit:297mm x 210mm

28/10/09

13:37

Page 73

ANSWERS

1.

Hepburn

(a)

Consolidated income statement for Hepburn for the year ended 31 March 2009

Revenue Cost of sales Gross profit Operating expenses Debenture interest Profit before tax Income tax expense Profit for the year Attributable to: Owners of the parent (_) Non-controlling interest (W5)

$000 1,600 (890) 22 710 (184) (6) 22 520 (120) 22 400 22 388 20

Marks

1 See W2

See W2 1/2

1/2

1/2

1

Consolidated Statement of Financial Position for Hepburn as at 31 March 2009 $000 Non-current assets Tangible: Property, plant and equipment (620 + 660 + 125) Intangible: Goodwill (W4) Investments (20 + 10) Current assets Inventory (240 + 280 – 10 (W7)) Accounts receivable (170 + 210 – 56 Interco) Bank (20 + 40 + 20 cash in transit) Total assets Equity and Liabilities Equity shares (400 + 300 (W8)) Share premium (W8) Retained earnings (W6) Non-controlling interest (W5) Non-current liabilities 8% debentures Current liabilities Trade accounts payable (170 + 155 – 36 Interco) Tax payable (50 + 45) Total equity and liabilities

1,405 180 30 22 1,615

1 1/2 1/2

510 324 80 22 914 22 2,529 22

1

700 600 500 22 1,800 195

1

150 289 95 384 22 2,529 22

1 1

1 See W6

1

1/2

1 1/2

73

Downloaded by isavic Alsina ([email protected])

lOMoARcPSD|4664814

LSB_F7_Rev Kit:297mm x 210mm

28/10/09

13:37

Page 74

FINANCIAL REPORTING (INTERNATIONAL) Workings W1

Group structure

H

80%

S 1 Oct 2008 (owned 6m) W2

Consolidation schedule H $000 1,200 (650) (10) (120) (20) – (100)

Revenues Cost of sales URP (W7) Operating expenses Impt of GW Debenture interest Income taxes W3

Adj $000 (100) 100

Group $000 1,600 (890)

1 1/2

(44)

(184)

(6) (20) 22 100

(6) (120)

1/2

See W4

Net assets of Salter

Share capital Retained earnings Fair value adj’t W4

S (6m) $000 500 (330)

@ Rep’g date $000 150 700

@ Acq’n date $000 150 600

125 22 975

125 22 875

Movement $000 100

(700 – 6/12 x 200)

Goodwill

Consideration (W8) Non-controlling interest at acquisition (20% x 875 (W3)) Fair value of net assets of Salter at acquisition (W3) Goodwill Impairment charge ( ) Carrying value of goodwill

$000 900 175 22 1,075 (875) 22 200 (20) 22 180

Tutorial note In this question, the NCI is measured as a proportion of net assets.Therefore there is no NCI goodwill.The impairment loss of $20m is therefore attributable to the parent company only.As such it is charged to the parent company’s profits in (W2) and (W6)

74

Downloaded by isavic Alsina ([email protected])

See W8 1/2

1/2

lOMoARcPSD|4664814

LSB_F7_Rev Kit:297mm x 210mm

28/10/09

13:37

Page 75

ANSWERS W5

Non-controlling interest

NCI in profit: 20% x 100 (W2) NCI in net assets: 20% x 975 (W3) W6

Group reserves

Hepburn Group share of Salter’s post acquisition profits (80% x 100 (W3)) Impairment of goodwill URP in inventory (W7) W7

$000 20 195

$000 450 80 (20) (10) 22 500

1/2 1/2

1/2

URP in inventory $100,000 x 1/2 x 25/125 = $10,000

W8

Share for share exchange Shares acquired:

150,000 x 80% = 120,000

Therefore shares issued:

120,000 x 5/2 = 300,000

1

Consideration:

300,000 x $3 = $900,000

1

Nominal value of new shares to be recorded in CSFP: $300,000 Premium to be recorded in CSFP: $600,000 (b)

Hepburn’s directors have treated Woodbridge as a long-term non-current asset investment rather than an associate or subsidiary. An associate is an entity over which another entity has significant influence.This is presumed to exist when between 20% and 50% of the voting power is held by the investing company. The presumption can, however be rebutted where is can be clearly shown that the investor does not have significant influence.

1

A subsidiary is an entity over which another entity has control, presumed to exist where more than 50% of the voting power is held by the investing company.

1

In their assessment of the investment in Woodbridge, directors have concentrated on the fact that Hepburn owns 25% of the total (type A and B) shares in Woodbridge however exerts no influence over the operating policies or management of the company. On this basis they appear to have rebutted the presumption that Woodbridge is an associate.

1

Hepburn does however own 60% of the equity voting A shares in Woodbridge and so controls 60% of the voting power, meaning that it meets the definition of a subsidiary.

1

On this basis it appears that the results of Woodbridge should be consolidated into the group accounts on a line by line basis. It is not an option to exclude Woodbridge from consolidation on the basis that it is loss-making and would adversely affect group results.

1

75

Downloaded by isavic Alsina ([email protected])

lOMoARcPSD|4664814

LSB_F7_Rev Kit:297mm x 210mm

28/10/09

13:37

Page 76

FINANCIAL REPORTING (INTERNATIONAL)

2.

Highveldt Tutorial note Although this is not a full consolidation question, it should be approached in the same way, using standard workings.

(a)

(i) Goodwill

$m

Consideration Cash ($3.50 x 75% x 80m) Deferred cash ($108m/1.08) Non-controlling interest at acquisition (25% x 296 (W2)) Fair value of net assets of Salter at acquisition (W2) Goodwill Impairment charge Carrying value of goodwill

210 100 22 310 74 22 384

Marks

1 1

1/2

for 25% + See W2

(296) 22 88 (22) 22 66

1

Tutorial note In this question, the NCI is measured as a proportion of net assets.Therefore there is no NCI goodwill.The impairment loss of $22m is therefore attributable to the parent company only.As such it is charged against group reserves (a) (iii). (a)

(ii) Non-controlling interest NCI in net assets: 25% x 326 (W2)

(a)

(iii) Group share premium Highveldt 75% post acquisition movement in share premium of Samson

Group revaluation reserve Highveldt 75% post acquisition movement in revaluation reserve of Samson (75% x 4)

Group retained earnings Highveldt 75% post acquisition movement in share premium of Samson (75% x 48 (W2) Write off of goodwill Interest ($60m x 10%) Unwinding of discount on deferred consideration

76

Downloaded by isavic Alsina ([email protected])

$m 81.5 $m

1/2

for 25% + see W2

80 Nil 22 80 $m 45 3 22 48

350 36 (22) 6 (8) 22 362

1

1/2 1/2

for 75% 1 for 4

1/2 1/2

for 75% + See W2 1 1 1

lOMoARcPSD|4664814

LSB_F7_Rev Kit:297mm x 210mm

28/10/09

13:37

Page 77

ANSWERS Workings W1

Group structure

H

75%

S 1 April 2008 (held 1yr) W2

Net assets of Salter @ Rep’g date $m Share capital 80 Share premium 40 Revaluation reserve – FV adjustment (i) 24 Retained earnings 210 Brand (iii) 36 Development costs (iv) (40) URP (v) 6m/3 (2) 22 348

@ Acq’n date $m 80 40 – 20 134 40 (18)

Movement $000 – – 4

1/2 1/2

1+1 1/2

+ 1/2

1+1

48

22 296

1+1

1

Tutorial notes:

(b)



The increase in the fair value of the land and buildings post acquisition is treated as part of the revaluation reserve



The brand is internally generated for Samson and so cannot appear on its own statement of financial position. For the group, however, it is a purchased assed and is therefore included as an asset at its fair value at acquisition. Subsequent amortisation is charged to group reserves.



The development costs should not have been capitalised as they do not meet the IAS 38 criteria. The capitalised amount is therefore reversed.

In the separate entity financial statements of a parent company, a subsidiary is held as a non-current asset investment at cost.Any dividend income is recorded in the parent’s income statement.

1 mark per valid point

The cost of the investment in the subsidiary is likely to differ significantly from the value of the net assets owned. In the first years of ownership, the cost will normally exceed the net assets owned (the excess being goodwill). After a number of years when the subsidiary has made profits, the cost of the investment will be less than the book value of the net assets owned. Similarly, the dividend paid by a subsidiary to its parent company and so reflected in their income statement is unlikely to be equal to the parent company’s share of that subsidiary’s profits. For these reasons, single entity financial statements of a parent company provide only a limited amount of useful information about its subsidiaries. Full consolidated accounts, whereby all of the subsidiary’s assets and liabilities and income and expenses are added on a line by line basis (and then the share owned by the non-controlling interest allocated to them) are more useful because: •

They reflect the fact that the whole of the subsidiary, its assets, liabilities and profits are controlled by the parent company. 77

Downloaded by isavic Alsina ([email protected])

lOMoARcPSD|4664814

LSB_F7_Rev Kit:297mm x 210mm

28/10/09

13:37

Page 78

FINANCIAL REPORTING (INTERNATIONAL) •

They reflect the historic performance of the subsidiary by including accumulated profits.



They quantify goodwill, being the premium paid on acquisition of the subsidiary.



The full position and performance of a subsidiary (such as high gearing or losses) are revealed to users of the accounts

3.

Hydan

(a)

Consolidated income statement for Hydan for the year ended 31 March 2009

Revenue Cost of sales Gross profit Operating expenses Interest income Finance costs Profit before tax Income tax expense Profit for the year Attributable to: Owners of the parent ( ) Non-controlling interest (W5)

$000 103,200 (77,500) 222 25,700 (20,175) 150 (420) 222 5,255 (3,200) 222 2,055 3,605 (1,550)

Marks

1 See W2

See W2 1 1

1/2

See W5

Consolidated Statement of Financial Position for Hydan as at 31 March 2009 $000 Non-current assets Property, plant and equipment (18,400 + 9,500 + 900) Goodwill (W4) Investments (16,000-10,800 (W4) – 4,000 loan) Current assets (18,000 + 7,200 – 200 URP – 1,000 Interco) Total assets Equity and Liabilities Equity shares Share premium Retained earnings (W6) Non-controlling interest (W5) Non-current liabilities Bank loan Current liabilities (11,400 + 3,900 – 1,000 Interco) Total equity and liabilities

78

Downloaded by isavic Alsina ([email protected])

28,800 2,925 1,200 222 32,925 24,000 222 56,925 222 10,000 5,000 17,675 222 32,675 3,950 6,000 14,300 222 56,925 222

1 See W4 1

11/2

1/2 1/2

See W6

See W5

1/2

1

lOMoARcPSD|4664814

LSB_F7_Rev Kit:297mm x 210mm

28/10/09

13:37

Page 79

ANSWERS Workings W1

Group structure H

60%

S 1 Oct 2008 (owned 6m) W2

Consolidation schedule H $000 98,000 (76,000)

Revenues Cost of sales URP (W7) Extra dep’n Operating expenses Impt of GW Interest income Finance costs Income taxes W3

(11,800) 350 (420) (4,200)

(200) 1,000 3,875

Adj $000 (30,000) 30,000

Group $000 103,200 1/2

1

(77,500)

1 1/2

(200) 200

(20,175) 150 (420) (3,200)

1

Net assets of Systan @ Rep’g date $000 2,000 500 6,300

Share capital Share premium Retained earnings Fair value adj’t Dep’n on FV adj’t URP (W7) GW impairment

W4

S $000 35,200 (31,000) (200) (300) (8,000) (375)

1,200 (300) (200) (375) 222 9,125

@ Acq’n date $000 2,000 500 9,300

Movement $000 1/2

(6,300 + 3,000 post acq’n loss)

1,200

1/2

1/2

(3,875)

1/2 1/2 1/2

222 13,000

Goodwill

Consideration ($9 x 60% x 2,000) Fair value of non-controlling interest at acquisition Fair value of net assets of Salter at acquisition (W3) Goodwill Impairment charge ( ) Carrying value of goodwill

$000 10,800 5,500 222 16,300 (13,000) 222 3,300 (375) 222 2,925

1 1/2

See W3

79

Downloaded by isavic Alsina ([email protected])

lOMoARcPSD|4664814

LSB_F7_Rev Kit:297mm x 210mm

28/10/09

13:37

Page 80

FINANCIAL REPORTING (INTERNATIONAL) Tutorial note In this question, the NCI is measured at fair value.Therefore goodwill attributable to the NCI arises. The impairment loss of $375,000 is therefore attributable to the NCI and parent company in proportion to their shareholdings. Including this impairment in the subsidiary’s column in W2 and the subsidiary’s net assets at the reporting date in W3 ensures that the NCI is allocated their share of the loss and the group share is included within profits attributable to the owners of the parent company and group reserves. W5

Non-controlling interest $000 (1,550)

NCI in profit: 40% x (3,875) (W2) NCI in net assets: 40% x 9,125 (W3) NCI goodwill (5,500 – (40% x 13,000)) W6

3,650 300 222 3,950

for 40% + See W3 See W3 1/2

Group reserves $000 20,000 (2,325) 222 17,675

Hydan Group share of Systan’s post acquisition loss (60% x (3,875) (W3)) W7

1/2

1/2 1/2

for 60% + see W3

URP in inventory $4m x 5% = $200,000

(b)

In the six months prior to its acquisition by Hydan, Systan made a profit of $3.6million. In the six months after the acquisition, the company made a loss of $3million.This appears to be a result of the following: •

In the post acquisition period, 85% of revenue is derived from sales to Hydan. On these sales, Systan makes a 5% profit on selling price. Prior to the acquisition a profit margin of 25% was made consistently ($6m/$24m). This indicates that Hydan has imposed a low transfer price on Systan which has contributed to the loss for the period.



Operating expenses have increased from $1.2 million in the six months prior to the acquisition to $8m in the six months post acquisition.This may be the result of Hydan passing group expenses back to Systan or re-charging management costs.



Finance costs of $200,000 are the direct result of the loan from Hydan, made to finance an increase in production capacity for the benefit of Hydan. In a normal commercial situation, the effect of such interest costs on profits would be negated by increased gross profits as a result of the increase in production. In this instance, however, the low transfer price means this is not the case.



Overall, therefore, the acquisition appears to have had a negative effect on Systan’s operating performance. Furthermore, Systan appear to have increased the margin charged on goods sold to third parties to over 50% (W) which is likely to result in a loss of external customers.This will further exacerbate the problem in future years.

Working

Revenue Profit Margin

Hydan $30m 5% x 30m = $1.5m 5%

Third parties $5.2m $2.7m

Total $35.2m $4.2m

52%

12%

80

Downloaded by isavic Alsina ([email protected])

1 mark per valid point

lOMoARcPSD|4664814

LSB_F7_Rev Kit:297mm x 210mm

28/10/09

13:37

Page 81

ANSWERS

4.

Holdrite

(a)

Goodwill $000 Consideration Shares: (10,000 x 75%) x 2/3 x $6 Loan note: (10,000 x 75%) /250 x $100 Non-controlling interest at acquisition 25% x 34,000 (W2) Fair value of net assets of Staybrite at acquisition (W2) Goodwill at 1 April 2009 Impairment charge Goodwill at 30 September 2009

30,000 3,000 222 33,000 8,500 222 41,500 (34,000) 222 7,500 (750) 222 6,750

1 1 1/2

for 25% + see W2

See W2

1/2

Tutorial note In this question, the NCI is measured as a proportion of net assets.Therefore there is no NCI goodwill.The impairment loss of $750,000 is therefore attributable to the parent company only.As such it is charged against the parent’s profits in W3 below. Investment in Associate Cost Shares: (5,000 x 40%) x3/4 x $6 Cash: (5,000 x 40%) x $1 Investment at 1 April 2009 Share of post-acquisition profits 40% x 2,000 (W2) Investment at 30 September 2009 (b)

9,000 2,000 222 11,000 800 222 11,800

1 1

1

Consolidated income statement for Holdrite for the year ended 30 September 2009

Revenue Cost of sales Gross profit Operating expenses Interest expense Share of profit of associates (4,000 x 6/12 x 40%) Profit before tax Income tax expense Profit for the year Attributable to: Owners of the parent ( ) Non-controlling interest (25% x 4,000 (W3)) (c)

$000

$000 85,350 (48,750) 222 36,600 (15,730) (170) 800 222 21,500 (6,300) 222 15,200 14,200 1,000

2 See W3

See W3 1 + 1 for 40% 1

1 1 + 1 for 25%

Movement on consolidated retained earnings attributable to Holdrite

Brought forward at 1 October 2008 Profit attributable to Holdrine’s shareholders Dividends paid Carried forward at 30 September 2009

$000 18,000 14,200 (5,000) 222 27,200

1/2

1 1/2

81

Downloaded by isavic Alsina ([email protected])

lOMoARcPSD|4664814

LSB_F7_Rev Kit:297mm x 210mm

28/10/09

13:37

Page 82

FINANCIAL REPORTING (INTERNATIONAL) W1

Group structure H 1

75%

40% A

S 1 April 08 (owned 6m) W2

Net assets of Staybrite at acquisition $000 10,000 4,000 12,000 3,000 5,000 222 34,000

Share capital Share premium Retained earnings (7,500 + (9,000 x 6/12)) FV Adjustment land FV adjustment plant

W3

1/2

1 1/2 1/2

Consolidation schedule

Revenue Cost of sales URP (4m x 1/4) Extra depreciation Operating expenses Impt of GW Interest expense Income taxes

H $000 75,000 (47,400) (1,000) (10,480) (750) (170) (4,800)

S (6m) $000 20,350 (9,850)

Adj $000 (10,000) 10,000

Group $000 85,350 2 1

(500) (4,500)

(1,500) 222 4,000

82

Downloaded by isavic Alsina ([email protected])

(48,750)

1 1

(15,730) (170) (6,300)

1

lOMoARcPSD|4664814

LSB_F7_Rev Kit:297mm x 210mm

28/10/09

13:37

Page 83

ANSWERS

5.

Hedra Consolidated Statement of Financial Position for Hedra as at 30 September 2009 Non-current assets Property, plant and equipment (358 + 240 + 25 FV adj’t + 12 rev’n + 15 FV adj’t) Goodwill (W3) Investments Investment in associate (W6) Current assets Inventory (130 + 80) Trade receivables (142 + 97) Cash and bank Total assets Equity and Liabilities Equity shares (400 + 80 (W7)) Share premium (40 + 120 (W7)) Revaluation reserve (W5) Retained earnings (W5) Non-controlling interest (W4) Non-current liabilities Deferred tax (45 – (40m x 25%)) Current liabilities Trade payables (118 + 141) Bank overdraft Current tax payable Deferred consideration (acquisition of Salvador) Total equity and liabilities

Marks

$m 650

1/2 + 1 + 1/2 + 1

94 45 220 222 1,009

See W3

210 239 4 222 453 222 1,462 222

1/2

480 160 30 269 222 939 118

1

35 259 12 50 49 222 370 222 1,462 222

1/2

See W6

1/2 1/2

1 See W5 See W5

See W4

1

1/2 1/2 1/2

1

83

Downloaded by isavic Alsina ([email protected])

lOMoARcPSD|4664814

LSB_F7_Rev Kit:297mm x 210mm

28/10/09

13:37

Page 84

FINANCIAL REPORTING (INTERNATIONAL) Workings W1

Group structure H

60%

40% A

S S bought 1 Oct 2008 (owned 1 yr) A bought 1 April 2009 (owned 6m) W2

Net assets of Salvador @ Rep’g date $m 120 50 25 60 15

Share capital Share premium Revaluation reserve FV adjustment (land) Retained earnings Fair value adj’t – plant FV adj’t – deferred tax asset (40 x 25%) GW impairment

10 (20) 222 260

@ Acq’n date $m 120 50 20 20 20

Movement $m

5

1 1/2

1

10 222 240

1

15 222 20

1/2

Net assets of Aragon @ Rep’g date $000 100 300

Share capital Retained earnings W3

@ Acq’n date $000 100 250

Movement $000 50

200 + (100 x 6/12)

Goodwill $m Consideration Cash Deferred cash Fair value of non-controlling interest at acquisition Fair value of net assets of Salvador at acquisition (W2) Goodwill Impairment charge Carrying value of goodwill

84

Downloaded by isavic Alsina ([email protected])

195 49 222 244 110 222 354 (240) 222 114 (20) 222 94

1/2

1

1/2

See W2

1/2

lOMoARcPSD|4664814

LSB_F7_Rev Kit:297mm x 210mm

28/10/09

13:37

Page 85

ANSWERS Tutorial note In this question, the NCI is measured at fair value.Therefore goodwill attributable to the NCI arises. The impairment loss of $20,000 is therefore attributable to the NCI and parent company in proportion to their shareholdings. Including this impairment in the subsidiary’s net assets at the reporting date in W2 ensures that the NCI is allocated their share of the loss and the group share is included within group reserves. W4

Non-controlling interest

FV at acquisition NCI share of post-acquisition movement in reserves 40% x 20 (W2) W5

$m 110 8 222 118

1 1

Group retained earnings

Hedra Group share of Salvador’s post acquisition profits (60% x 15 (W2)) Group share of Aragon’s post acquisition profits (40% x 50 (W2))

Group revaluation reserve Hedra Revaluation of Hedra’s land and buildings Group share of Salvador’s post acquisition revaluation surplus (60% x 5 (W2))

$m 240 9 20 222 269 $m 15 12 3 222 30

1/2 for 60% + see W2

1/2

1

1/2 1/2

1

Tutorial note Increases in the fair value of non-current assets of the subsidiary since acquisition should be recorded as a revaluation reserve. W6

Investment in associate

Cost (W7) Group share of post acquisition profits 40% x 50 (W2) W7

$m 200 20 222 220

See W7 1/2

Share for share exchange Shares issued

40m x 2 = 80m Consideration: 300,000 x $2.50 = $200m Nominal value of new shares to be recorded in CSFP: $80m Premium to be recorded in CSFP: $120m

1 1/2

85

Downloaded by isavic Alsina ([email protected])

lOMoARcPSD|4664814

LSB_F7_Rev Kit:297mm x 210mm

28/10/09

13:37

Page 86

FINANCIAL REPORTING (INTERNATIONAL)

6.

Hosterling

(a)

Goodwill

Marks

$m Consideration 3/5 x 20,000 x 80% x $5 Non-controlling interest at acquisition 20% x 50,000 (W2) Fair value of net assets of Sunlee at acquisition (W2) Goodwill at 1 October 2008

1

48,000 10,000 222 58,000 (50,000) 222 8,000

1/2

for 20% + W2

See W2

Investment in Associate $000 Cost Cash: $3 x 6m Loan notes: 6m/100 x $100 Group share of post-acquisition loss 40% x 20m x 3/12 Investment at 30 September 2009 Impairment loss Carrying value at 30 September 2009 (b)

18,000 6,000 222 24,000

1

(2,000) 222 22,000 (500) 222 21,500

2

1

Consolidated income statement for Hosterling for the year ended 30 September 2009

Revenue Cost of sales Gross profit Distribution costs Administrative expenses Finance costs Share of profit of associates (W4) Profit before tax Income tax expense Profit for the year Attributable to: Owners of the parent ( ) Non-controlling interest (20% x 12,000 (W3))

86

Downloaded by isavic Alsina ([email protected])

$000 149,000 (89,000) 222 60,000 (6,000) (16,100) (2,100) (2,500) 222 33,300 (11,300) 222 22,000 19,600 2,400

1 1/2 See W3

1/2

See W3 1/2

See W6

1/2

1 1 + 1 for 20%

lOMoARcPSD|4664814

LSB_F7_Rev Kit:297mm x 210mm

28/10/09

13:37

Page 87

ANSWERS W1

Group structure H

80%

40% A

S S acquired 1 October 2008 (1 year) A acquired 1 July 2009 (3m) W2

Net assets of Sunlee at acquisition $000 20,000 18,000 4,000 3,000 5,000 222 50,000

Share capital Retained earnings FV adjustment intellectual property FV Adjustment land FV adjustment plant W3

1/2

1 1 1

Consolidation schedule

Revenue Cost of sales URP (7.5m x 25/125) Extra depreciation Distribution costs Administrative expenses Impt of GW in S Finance costs Income taxes

H $000 105,000 (68,000) (1,500) (4,000) (7,500) (1,600) (1,200) (8,700)

S $000 62,000 (36,500)

Adj $000 (18,000) 18,000

Group $000 149,000

1 1/2 1 1/2 1

(1,000) (2,000) (7,000) (900) (2,600) 222 12,000

(89,000) (6,000)

1 1/2 1/2

(16,100) (2,100) (11,300)

1 1/2 1/2

Tutorial note In this question, the NCI is measured as a proportion of net assets.Therefore there is no NCI goodwill.The impairment loss of $1,600,000 is therefore attributable to the parent company only.As such it is charged in H’s column in W3 W4

Share of losses of Associate $000 (2,000) (500) 222 (2,500)

40% x (20m x 3/12) Impairment (part (a))

1 1

87

Downloaded by isavic Alsina ([email protected])

lOMoARcPSD|4664814

LSB_F7_Rev Kit:297mm x 210mm

28/10/09

13:37

Page 88

FINANCIAL REPORTING (INTERNATIONAL)

7.

Horsefield Consolidated Statement of Financial Position for Horsefield as at 31 March 2009

Marks

$000 Non-current assets Property, plant and equipment (8,050 + 3,600 + 120 FV adjustment) Goodwill (W3) Licence (180 – 60) Investments (4,000 – 3,240 – 630 + 910) Investment in associate (W6) Current assets Inventory (830 + 340) Accounts receivable (520 + 290 – 40 Interco) Cash and bank (240 + 40 cash in transit)

11,770 1,190 120 1,040 717 222 14,837 1,170 770 280 222 2,220 222 17,057 222

Total assets Equity and Liabilities Equity shares Retained earnings (W5)

5,000 8,883 222 13,883 394

Non-controlling interest (W4) Non-current liabilities Loan notes (500 + 240) Current liabilities Accounts payable (420 + 960) Taxation (220 + 250) Overdraft Total equity and liabilities

1/2

1 1

1/2

See W5 See W4

1/2

30% A

S S bought 1 April 07 (owned 2 yrs) A bought 1 Oct 08 (owned 6m)

Downloaded by isavic Alsina ([email protected])

See W6

1,380 470 190 222 2,040 222 17,057 222

H

88

1 1 1/2

1/2

Group structure

90%

See W3

740

Workings W1

1 1/2

1/2 1/2

lOMoARcPSD|4664814

LSB_F7_Rev Kit:297mm x 210mm

28/10/09

13:37

Page 89

ANSWERS W2

Net assets of Sandfly

W3

@ Rep’g date $000 Share capital 1,200 Retained earnings 2,300 Fair value adj’t – property 120 FV adj’t – licence 180 Amortisation of licence (2/6 x 180) (60) 222 3,740 Goodwill

@ Acq’n date $000 1,200 800 120 180

Movement $m 1/2 1/2

1+1 1+1 1

1,440 222 2,300

$000 Consideration 90% x 1,200,000 x $3 Fair value of non-controlling interest at acquisition Fair value of net assets of Salvador at acquisition (W2) Goodwill W4

See W2

$000 250 144 222 394

1/2

1 for 10% + W2

$000 7,500 1,296 90 (3) 222 8,883

1 1/2

for 90% + W2 1 1

Investment in associate

Cost (30% x 600,000 x $3.50) Group share of post acquisition profits (30% x (600 x 6/12)) Group share of URP (30% x $1,000 (W7)) W7

1

Group retained earnings

Horsefield Group share of Sandfly’s post acquisition profits (90% x 1,440 (W2)) Group share of Anthill’s post acquisition profits (30% x (600 x 6/12)) Group share of URP (30% x $1,000 (W7)) W6

1

Non-controlling interest

FV at acquisition NCI share of post-acquisition profits (10% x 1,440 (W2))

W5

3,240 250 222 3,490 (2,300) 222 1,190

$m 630 90 (3) 222 717

1

1 1

URP in inventory

2/3 x $65,000 x 30/130

$000 1

89

Downloaded by isavic Alsina ([email protected])

lOMoARcPSD|4664814

LSB_F7_Rev Kit:297mm x 210mm

28/10/09

13:37

Page 90

FINANCIAL REPORTING (INTERNATIONAL) Tutorial note Where the associate is the selling company, the group share of the URP is deducted from both group reserves and the investment in the associate. (b)

An associate is defined as an entity over which an investor has a significant influence. Significant influence is the power to participate in the financial and operating policy decisions of the investee.

1 mark per valid point

Significant influence is presumed to exist where an investor has at least 20% of the voting power in another company (but normally not more than 50% as this indicates control). This presumption can however be rebutted where it can clearly be demonstrated that the investor does not have significant influence. •

Significant influence can be evidenced in a number of different ways, for example:



Participation in policy-making decisions such as the amount of a dividend



Representation on the board of directors



Material transactions between the investor and investee



Interchange of management personnel between the two companies



Provision by one company to another of essential technical information.

8.

Winger

(a)

Income statement for Winger for the year ended 31 March 2009

Marks

Revenue (358,450 – 27,000) Cost of sales (W1) Gross profit Distribution expenses Administration expenses Profit on disposal of land and buildings (W2) Write off of research project Profit before interest and tax Finance costs (7,200 + (50m x 8%)) Profit before tax Income tax (15,000 – 2,200) Profit for the year

$000 331,450 (208,550) 222 122,900 (28,700) (15,000) 15,000 (30,000) 222 64,200 (11,200) 222 53,000 (12,800) 222 40,200

1/2

See W1

1/2 1/2 1/2 1/2

1 1/2

1

Tutorial note The writing off of the development costs should not be treated as a prior period adjustment through reserves as it does not constitute a change in accounting policy. Winger’s accounting policy has in fact not changed, the costs are being written off as they no longer meet the capitalisation criteria of IAS 38. Statement of changes in equity

Balance at 1 April 2008 Total comprehensive income Dividends paid Transfer from revaluation surplus (W2) Balance at 31 March 2009 90

Downloaded by isavic Alsina ([email protected])

Retained earnings $000 71,600 40,200 (12,000) 30,000 222 129,800

1/2 1/2

1

lOMoARcPSD|4664814

LSB_F7_Rev Kit:297mm x 210mm

28/10/09

13:37

Page 91

ANSWERS (b)

Statement of financial position of Winger at 31 March 2009 $000 Non-current assets Property (W4) Plant and equipment (W4) Current assets Inventory (28,240 + 22,500 sale or return) Receivables (55,000 – 27,000 sale or return) Cash Total assets Equity and liability 25c Equity shares Retained earnings (part (a))

194,000 160,000 222 354,000 50,740 28,000 10,660 222 89,400 222 443,300 222 150,000 129,800 222 279,800

Non-current liabilities Finance lease (W3) 8% loan notes Current liabilities Trade accounts payable Income tax Finance lease (W3) Loan interest accrual ((50,000 x 8%) – 2,000) Total equity and liabilities

47,200 50,000 29,400 15,000 20,000 2,000 222 66,400 222 443,300 222

1 1 1/2

1 1 1/2

1/2 1/2

1 1/2

1/2

1 1 1

Workings (W1) Cost of sales

Draft per TB Sale or return inventory (27,000 x 100/120) Depreciation of plant (W5) Depreciation of property (W5)

$000 185,050 (22,500) 40,000 6,000 222 208,550

1 1/2 1/2

(W2) Disposal of revalued property

Proceeds Carrying value Profit on disposal Carrying value at disposal Historic cost Transfer from revaluation reserve to retained earnings

$000 95,000 (80,000) 222 15,000 80,000 (50,000) 222 30,000

91

Downloaded by isavic Alsina ([email protected])

lOMoARcPSD|4664814

LSB_F7_Rev Kit:297mm x 210mm

28/10/09

13:37

Page 92

FINANCIAL REPORTING (INTERNATIONAL) (W3) Finance lease b/f

Repayment

$000 80,000 67,200

y/e 31 Mar 09 y/e 31 mar 10

c/f

$000 (20,000) (20,000)

$000 60,000 47,200

Interest at 12% $000 7,200 5,664

c/f $000 67,200 52,864

Therefore: Liability due > 1 year is $47,200 Liability due < 1 year is $20,000 (67,200 – 47,200) (W4) Non-current assets Property $000 80,000 200,000 (80,000) 222 200,000 – 6,000 222 6,000 194,000

Cost b/f Additions Disposals Cost c/f Depreciation b/f Depreciation charge (W5) Depreciation c/f Carrying value c/f

P&E $000 154,800 80,000 – 222 234,800 34,800 40,000 222 74,800 160,000

(W5) Depreciation charge $000 Property

Plant

(c)

Land Heating system Lifts Building Leased Old plant

Not depreciated 20m / 10 years 30m / 15 years 100m / 50 years 80m / 5 years (154,800-34,800) x 20%

2,000 2,000 2,000 222 6,000 16,000 24,000 222 40,000

IAS 16 Property, Plant and Equipment requires that the depreciable value of all assets other than land is written off over the useful life of the asset. Depreciable value is calculated as cost less the expected residual value at the end of the asset’s useful life. This requirement holds even where: •

A property is maintained to a high standard



The current market value of the property exceeds its cost

On this basis, the previous policy of Winger was inappropriate and the directors were correct to start depreciating the new property One potentially valid argument for non-depreciation is materiality: accounting standards are not applicable to items which are not material. In the case of depreciation, however, this materiality level refers to both the charge for the year and accumulated depreciation. It is likely that even where the depreciation charge for the year is immaterial, accumulated depreciation will not be.

92

Downloaded by isavic Alsina ([email protected])

1 mark per valid point

lOMoARcPSD|4664814

LSB_F7_Rev Kit:297mm x 210mm

28/10/09

13:37

Page 93

ANSWERS

9.

Petra

(a)

Income statement for Petra for the year ended 30 September 2009

Revenue (197,800 – 12,000 + 1,000 (W1)) Cost of sales (W2) Gross profit Distribution costs Administration expenses Profit before interest and tax Finance costs (50m x 6%) Profit before tax Income tax ( W3) Profit for the year (b)

Marks

$000 331,450 (128,100) 222 58,700 (17,000) (18,000) 222 23,700 (3,000) 222 20,700 (7,600) 222 13,100

2 See W2

1/2 1/2

1

See W3

Statement of financial position of Petra at 30 September 2009 $000 Non-current assets Land and Buildings (W5) Plant and equipment (W5) Development costs (W6) Current assets Inventory Receivables Cash Non-current assets held for sale (W4) Total assets Equity and liability Ordinary share capital Share premium Retained earnings (34,000 + 13,100) Non-current liabilities 6% redeemable loan Deferred tax Current liabilities Payables Income tax Loan interest accrual ((50,000 x 6%) – 1,500) Total equity and liabilities

82,000 24,000 18,000 222 124,000 21,300 24,000 11,000 222 56,300 6,900 222 187,200 222

40,000 12,000 47,100 222 99,100

1/2

1 1/2

1/2 1/2 1/2

1

1/2

1

50,000 17,600

1/2

15,000 4,000 1,500 222 20,500 222 187,200 222

1/2

1

1 1

93

Downloaded by isavic Alsina ([email protected])

lOMoARcPSD|4664814

LSB_F7_Rev Kit:297mm x 210mm

28/10/09

13:37

Page 94

FINANCIAL REPORTING (INTERNATIONAL) Workings (W1) Commission sales $000 12,000 11,000

Revenue Cost of sales

2222 1,000

Profit

Deduct from revenue in IS Deduct from cost of sales in IS (being 8,000 + 3,000) Include in revenue

Tutorial note IAS 18 Revenues requires that only the commission element of commission sales is recorded as income. (W2) Cost of sales $000 114,000 (8,000) (3,000) 6,000 2,000 3,100 14,000 222 128,100

Draft per TB Commission sales – cost of sales Commission sales – remitted amount Depreciation of plant (W5) Depreciation of land and buildings (W5) Loss on transfer of asset to held for sale (W4) Impairment of development costs

1/2 1/2 1/2 1/2 1/2

1 1/2

(W3) Income tax charge $000 4,000 1,000

Charge for the year Underprovision from previous year Movement in deferred tax provision (17,600 – 15,000)

2,600 222 7,600

(W4) Non-current assets held for sale $000 10,000 6,900 3,100

Carrying value of asset at transfer (16,000 – 6,000) Fair value less costs to sell (7,500 x 92%) Loss on transfer to held for sale (W5) Non-current assets L&B $000 100,000 – 222 100,000 16,000 – 2,000 222 18,000 82,000

Cost b/f Disposals Cost c/f Depreciation b/f Disposals Depreciation charge Depreciation c/f Carrying value c/f Depreciation charge L&B Depreciation charge P & E

(100,000 – 40,000)/30yrs = 2,000 [(66-16) – (26-6)] x 20% = 6,000

94

Downloaded by isavic Alsina ([email protected])

P&E $000 66,000 (16,000) 222 50,000 26,000 (6,000) 6,000 222 26,000 24,000

1/2 1/2

1

lOMoARcPSD|4664814

LSB_F7_Rev Kit:297mm x 210mm

28/10/09

13:37

Page 95

ANSWERS (W6) Development costs $000 40,000 (8,000) 222 32,000 (14,000) 222 18,000

Cost b/f Amortisation b/f Impairment ( ) Carrying value (c)

Basic EPS Profits attributable to ordinary shareholders 22222222222222222 Number of ordinary shares 13,100,000 22222 160,000,000 Fully diluted EPS Options

Proceeds Number of shares at MV (7.2m/90c) Therefore number of free shares ( )

8.2c

$7,200,000 8,000,000 16,000,000 24,000,000

2

1

Therefore DEPS: $13,100,000 2222222222 (160,000,000 + 16,000,000)

7.4c

2

10. Allgone (a)

Income statement for Allgone for the year ended 31 March 2009

Revenue (236,200 – 8,000 (W1)) Cost of sales (W2) Gross profit Operating expenses (12,400 + 3,000 L&B depreciation W3) Profit before interest and tax Finance costs (W4) Profit before tax Income tax ( W5) Profit for the year

Marks

$000 228,200 (147,000) 222 81,200 (15,400) 222 65,800 (5,850) 222 59,950 (13,100) 222 46,850

1/2

See W2

1/2

See W4

See W5

Statement of comprehensive income for Allgone for the year ended 31 March 2009

Profit for the year Surplus on revaluation of land and buildings (W6) Loss on AFS investments Total comprehensive income for the year

$000 46,850 40,000 (1,200) 222 85,650

1/2 1/2

95

Downloaded by isavic Alsina ([email protected])

lOMoARcPSD|4664814

LSB_F7_Rev Kit:297mm x 210mm

28/10/09

13:37

Page 96

FINANCIAL REPORTING (INTERNATIONAL) (b)

Statement of changes in equity for Allgone for the year ended 31 March 2009

At 1 April 2008 Prior period adjustment (W7)

(c)

Share capital $000 60,000

Restated Total comprehensive income Transfer (W3)

222 60,000 –

At 31 March 2009

222 60,000

Retained earnings $000 4,350 (32,000) 222 (27,650) 46,850 1,000 222 20,200

Revaluation reserve $000 5,000

1/2

1

222 5,000 38,800 (1,000) 222 42,800

2 1 1/2

Statement of financial position of Allgone at 31 March 2009 $000 Non-current assets Land and Buildings (W8) Plant and equipment (W8) Software (W8) Available-for-sale investments (W9) Current assets Inventory (8,500 – 200 write down + 6,000 Funders) Receivables Total assets Equity and liability Ordinary share capital (SOCE) Revaluation reserve (SOCE) Retained earnings (SOCE) Non-current liabilities 10% redeemable preference shares 12% loan notes Deferred tax Current liabilities Bank Payables Income tax Funders loan Accruals (W4) Total equity and liabilities

96

Downloaded by isavic Alsina ([email protected])

127,000 48,000 2,000 10,800 222 187,800

1

14,300 23,000 222 37,300 222 225,100 222

1 1/2

60,000 42,800 20,200 222 123,000

1/2

20,000 40,000 4,800

1/2

350 15,200 11,300 8,000 2,450 222 37,300 222 225,100 222

1/2

1 1 1

1/2

1/2 1/2

1/2

1

1/2 1/2

1 1

lOMoARcPSD|4664814

LSB_F7_Rev Kit:297mm x 210mm

28/10/09

13:37

Page 97

ANSWERS Workings (W1) Funders Bank The substance of the transaction with Funders Bank is that it is a loan of $8m secured on goods.The $8m recorded as revenue should therefore be reversed and instead recorded as a current liability.The facilitating fee should be treated as an accrued finance cost. (W2) Cost of sales $000 19,450 127,850 (8,300) (6,000) 12,000 2,000 222 147,000

Opening inventory Purchases Closing inventory Cost of goods re Funders Depreciation of plant and equipment (W3) Depreciation of software (W3)

1/2

1/2 1/2 1/2 1/2

(W3) Depreciation

Building

Plant and equipment Software

105,000 / (40 – 5 yrs) Historic cost depreciation: 80,000/40 years = 2,000 Therefore reserves transfer of $1,000 per annum from revaluation reserve to retained earnings. (84,300 – 24,300) x 20% sum of digits = n(n+1)/2 = 5 x 6/2 = 15 Therefore in year 3 3/15 x 10,000

$000 3,000

12,000

2,000

(W4) Finance costs Income statement $000 Redeemable preference shares (10% x 20m) 12% loan notes (12% x 40m x 9/12) Funder Bank’s fee

2,000 3,600 250 222 5,850

Paid per TB $000 1,000 (2,400) –

Accrual $000 1,000 1,200 250 222 2,450

1/2 1/2 1/2

(W5) Taxation

Charge for year Movement on deferred tax Provision req’d ($16m x 30%) Provision per TB Tax charge (IS)

$000 11,300 4,800 (3,000) 222 1,800 222 13,100

1/2

1/2

97

Downloaded by isavic Alsina ([email protected])

lOMoARcPSD|4664814

LSB_F7_Rev Kit:297mm x 210mm

28/10/09

13:37

Page 98

FINANCIAL REPORTING (INTERNATIONAL) (W6) Surplus on revaluation of land and buildings

Cost Depreciation to revaluation 80,000 x 5/40yrs CV at date of revaluation Revalued amount Revaluation surplus

Land $000 20,000

Buildings $000 80,000

222 20,000 25,000 222 5,000

(10,000) 222 70,000 105,000 222 35,000

Total $000

40,000

(W7) Fraud Although not highlighted until the current year, the fraud occurred in the previous financial year. It must therefore be accounted for as a prior period adjustment through the statement of changes in equity. (W8) Non-current assets

Cost b/f Revaluation Cost c/f Depreciation b/f Revaluation Charge for the year (W 3) 3,000 CV in SFP

Land and Plant and buildings equipment $000 $000 100,000 84,300 30,000 – 222 222 130,000 84,300 10,000 (W6) 24,300 (10,000) – 3,000 12,000 222 222 36,300 8,000 127,000 48,000

Software $000 10,000 – 222 10,000 6,000 – 2,000 222 2,000

(W9) Available for sale investments

Fair value at 1 April 2008 Fair value at 31 March 2009 (2.25/2.50 x 12m) Fall in FV charged as other comprehensive income against previous revaluation surplus of $5m

98

Downloaded by isavic Alsina ([email protected])

$000 12,000 10,800 222 1,200

lOMoARcPSD|4664814

LSB_F7_Rev Kit:297mm x 210mm

28/10/09

13:37

Page 99

ANSWERS

11. Tadeon (a)

Statement of comprehensive income for Tadeon for the year ended 30 September 2009

Revenue Cost of sales (W2) Gross profit Investment income Operating expenses (40,000 + 1,200 operating lease rental) Finance costs (2,750 (W3) + 1,500 (W4)) Profit before tax Income tax ( W5) Profit for the year Other comprehensive income Surplus on revaluation (W6) Deferred tax on revaluation surplus Total comprehensive income for the year (b)

Marks

$000 277,800 (144,000) 222 133,800 2,000 (41,200) (4,250) 222 90,350 (36,800) 222 53,550 20,000 (4,000) 222 69,550

1/2

See W2

1/2

1 2

See W5

1 1

Statement of financial position of Tadeon at 30 September 2009 $000 Non-current assets Leasehold property Plant and equipment (W7) Vehicles (W7) Investments Current assets Inventory Receivables

200,000 84,000 15,000 42,000 222 341,000 33,300 53,500 222 86,800 222 427,800 222

Total assets

1/2

1 1 1/2

1/2 1/2

99

Downloaded by isavic Alsina ([email protected])

lOMoARcPSD|4664814

LSB_F7_Rev Kit:297mm x 210mm

28/10/09

13:37

Page 100

FINANCIAL REPORTING (INTERNATIONAL) Equity and liability Ordinary share capital (150,000 + 50,000 (W8)) Share premium (W8) Revaluation reserve (20,000 – 4,000) Retained earnings (18,600 + 53,550 – 30,000 dividend)

200,000 28,000 16,000 42,150 222 286,150

Non-current liabilities 2% Loan note (W3) Deferred tax (20% x 74m) Finance lease obligation (W4) Current liabilities Bank Payables Income tax Finance lease obligation (W4)

51,750 14,800 10,500 1,900 18,700 38,000 6,000 222 64,600 222 427,800 222

Total equity and liabilities

1 1 1 1 1/2

1 1/2 1 1

1/2 1/2

1 1

Workings (W1) Cost of sales $000 118,000 9,000 12,000 5,000 222 144,000

Per TB Depreciation of leasehold (W2) Depreciation of plant and equipment (W2) Depreciation of vehicles (W2) (W2) Depreciation

Leasehold Plant and equipment Vehicles

225,000 / 25 years (181,000 – 85,000) x 12 ½ % 20,000 / 4 years

$000 9,000 12,000 5,000

(W3) Loan note

y/e 30 Sept 2009

B/f $000 50,000

Interest at 5.5% Interest paid (TB) $000 $000 2,750 (1,000)

c/f $000 51,750

(W4) Finance lease

y/e 30 Sept 2009 y/e 30 Sept 2010 Due > 1 year Due < 1 year

b/f $000 20,000 16,500

Repayment $000 (5,000) (6,000)

(16,500 – 10,500)

c/fInterest at 10% $000 $000 15,000 1,500 10,500 1,050 $10,500 $6,000

100

Downloaded by isavic Alsina ([email protected])

c/f $000 16,500 11,550

1/2 1/2 1/2 1/2

lOMoARcPSD|4664814

LSB_F7_Rev Kit:297mm x 210mm

28/10/09

13:37

Page 101

ANSWERS (W5) Tax charge

Tax charge for year Deferred tax provision @ 1 Oct 08 @ 30 Sept 09 (54m x 20%)

$000 38,000

1/2

12,000

1

10,800 222 (1,200) 222 36,800

Tutorial note Of the taxable temporary differences of $74m, $20m relate to the revaluation of the leasehold property. In accordance with IAS 12, deferred tax arising on an item of other comprehensive income is dealt with as other comprehensive income.Therefore the tax charge in the income statement includes only the deferred tax relating to the other taxable temporary differences. (W6) Revaluation surplus $000 Valuation Carrying value Cost Dep to 1.10.08 Dep for yr (W2)

$000 200,000

225,000 (36,000) (9,000) 222 (180,000) 222 20,000

Surplus on revaluation (W7) Plant and equipment and vehicles P&E $000 181,000 – 222 181,000 85,000 12,000 222 97,000 84,000

Cost b/f Additions Cost c/f Depreciation b/f Charge (W2) Depreciation c/f CV at 30 Sept 09

Vehicles $000 – 20,000 222 20,000 – 5,000 222 5,000 15,000

(W8) Suspense account

Per TB Dividend Rights issue Nominal value Premium

(5% x 80c x 150m/0.2) (150m/3) [(150m/0.2)/3 x 12c] – issue costs of 2,000

$000 48,000 30,000 (50,000) (28,000) 222 nil

101

Downloaded by isavic Alsina ([email protected])

lOMoARcPSD|4664814

LSB_F7_Rev Kit:297mm x 210mm

28/10/09

13:37

Page 102

FINANCIAL REPORTING (INTERNATIONAL)

12. Kala (a)

Statement of comprehensive income for Kala for the year ended 31 March 2009 $000 278,400 (115,700) 222 162,700 10,800 (15,500) (10,000) 222 148,000 (29,900) 222

Revenue Cost of sales (W1) Gross profit Investment income (4,500 + 6,300 gain on inv’t property) Operating expenses Finance costs (3,000 (W3) + 7,000 (W4)) Profit before tax Income tax ( 28,300 + (14,100 – 12,500)) Profit for the year Other comprehensive income Surplus on revaluation (W6)

118,100 45,000 222 163,100

Total comprehensive income for the year (b)

1/2

See W2

1 1/2

2

1 1/2

1 1/2

Statement of changes in equity for Kala for the year ended 31 March 2009 Share capital $000 150,000

Balance at 1 April 2008 Total comprehensive income Dividends

222 150,000

Balance at 31 March 2009 (c)

Marks

Retained earnings $000 119,500 118,100 (15,000) 222 222,600

Revaluation surplus $000 – 45,000

1 1 1

222 45,000

1

Statement of financial position of Kala at 31 March 2009 $000 Non-current assets Land and Buildings (W6) Plant and equipment (W5) Investment Property (90,000 x 1.07) Current assets Inventory Receivables Total assets

102

Downloaded by isavic Alsina ([email protected])

250,000 184,100 96,300 222 530,400

1

43,200 53,200 222 96,400 222 626,800 222

1/2

1 1

1/2

lOMoARcPSD|4664814

LSB_F7_Rev Kit:297mm x 210mm

28/10/09

13:37

Page 103

ANSWERS Equity and liability Ordinary share capital Revaluation reserve (SOCE) Retained earnings (SOCE) Non-current liabilities 8% Loan note Deferred tax Finance lease obligation (W4) Current liabilities Bank Payables Income tax Finance lease obligation (W4) Accruals Total equity and liabilities

150,000 45,000 222,600 222 417,600

1/2

50,000 14,100 55,000

1/2

5,400 33,400 28,300 22,000 1,000 222 90,100 222 626,800 222

1/2

1/2 1/2

1 1

1/2 1/2

1 1/2

Workings (W1) Cost of sales $000 37,800 78,200 (43,200) 18,400 19,500 5,000 222 115,700

Opening inventory Purchases Closing inventory Depreciation of leased plant (W2) Depreciation of owned plant (W2) Depreciation of land and buildings (W2)

– – – – – –

(W2) Depreciation

Leased plant Owned plant

92,000/5 years (156,000 – 26,000) x 15%

Land and buildings

175m/(50 – 15) years

$000 18,400 19,500 222 37,900 5,000

(W3) Finance costs – loan notes Income statement $000 3,000

8% x 50,000 x 9/12

Paid per TB $000 (2,000)

Accrual $000 1,000

(W4) Finance lease

y/e 30 March 09 y/e 30 March 10 Due > 1 year Due < 1 year

b/f $000 92,000 77,000

Repayment $000 (22,000) (22,000)

c/fInterest at 10% $000 $000 70,000 7,000 55,000 5,500

(77,000 – 55,000)

c/f $000 77,000 60,500 $55,000 $22,000 103

Downloaded by isavic Alsina ([email protected])

lOMoARcPSD|4664814

LSB_F7_Rev Kit:297mm x 210mm

28/10/09

13:37

Page 104

FINANCIAL REPORTING (INTERNATIONAL) (W5) Plant and equipment $000 156,000 92,000 222 248,000 (26,000 222 184,100

B/f at 1 April 08 Additions (lease) Depreciation b/f Depreciation charge (W2) (37,900) (W6) Land and buildings

Historic cost Depreciation to 1 April 2008 Revaluation surplus ( ) Valuation at 1 April 2008 Depreciation for the year (W2) CV at 31 March 2009

Land $000 70,000 – 70,000 10,000 222 80,000 – 222 80,000

Buildings $000 200,000 (60,000) 140,000 35,000 222 175,000 (5,000) 222 170,000

Total

45,000

250,000

13. Telenorth (a)

i) Income statement for Telenorth for the year ended 30 September 2009

Revenue Cost of sales (W1) Gross profit Investment income Distribution expenses Administration expenses (34,440 + 10,160 (W3) – 2,400 (W5)) Finance costs (W4) Profit before tax Income tax ( 23,400 + 1,200) Profit for the year

104

Downloaded by isavic Alsina ([email protected])

$000 283,460 (155,170) 222 128,290 1,500 (22,300) (42,200) (1,656) 222 63,634 (24,600) 222 39,034

Marks

1/2

See W1

1/2 1/2

1 1/2 1 1/2

1

lOMoARcPSD|4664814

LSB_F7_Rev Kit:297mm x 210mm

28/10/09

13:37

Page 105

ANSWERS ii) Statement of financial position of Telenorth at 30 September 2009 $000 Non-current assets Land and Buildings (W6) Plant and equipment (W6) Computer system (W6) Investments Current assets Inventory (W2) Trade accounts receivable (35,700 + 12,000 (W5)) Total assets Equity and liability Ordinary share capital (20,000 + 10,000 (W7)) Share premium (W7) Revaluation reserve (3,400 – 1,000) Retained earnings (14,160 + 39,034 – 2,000 ord divi) Non-current liabilities 6% Loan note Deferred tax (5,200 + 2,200) 8% redeemable preference shares Current liabilities Bank Trade accounts payable Income tax Loan (W5) Accruals (W4) Total equity and liabilities

36,000 32,200 15,240 34,500 222 117,940

1/2

16,680 47,700 222 64,380 222 182,320 222

1

30,000 16,000 2,400 51,194 222 99,594

1/2 1/2 1/2

1

1 1/2 1/2

1

10,000 7,400 12,000

1/2

1,680 17,770 23,400 9,600 876 222 53,326 222 182,320 222

1/2

1/2 1/2

1/2 1/2

1 1

Workings (W1) Cost of sales

Opening inventory Purchases Closing inventory (W2) Depreciation of buildings (W3) Depreciation of plant and equipment (W3)

$000 12,400 147,200 (16,680) 2,250 10,000 222 155,170

1/2 1/2 1/2 1/2 1/2

105

Downloaded by isavic Alsina ([email protected])

lOMoARcPSD|4664814

LSB_F7_Rev Kit:297mm x 210mm

28/10/09

13:37

Page 106

FINANCIAL REPORTING (INTERNATIONAL) (W2) Closing inventory $000 16,000

At 4 October 2009 Add back: Normal sales 1,400 x 100/140 Sale or return sales 650 x 100/130 Deduct: Purchases

1,000 500 (820) 222 16,680

(W3) Depreciation

P&E Computer system Land and buildings

$000 10,000 10,160 2,250

(55,000 – 5,000) / 5 years (35,000 – 9,600) x 40% 56,250/25years

(W4) Finance costs Income statement $000 Loan notes 6% x 10,000 600 Redeemable preference shares 8% x 12,000 960 Factoring fee (W5) 96 222 1,656

Paid per TB $000 (300) (480) –

Accrual $000 300 480 96 222 876

(W5) Kwikfinance Entries recorded

Entry should have been Therefore correcting entries

Dr Dr Cr Dr Cr Dr Cr Cr

Admin Cash (80% x 12,000) Receivables Cash Loan Receivables Admin expense Loan

2,400 9,600 12,000 9,600 9,600 12,000 2,400 9,600

Interest on the loan arrangement should also be recorded: Dr Cr

Finance costs (9,600 x 1%) Accrual

96 96

Tutorial note The substance of this arrangement is that Kwikfinance has made a loan to Telenorth, secured on its accounts receivables book.The risks and rewards of the receivables have not been transferred to Kwikfinance as any accounts unpaid after 4 months are transferred back to Telenorth. It is therefore incorrect to derecognise the receivables.

106

Downloaded by isavic Alsina ([email protected])

lOMoARcPSD|4664814

LSB_F7_Rev Kit:297mm x 210mm

28/10/09

13:37

Page 107

ANSWERS (W6) Non-current assets Building $000 56,250 (18,000) (2,250) 222 36,000

Cost b/f Depreciation b/f Depreciation for the year CV

Plant and equipment $000 55,000 (12,800) (10,000) 222 32,200

Computer system $000 35,000 (9,600) (10,160) 222 15,240

(W7) Suspense and share issues Share capital $000 Per TB Options NV 4m x $1 Premium 4m x $1 Rights issue NV 6m x $1 Premium 6m x $2

Share premium $000

4,000

(b)

4,000

(4,000) (4,000)

12,000 222 16,000

(6,000) (12,000) 222 nil

6,000 222 10,000

Suspense $000 26,000

Earnings per share Basic EPS

Profits attributable to ordinary shareholders 22222222222222222 Number of ordinary shares in issue

Number of ordinary shares in issue Date 1 Oct X0 – 30 June 09 1 July X1 – 30 Sept 09 Therefore Basic EPS

Number of shares in issue 24,000 30,000

Bonus fraction X 4/3.80 (W)

Time apportionment X 9/12 X 3/12

$39,034,000 22222 26,447,000

Weighted average shares 18,947 7,500 222 26,447 = 148c

3 1 1 for profit from IS

Working Bonus fraction

TERP

So bonus fraction is

MV pre rights 22222 TERP ‘old’ shares ‘new shares

4 x $4 1 x $3 2 5 $19/5 gives TERP of $3.80 $4.00 22 $3.80

16 3 2 19

107

Downloaded by isavic Alsina ([email protected])

lOMoARcPSD|4664814

LSB_F7_Rev Kit:297mm x 210mm

28/10/09

13:37

Page 108

FINANCIAL REPORTING (INTERNATIONAL)

14. Tourmalet (a)

Sale of plant

1 mark per valid point

Tourmalet has sold an item of plant during the year and then leased it back for the remainder of its useful life. The period of the lease indicates that this is a finance lease. As Tourmalet will continue to use the asset for the remainder of its useful life, it has not relinquished the risks and rewards of ownership of the asset and should therefore not derecognise it. The commercial substance of the sale and leaseback transaction is in fact that of a financing arrangement, whereby Tourmalet have been loaned $50million secured on the item of plant. This transaction should therefore not be accounted for as a sale and separate leaseback as the directors propose. Instead:

(b)



The asset should continue to be held in the statement of financial position at book value of $40million (less subsequent depreciation)



The proceeds received of $50million should be recorded as a loan in the statement of financial position



Interest arising at 12% per annum should be recorded as a finance cost in the income statement

Income statement for Tourmalet for the year ended 30 September 2009 $000 247,800 (128,800) 222 119,000 1,200 (26,400) (20,000) (200)

Revenue (313,000 – 50,000 – 15,200 discont’d ops) Cost of sales (W1) Gross profit Investment income Distribution expenses Administration expenses (23,200 – 3,200 discont’d ops) Other expenses (10,000 – 9,800 investment property) Finance costs Redeemable preference dividend (6% x 30,000) Loan (12% x 50,000 x 6/12)

1,800 2,000 222

Profit before tax Income tax (9,200 – 2,100 overprovision) Profit for the year from continuing activities Loss for year from discontinued operations (W4) Profit for the year

108

Downloaded by isavic Alsina ([email protected])

Marks

2 See W1

1/2 1/2

1 1

1

(3,800) 222 69,800 (7,100) 222 62,700 (5,500) 222 57,200

1

1

See W5

lOMoARcPSD|4664814

LSB_F7_Rev Kit:297mm x 210mm

28/10/09

13:37

Page 109

ANSWERS Workings (W1) Cost of sales $000 26,550 158,450 (26,000) (40,000) (16,000) 8,000 14,800 3,000 222 128,800

Opening inventory Purchases Closing inventory (W2) Cost of plant (sale and leaseback transaction) Costs relating to discontinued operations Depreciation of leased plant (W3) Depreciation of owned plant (W3) Depreciation of buildings (W3)

1/2 1/2

1 1 1/2

1 1 1

(W2) Closing inventory $000 28,500 (2,500) 222 26,000

Closing inventory Write down for slow moving goods (W3) Depreciation

Leased plant Other plant Buildings

$000 8,000 14,800 3,000

40m / 5 years (98,600 – 24,600) x 20% (150,000 – 30,000)/40 years

(W4) Discontinued operations $000 15,200 (16,000) (3,200) (1,500) 222 (5,500)

Revenue Cost of sales Operating expenses Termination of lease penalty

(c)

1/2 1/2 1/2

1

Statement of changes in equity for Tourmalet for the year ended 30 September 2009

At 1 October 2008 Total comprehensive income Transfer Dividend At 30 Sept 2009

Share capital $000 50,000

222 50,000

Retained earnings $000 47,800 57,200 500 (2,500) 222 103,000

Revaluation reserve $000 18,500

Total $000 116,300 57,200

(500)

1/2 1/2 1/2

1

222 18,000

222 171,000

1/2

109

Downloaded by isavic Alsina ([email protected])

lOMoARcPSD|4664814

LSB_F7_Rev Kit:297mm x 210mm

28/10/09

13:37

Page 110

FINANCIAL REPORTING (INTERNATIONAL)

15. Wellmay

Marks

Statement of comprehensive income for Wellmay for the year ended 31 March 2009 $000 3,700 (2,417) 222 1,283 20 (478) (25) (113) 222 687 (390) 222 297 190 222 487

Revenue (4,200 – 500 (W1)) Cost of sales (2,700 – 200 (W1) – 75 (W2) – 8 (W3)) Gross profit Investment income Operating expenses (470 + 8 (W3)) Loss on investment property (W3) Finance costs (55 + 50 (W1) – 48 + 56 (W4)) Profit before tax Income tax ( 360 + 30 (W5)) Profit for the year Surplus on revaluation of factory (W3) Total comprehensive income

1 2

1/2

1 1 1 1/2

1

1

Statement of changes in Equity for Wellmay for the year ended 31 March 2009

b/f Total comprehensive income Bonus issue Loan note issue (W4) Dividends

Share capital $000 1,200

Equity in loan $000 –

300

Retained earnings $000 2,615 297 (300)

Revaluation reserve $000 350 190

c/f

222 40

(400) 222 2,212

1/2

1 1

40 222 1,500

Total $000 4,165 487

222 540

40 (400) 222 4,292

1 1/2 1/2

Statement of financial position of Wellmay at 31 March 2009 $000 Non-current assets Property , plant and equipment (W3) Investment property Current assets (1,400 + 200 (W1)) Total assets Equity and liability Ordinary share capital Equity in loan Retained earnings Revaluation reserve Non-current liabilities 8% convertible loan note (W4) Deferred tax (W5) Current liabilities (820 + 550 (W1) – 75 (W2) Total equity and liabilities

110

Downloaded by isavic Alsina ([email protected])

4,390 375 222 4,765 1,600 222 6,365 222

2

1,500 40 2,212 540 222 4,292

1/2

568 210 1,295 222 6,365 222

1

1

1 1/2 1/2

2 1 2

lOMoARcPSD|4664814

LSB_F7_Rev Kit:297mm x 210mm

28/10/09

13:37

Page 111

ANSWERS Workings (W1) Sale and repurchase This transaction is a financing arrangement in substance.The $500,000 advance to Wellmay is a loan secured on the maturing goods. No sale should be recorded and instead: •

A loan (current) of $500,000 is recorded at 1 April 2008



In the year to 31 March 2009 interest of $50,000 ($500,000 x 10%) is charged to profits and increases the loan capital outstanding to $550,000

To reverse the sale already recorded: •

Deduct $500,000 from revenue



Deduct $200,000 from cost of sales



Add $200,000 to closing inventory

(W2) Contingency provision This provision does not meet the recognition criteria of IAS 37 and must therefore be reversed by deducting $75,000 from current liabilities and cost of sales. (W3) Properties Owner-occupied property $000 (i) revaluation Valuation at 31 March 2008 Depreciation

Revaluation gain ( ) Valuation at 31 March 2009 (ii) depreciation adjustment $40,000 x 20% – deduct from cost of sales – add to operating expenses

1,200 (40) 222 222 1,160 190 222 1,350 $000 8,000

Investment property Valuation at 31 March 2008 Fall in value Valuation at 31 March 2009

$000 400 (25) 222 222 375

111

Downloaded by isavic Alsina ([email protected])

lOMoARcPSD|4664814

LSB_F7_Rev Kit:297mm x 210mm

28/10/09

13:37

Page 112

FINANCIAL REPORTING (INTERNATIONAL) (W4) Convertible loan note $000 600 $000 43.6 39.8 36.0 440.6 222

Proceeds PV of equivalent redeemable loan note: 09: (600,000 x 8%) x 0.91 10: (600,000 x 8%) x 0.83 11: (600,000 x 8%) x 0.75 12: [600,000 + (600,000 x 8%)] x 0.68 Debt element of loan notes

(560) 222 40

Equity element of loan notes Carrying value of liability b/f $ 560,000

y/e 31.3.09

Int at 10% $ 56,000

Paid $ (48,000)

c/f $ 568,000

(W5) Deferred tax

Deferred tax provision required (600,000 x 35%) Provision b/f Charge to IS

$000 210 (180) 222 30

(W6) Bonus issue $000 Number of shares in issue 1,200,000/0.50 Bonus shares 2,400,000/4 Dr Retained earnings Cr Share capital

2,400,000 600,000 600,000 x 50c

300 300

16. Peterlee (a)

Purpose and status of the Framework Purpose The Framework provides the underlying concepts and principles on which accounting standards are based.To this end it includes, for example, explanations of the quantitative characteristics of financial statements (relevance, reliability, understandability and comparability) and definitions of an asset and liability.

1

The purpose of the framework is to: •

Assist in the development of future accounting standards



Provide principles-based guidance where an accounting standard does not deal with a transaction



Assist auditors in forming their opinion as to whether financial statements conform to accounting standards



Assist users in the interpretation of financial statements



Provide a basis for reducing the number of alternative accounting treatments permitted by standards



Assist national standard-setters in the development of national accounting standards.

112

Downloaded by isavic Alsina ([email protected])

1 per point to max of 3

lOMoARcPSD|4664814

LSB_F7_Rev Kit:297mm x 210mm

28/10/09

13:37

Page 113

ANSWERS Status The Framework is not an accounting standard, nor do its contents override any accounting standard. Where there is a conflict between an accounting standard and the Framework, the accounting standard prevails. (b)

1

Assets and liabilities: definitions and recognition criteria An asset is defined as a resource controlled by an entity as a result of past events and from which future economic benefits are expected to flow to the entity.

1

A liability is defined as a present obligation of the entity arising from past events, the settlement of which is expected to result in an outflow from the entity of resources embodying economic benefits.

1

Assets The application of the definition of an asset ensures that all quantifiable items which provide a value or economic benefits to an entity are included in the statement of financial position, but items which will not provide economic benefits are not.

1

The definition does not require assets to be owned, but rather controlled by an entity. For this reason, assets held under a finance lease are reported as assets, since they provide clear future economic benefits.

1 for discussion of control

The definition requires that economic benefits are expected to flow to the entity.Thus, for example, a contingent or uncertain asset is not included in the statement of financial position unless an economic inflow is virtually certain.

1 for discussion of economic benefits 1 for discussion of past events

The requirement for an asset to arise from a past event precludes the inclusion of future expected assets such as receivables from future sales in the statement of financial position, and so reduces the potential for overstatement of assets and manipulation of accounts. Liabilities Again, the definition of a liability ensures that all known quantifiable liabilities are included within the statement of financial position, however liabilities cannot be ‘created’ in order to manipulate the accounts. A liability is a present obligation as the result of a past event. In other words, an event has already occurred which gives rise to an obligation to transfer payment.Thus future expected liabilities are not included in the financial statements. The definition requires an expected outflow of economic resources. In the case of provisions accounting, expected is defined as ‘more likely than not’. Thus provisions without purpose are prohibited and so can not be used to manipulate profits, as was the case historically with ‘big bath accounting’. In conclusion, the definitions of both assets and liabilities ensure that the statement of financial position provides reliable and complete data and so is useful to the users of the financial statements.

1 1 for discussion of present obligation and past event 1 for discussion of expected outflow 1 Max 8 marks for part (b)

17. Derringdo (a)

Recognition of income

Marks

The Framework defines income as revenue and gains and states that it should be recognised in the income statement when an increase in an asset or decrease in a liability that can be measured reliably has arisen.

1

Thus, the framework includes all gains, whether realised or unrealised within its definition of income. The revision to IAS 1 in 2007 applied this element of the framework by extending the income statement to include other comprehensive income, being unrealised income e.g. a property revaluation.

1

113

Downloaded by isavic Alsina ([email protected])

lOMoARcPSD|4664814

LSB_F7_Rev Kit:297mm x 210mm

28/10/09

13:37

Page 114

FINANCIAL REPORTING (INTERNATIONAL) IAS 18 Revenue on the other hand concentrates only on realised amounts, stating that revenue includes only economic benefits received and receivable. It requires that revenue is recognised in profit or loss.

1

This traditional approach to recording only realised amounts as income in profit or loss is in conflict with certain newer accounting standards. For example:

(b)



IAS 39 requires that changes in the fair value of financial instruments categorised as fair value through profit and loss are recognised in profit or loss.

1



IAS 49 requires that changes in the fair value of investment properties are recognised in profit or loss.

1

IAS 18 Revenue applies the concept of substance over form by requiring that revenue is recorded when the significant risks and rewards of ownership of goods have been transferred to the purchaser.

1

An application of this requirement to a sale and repurchase agreement means that no revenue is recognised. In substance such an agreement is a provision of finance and the sales proceeds are recognised as a loan.

1

Gungho Extract of income statement for Derringdo for y/e 31 March 2009 $000 Revenue Grade A (W1) Grade B (W2)

4,600 11,400 222 16,000 (8,550) 222 7,450

Cost of sales (W2) Gross profit Workings (W1) Grade A

Derringdo acts as an agent for the sale of Grade A goods and therefore should record only its sales commission as revenue.

18,400 x 100/50 38,800 x 12.5%

$000 2,400 18,000 (2,000) 222 18,400 36,800 4,600

8,550 x 100/75

$000 1,000 8,800 (1,250) 222 8,550 11,400

Opening inventory From Gungho Closing inventory Cost of goods sold Selling price Commission (W2) Grade B

Opening inventory From Gungho Closing inventory Cost of sales Revenue

114

Downloaded by isavic Alsina ([email protected])

Max 6 marks for part (a)

lOMoARcPSD|4664814

LSB_F7_Rev Kit:297mm x 210mm

28/10/09

13:37

Page 115

ANSWERS (c)

(i) Government grant In line with the requirements of IAS 20 Accounting for Government Grants and Disclosure of Government Assistance, Derringdo has treated the $240,000 ($800,000 x 30%) grant received as a deferred credit, in other words a liability, in the statement of financial position.

1

According to the Framework, a liability is a present obligation arising from a past event, the settlement of which is expected to result in an outflow of economic benefits.

1

The grant does not meet this definition in the following respects: •

The grant will only result in an outflow of economic benefits if the asset for which it was awarded is sold within 4 years of its purchase



By the year end the asset has not been sold and so there is no past event or present obligation

1

This is an example of an accounting standard conflicting with the Framework. In these instances, the requirements of the standard override those of the Framework. (ii)Extracts from Derringdo’s financial statements for the year ended 31 March 2009 Applying company policy Income statement

Depreciation charge ((800 x 85%)/10 years x 6/12m) Release of grant (240,000/10 years x 6/12m)

$000 (34) 12

1 1

Statement of financial position Property, plant and equipment (800 – 34) Current liabilities Deferred income (240,000/10) Non-current liabilities Deferred income (240 – 12 released – 12 current)

$000 766

1

24

1

204

1

$000 (34) 240

1

In compliance with Framework Income statement

Depreciation charge ( as before) Grant income Statement of financial position

Property, plant and equipment (as before)

$000 766

Tutorial note As the grant does not meet the definition of a liability, the Framework would require that it is recognised as income when received.

115

Downloaded by isavic Alsina ([email protected])

lOMoARcPSD|4664814

LSB_F7_Rev Kit:297mm x 210mm

28/10/09

13:37

Page 116

FINANCIAL REPORTING (INTERNATIONAL) (d)

Sale of carpets A change in accounting policy occurs if there has been a change in: •

Recognition (for example an item previously recognised as an asset is now recognised as an expense)



Presentation, or



Measurement basis.

In this case Derringdo has changed when revenue is recorded. It has not, however, changed the recognition, presentation or measurement basis of revenue. Therefore this does not constitute a change in accounting policy, and the change should be accounted for prospectively i.e. from the date of change.

1

1

1

Income statement for Derringdo for year ended 31 March 2009 $000 Revenue Sales before 1 April 2008 Sales after 1 April 2008

1,200 23,000 222 24,200

1 1

Tutorial note Sales before the change on 1 April 2008 should be recorded when the carpets are fitted.Therefore all carpets fitted within the first 14 days of the financial year form part of the year’s revenue. Sales after the change on 1 April 2008 should be recorded at the date of order and payment, regardless of when they are fitted. Therefore the carpets fitted in the first 14 days of April 2009 do form part of the revenue for the year ended 31 March 09, as they are included in sales made in retail outlets in this period.

18. Broadoak (a)

Measurement of non-current assets (i) IAS 16 requires that property, plant and equipment is measured initially at cost.

1

Cost includes: •

Purchase price (after trade discounts)



Import duties



Direct costs in bringing the asset to the location and condition necessary for normal operation, such as: – – – – –

Costs of site preparation Delivery costs Installation costs Testing costs Professional fees

In the case of self-constructed assets, direct costs will also include labour costs. •

An initial estimate of future dismantling costs where there is an obligation to dismantle the asset after use.

116

Downloaded by isavic Alsina ([email protected])

1/2 per example to max of 2

lOMoARcPSD|4664814

LSB_F7_Rev Kit:297mm x 210mm

28/10/09

13:37

Page 117

ANSWERS The following costs may not be capitalised as part of property, plant and equipment: •

A proportion of administration costs



Abnormal costs such as those arising from planning error or strike action

(ii) Subsequent expenditure on property, plant and equipment may only be capitalised where it meets the definition of an asset. In other words the expenditure enhances the economic benefits expected to flow from the asset.This may be the result of: •

An extension of useful life



Increased quality of output



Reduced operating costs

IAS 16 expressly states that subsequent expenditure on day to day servicing and repairs and maintenance does not meet this definition and so must be expensed to profit or loss. An example of subsequent expenditure which does meet the definition of an asset and may be capitalised is an extension to a property which enhances future economic benefits by providing increased operational capacity.

1

1

1

1

1

IAS 16 also considers complex assets and major inspections: •

Complex assets are those which are comprised of a number of different parts, each depreciated separately. Where one of the parts reaches the end of its useful life and is replaced, the replacement cost may be capitalised.



Certain assets, such as an aircraft, require major inspections periodically. IAS 16 allows the cost of these to be capitalised and depreciated to the next inspection.

1

1 Max 3 for part (a) (ii)

(b)

Revaluation surpluses and deficits IAS 16 permits either the cost model or the revaluation model to be applied to non-current assets. Under the revaluation model, assets whose fair value can be measured reliably are carried at their fair value at the date of revaluation less any subsequent depreciation and impairment losses.

1

Where the revaluation model is applied: •

It must be applied consistently to an entire class of property, plant and equipment;

1



Revaluations should be made sufficiently regularly such that the carrying amount of an asset does not differ materially from fair value.

1

Determination of the fair value of assets is normally undertaken by a professionally qualified valuer.

1

Accounting for a revaluation surplus A revaluation surplus arises where the fair value of an asset exceeds its carrying amount. Its treatment depends on whether the surplus reverses a previous revaluation deficit: •

Where a revaluation surplus does not reverse a previous deficit, it is unrealised and therefore is not recognised in profit or loss. Rather, it is recognised as other comprehensive income in the statement of comprehensive income and accumulated in a revaluation surplus within equity.



An increase which reverses a previous revaluation deficit recognised in profit or loss is itself recognised in profit or loss.

1

1

Accounting for a revaluation deficit A revaluation deficit arises where the fair value of an asset is less than its carrying amount.Again, the treatment depends on whether the deficit reverses a previous revaluation surplus: •

A deficit which does not reverse a previous revaluation surplus is recognised in profit or loss.

1

117

Downloaded by isavic Alsina ([email protected])

lOMoARcPSD|4664814

LSB_F7_Rev Kit:297mm x 210mm

28/10/09

13:37

Page 118

FINANCIAL REPORTING (INTERNATIONAL) •

A deficit which reverses a previous revaluation surplus recognised as other comprehensive income and accumulated in a revaluation surplus in equity is itself recognised in the same way.That is, the deficit is presented as an expense within other comprehensive income and debited to the revaluation surplus.

1

The fair value of revalued assets is depreciated over the asset’s remaining useful life.Where this results in an increased depreciation charge compared to that based on historical cost, the extra amount of depreciation is transferred annually from the revaluation reserve to retained earnings.

1

When a revalued asset is disposed of, profit recognised in the income statement is calculated as the difference between sales proceeds and carrying value at the date of disposal. Any surplus in the revaluation reserve relating to the asset is transferred to retained earnings by way of a reserves transfer.

1

1

1 Max 8 for part (b)

(c)

(i) Measurement of plant $ 240,000 (30,000) 222 210,000 2,750 12,500

List price Trade discount (12.5% x 240,000) Shipping and handling costs Estimated pre-production testing Site preparation costs: Cabling (14,000 – 6,000 abnormal costs) Concrete reinforcement Own labour costs Dismantling and restoration costs (15,000 + 3,000)

8,000 4,500 7,500 18,000 222 263,250

1 1/2 1/2

1 1/2 1/2

1

Tutorial note Early settlement discounts are not taken into account when determining the cost of an asset – only trade discounts. (ii)Revaluation of property lease Statement of financial position of Broadoak at 30 September

Property, plant and equipment (W1) Revaluation surplus (W2)

2008 $ 231,000 61,000

Statement of comprehensive income of Broadoak for the year ended 30 September 2008 $ Depreciation (W1) 20,000 Revaluation deficit (35,000 – 10 (W2)) – Other comprehensive income Revaluation surplus (W2) 11,000

118

Downloaded by isavic Alsina ([email protected])

2009 $ 175,000 50,000

2009 $ 21,000 25,000 (10,000)

1 1

1 1

1

lOMoARcPSD|4664814

LSB_F7_Rev Kit:297mm x 210mm

28/10/09

13:37

Page 119

ANSWERS (W1) $ 240,000 (20,000) 222 220,000 11,000 222 231,000 (21,000) 222 210,000 (35,000) 222 175,000

Cost (1.10.07) Amortisation (240,000/12 years) CV (30.9.08) Revaluation surplus ( ) Valuation (30.9.08) Amortisation (231,000/11 years) CV (30.9.09) Revaluation deficit ( ) Valuation (30.9.09) (W2) Revaluation reserve

Brought forward Surplus in 2008 (W1) At 30 Sept 2008 Excess depreciation transfer (21,000 – 20,000) Revaluation deficit

Old $ 50,000 – 222 50,000 – 222 50,000

New $ – 11,000 222 11,000 (1,000) (10,000) 222 –

Total $ 50,000 11,000 222 61,000 (1,000) (10,000) 222 50,000

Remaining 2009 revaluation deficit of $25,000 is charged to profits.

19. Wilderness (a)

Impairment losses (i) An impairment loss is defined by IAS 36 as the amount by which the carrying amount of an asset exceeds its recoverable amount. The recoverable amount of an asset is the higher of its fair value less costs to sell and its value in use, in other words its value to the business. Fair value less costs to sell is the amount which could be obtained from the sale of an asset in an arm’s length transaction between willing parties less the incidental costs of disposal. Value in use is the present value of the future cash flows expected to be derived from an asset, including those on its ultimate disposal. This is a subjective measure involving estimates of future cash flows and appropriate discount rates. An impairment test involves assessing the recoverable amount of an asset and comparing it to carrying value.Where this can not be assessed for an individual asset, then the cash-generating unit to which an asset belongs is tested for impairment. An impairment test is required for all assets or cash-generating units when there is an indication of impairment at the reporting date.

1

1

1

1

1

1

In addition IAS 36 requires certain assets to be tested for impairment annually.These are: •

Goodwill acquired in a business combination



Intangible assets with an indefinite useful life



Intangible assets which are not yet available for use.

1

119

Downloaded by isavic Alsina ([email protected])

lOMoARcPSD|4664814

LSB_F7_Rev Kit:297mm x 210mm

28/10/09

13:37

Page 120

FINANCIAL REPORTING (INTERNATIONAL) (ii)An impaired asset or cash-generating unit must be written down to its recoverable amount, normally by crediting accumulated depreciation .The corresponding debit entry will depend on whether the asset has been revalued upwards in the past. •

Impairment losses on non-revalued assets are recognised in profit or loss



Impairment losses on revalued assets are recognised:

1

1

1. directly against any revaluation surplus to the extent that it relates to the asset which is impaired, and then

1

2. in profit or loss. Where a cash-generating unit is written down to its recoverable amount, assets are written down in the following order: 1. goodwill

1

2. other assets on a pro-rata basis No asset is written down below its recoverable amount. Future depreciation charges are based on the impaired value of the asset spread over its remaining useful life. (b)

Wilderness impairments (i) Statement of financial position of Wilderness as at 30 September 2009 $ 112,500

Plant (W)

Income statement for Wilderness for the year ended 30 September 2009 $ Depreciation 1 Oct 08 – 31 March 09 (W) 1 April 09 – 30 Sept 09 (W) Impairment

40,000 37,500 50,000

Working Cost of plant Depreciation to start of year CV at start of year Depreciation to accident (640,000 x 12.5% x 6/12m) CV at 1 April 09 Impairment loss ( ) Recoverable amount higher of: Value in use 150,000 Net realisable value 20,000 Depreciation for second half of year (150,000 /4) CV at 30 Sept 09

$ 640,000 (400,000) 222 240,000 (40,000) 222 200,000 (50,000) 222

150,000 (37,500) 222 112,500

Notes •

The asset is depreciated on the old basis up until the date of the accident



On the date of the accident there is an obvious impairment, and recoverable amount is assessed

120

Downloaded by isavic Alsina ([email protected])

1

lOMoARcPSD|4664814

LSB_F7_Rev Kit:297mm x 210mm

28/10/09

13:37

Page 121

ANSWERS •

The trade in value is irrelevant since Wilderness do not intend to take advantage of the trade in offer.Therefore the fair value less costs to sell is the lower amount of $20,000



Value in use is higher than fair value less costs to sell and therefore this is recoverable amount



The difference between the carrying value on the date of the accident and value in use is the impairment loss, charged to profits



The impaired value is depreciated over the remaining useful life of two years.

(ii) Before considering an impairment of Mossel as a cash generating unit, impairment of the individual assets should be considered: Carrying value $000 7,000 12,000 8,000 5,000 222

Brand Land Plant Inventories

Impairment $000 (7,000) – – – 222

Impaired value $000 – 12,000 8,000 5,000 222 25,000

1

Brand The brand name Quencher is no longer used and so will not lead to economic benefits.Therefore it does not meet the definition of an asset and must be written off in full.

1

The new brand name is internally generated and therefore may not be capitalised according to IAS 38.

1

Plant The directors believe that it is necessary to spend money on upgrading the bottling plant, however there is no indication that the plant is impaired on an individual basis at this time. Furthermore the future expenditure is not provided for on the basis that there is no obligation.

1

Inventories Inventories should be held at the lower of cost and net realisable value. The NRV of the bottles of Quencher is $2.75 million ((2m x 150% )- 250,000).This is higher than cost of $2million and therefore no adjustment is needed.

1

Subsequent to the assessment of individual assets, the carrying value of the CGU is $25million. The value in use of the CGU is $20 million, indicating an impairment loss of $5million. This loss is prorated between the land and plant as follows:

Land Plant

CV 12,000 8,000

Impairment 12/20 x $5m = 3,000 8/20 x $5m = 2,000

Impaired value 9,000 6,000

Note that no impairment loss is allocated to the inventories as IAS 36 states that an impairment loss shall not reduce the carrying value of an asset below recoverable amount.The recoverable amount of the inventories is their NRV, and they are already held at cost which is lower than this.

1 1

1

121

Downloaded by isavic Alsina ([email protected])

lOMoARcPSD|4664814

LSB_F7_Rev Kit:297mm x 210mm

28/10/09

13:37

Page 122

FINANCIAL REPORTING (INTERNATIONAL)

20. Elite Leisure Statement of financial position for Elite as at 30 September 2009 $m Cruise ship (W2) Propulsion system Cabins Fabric

126 88 192 222 406

1 1 1

Income statement for Elite for the year ended 30 September 2009 $m Depreciation (W2) Propulsion system Cabins Fabric Write off of old propulsion system (W2) Repainting of fabric

14 22 12 25 20

1 1 1 1 1

(W1) Carrying value of components at 30 Sept 08

Fabric Cabins Propulsion system

Cost $m 300 150 100

Acc’d depreciation $m 8/25 X 300 = 96 8/12 x 150 = 100 30/40 x 100 = 75

CV $m 204 50 25

(W2) Carrying value of components at 30 Sept 2009

Propulsion system

CV at 30 Sept 08 Write off Cost new system Depreciation 5/50 x 140

Cabin upgrade

Fabric

CV at 30 Sept 08 Upgrade

SFP $m 25 (25) 140 (14) 126 22 50 60 22 110

Depreciation 1/5 x 110

(22) 22 88

CV at 30 Sept 08 Painting Depreciation 300/25yrs

204 (12) 22 192

IS $m 25

14

22

20 12

Notes The old propulsion system is written off and therefore the carrying value of $25m must be charged to profits. The new propulsion system is capitalised at cost and depreciated over 50,000 hours. 122

Downloaded by isavic Alsina ([email protected])

4 marks for explanator y notes.

lOMoARcPSD|4664814

LSB_F7_Rev Kit:297mm x 210mm

28/10/09

13:37

Page 123

ANSWERS The Cabin upgrade is capitalised as it enhances the economic benefits of the cabins by extending their useful life. The upgrade plus the carrying value of the cabins prior to the upgrade is depreciated over the new remaining useful life. The repainting of the fabric does not enhance the economic benefits of the ship and is therefore charged to profits. The fabric continues to be depreciated over 25 years as before.

21. Linnet (a)

Construction contracts Accounting for construction contracts in accordance with IAS 11 is dependent upon whether the outcome of a contract can be measured reliably. A contract can be measured reliably when: •

it is probable that the economic benefits associated with the contract will flow to the entity



the contract costs can be identified and measured reliably.

1

In the case of a fixed price contract the following must also be measured reliably:

(b)



total contract revenue



contract costs to completion



stage of contract completion at reporting date.

Where a contract can be measured reliably, revenue is recognised by reference to the stage of completion.

1

This is an application of the concept of accruals.

1

Where a contract can not be measured reliably, revenue is recognised only to the extent that contract costs incurred are recoverable.

1

This is an application of the concept of prudence.

1

Income statement for Linnet for the year ended 31 March 2009 $m 70 (81) 222 (11)

Revenue (W3) Cost of sales (W3) Gross loss (W3)

1 2 1

Statement of financial position of Linnet at 31 March 2009 $m Current assets Amounts recoverable on construction contracts (W4)

59

See W4

123

Downloaded by isavic Alsina ([email protected])

lOMoARcPSD|4664814

LSB_F7_Rev Kit:297mm x 210mm

28/10/09

13:37

Page 124

FINANCIAL REPORTING (INTERNATIONAL) Workings The outcome of this contract can be estimated reliably and therefore income statement amounts are recognised by reference to the stage of completion: (W1) Expected profit $m 300 (195) (45) 222 60

Contract price Costs to date Estimated costs to complete

Note that rectification costs are not taken into account at this stage (although they are contract costs).This is because they are expensed in the period in which they are incurred rather than spread across the contract. (W2) Stage of completion Sales value to date 2222222 Contract price

=

220* 22 300

=

73.3% $m

* Sales value to date: Work certified to 29 Feb 09 (100/90 x 180) Work complete in March 09

200 20 222 220

(W3) Income statement

Revenue (work certified) Cost of sales ( ) Profit (73.3% x 60 (W1))

Cumulative $m 220 176 222 44

Taken previously $m (150) (112) 222 (38)

Current year $m 70 (64) 222 6

Rectification costs must be recognised in the year incurred and therefore the current year figures calculated are amended as follows: $m 70 (81) 222 (11)

Revenue Cost of sales (64 + 17) Loss (W4) Gross amount due to or from customers

$000 212 44 (180) (17) 222 59

Costs to date Profit to date (W3) Less: progress billings Less: rectification costs Due from customers

124

Downloaded by isavic Alsina ([email protected])

1 1 1 1

lOMoARcPSD|4664814

LSB_F7_Rev Kit:297mm x 210mm

28/10/09

13:37

Page 125

ANSWERS

22. Torrent Income statement for Torrent for the year ended 31 March 2009 $m 14.8 (14.5) 222 0.3

Revenue (W3) Cost of sales (W3) Gross profit (W3)

See w3 See w3 See w3

Statement of financial position of Torrent at 31 March 2009 $m Current assets Amounts due from customers (W4) Amounts due to customers (W4) Receivables (14 – 12.6) + (2 – 1.8)

5.8 (1.5) 1.6

See w4 See w4 1

Workings (W1) Estimated profit on contracts

Contract price Costs to date (excluding rectification costs) Costs to completion Expected profit/loss

Alfa 08 $m 20

Alfa 09 $m 20

(5) (10) 222 5

(11.5) (3.5) 222 5

Beta $m 6 (2) (5.5) 222 (1.5)

Ceta $m 12 (4) (6) 222 2

(W2) Stage of completion Alfa 08 Alfa 09 Sales basis: work certified 222222 Contract price

Profit/loss to be recognised

Beta Ceta costs basis: costs to date 222222 total costs

5.4 x 100/90 22222 20 30%

12.6 x 100/90 22222 20 70%

n/r as loss making

4 2 10 40%

30% x 5 = 1.5

70% x 5 = 3.5 (cumulative)

Full loss (1.5)

40% x 2 = 0.8

125

Downloaded by isavic Alsina ([email protected])

lOMoARcPSD|4664814

LSB_F7_Rev Kit:297mm x 210mm

28/10/09

13:37

Page 126

FINANCIAL REPORTING (INTERNATIONAL) (W3) Income statement figures Alfa and Beta are assessed on the sales basis and therefore revenue is equal to work certified; cost of sales is a balancing figure. Ceta is assessed on the costs basis and therefore cost of sales is equal to costs incurred to date; revenue is a balancing figure

Revenue (Work certified/ ) Cost of sales ( /costs incurred)

Alfa cumulative $m

Alfa 08 $m

Alfa 09 $m

Beta $m

Ceta $m

Total $m

14

6

8

2

4.8

14.8

1 1/2 + 1 + 1

(13.5) 22 1.3

1 + 1/2 + 1/2 1/2 + 1 + 1/2

(10.5) 22 3.5

Profit / loss (W2)

(4.5) 22 1.5

(6) 22 2

(3.5) 22 (1.5)

(4) 22 0.8

Amendment for rectification costs in Alfa

Revenue Cost of sales

Alfa 09 $m 8 (6) (1) 222 1

As above Rectification costs

Profit

Total $m 14.8 (13.5) (1) 222 0.3

1

(W4) Amounts due to / from customers

Costs to date Profit / loss to date Rectification costs Progress billings (work certified) Due from customers Due to customers

Alfa $m 12.5 3.5 (1)

Beta $m 2 (1.5) –

Ceta $m 4 0.8 –

(14) 222 1

(2) 222

– 222 4.8

1.5

Total $m 1 1 1

1

5.8 1.5

23. Bowtock Income statement of Bowtock for the year ended 30 September 2009

Depreciation (52,000/5 years) Finance costs (800 + 1,872) Statement of financial position for Bowtock as at 30 September 2009

PPE (52 – (10.4 x 9/12) – 10.4) Current liabilities Finance lease obligation (W) Non-current liabilities Finance lease obligation (W)

126

Downloaded by isavic Alsina ([email protected])

$ 10,400 2,672

1 1

$ 33,800

1

11,376

1

21,696

1

lOMoARcPSD|4664814

LSB_F7_Rev Kit:297mm x 210mm

28/10/09

13:37

Page 127

ANSWERS Working b/f at 1 Oct – 42,400 33,072

y/e 30 Sept 08 y/e 30 Sept 09 y/e 30 Sept 10

Int x 3/12 – 800 624

c/f 52,000 43,200 33,696

Repay’t on 1 Jan (12,000) (12,000) (12,000)

c/f at 1 Jan 40,000 31,200 21,696

Int x 9/12 2,400 1,872

c/f at 30 Sept 42,400 33,072

Liability due > 1 year is $21,696 Liability due < 1 year is $11,376 (33,072 – 21,696) Interest Year 1 of lease Year 2 of lease

40,000 x 8% 31,200 x 8%

Total 3,200 2,496

9m 2,400 1,872

3m 800 624

Note Interest is charged on the capital liability outstanding after each repayment is made. Due to the fact that the lease repayments occur 3m into the financial year, each year’s interest falls 9m within one financial year and 3 m within the next.

24. Triangle Marks

(i) New processing plant The initial measurement of the processing plant on 1 April 2008 should include both the construction cost and the present value of future decontamination costs: $000 15,000 5,000 222 20,000

Construction cost PV of decontamination

At this stage a provision of $5m for the decontamination costs should also be made, as there is a legal obligation to decontaminate the plant.The other recognition criteria for a provision are met as the future decontamination costs are probable and can be reliably measured. In the year ended 31 March 2009, the total cost of the plant should be depreciated by $2million ($20million/10 years). Therefore at the year end the plant will have a carrying value of $18million ($20million – $2 million). One year of discounting in the provision should also be unwound, resulting in a finance cost of $400,000 ($5m x 8%). This amount will also increase the carrying value of the provision to $5.4million. The treatment adopted by the management of Triangle is therefore wrong in that the $5million provision is not made in full and not treated as part of the cost of the asset.

127

Downloaded by isavic Alsina ([email protected])

lOMoARcPSD|4664814

LSB_F7_Rev Kit:297mm x 210mm

28/10/09

13:37

Page 128

FINANCIAL REPORTING (INTERNATIONAL) Marking guide Plant cost to include decontamination costs Initial cost $20m Provision for $5 to be made Meets provision recognition criteria Depreciation charge $2m Year end carrying value of plant $18m Discount to be unwound Finance cost of $400,000 Provision CV of $5.4m

1 1 1 1 1 1 1 1 1

Max 8

(ii)Fraud The fraud was discovered on 15 May 2009.This is after the 09 reporting date. Assuming that the accounts have not been authorised for issue by this date, the fraud should be treated as an event after the reporting period. $210,000 of the fraud related to year ended 31 March 2009 and its discovery provides additional evidence of conditions at the reporting date.Therefore this is an adjusting event and the loss should be charged to the income statement in the year. The remaining $30,000 of the loss has arisen after the reporting date and is therefore a nonadjusting event. The financial statements should not be amended for this amount, however it may be disclosed if considered material. Marking guide Event after the reporting period $210,000 is adjusting event as provides extra evidence $210,000 charged to income statement in X5 $30,000 is non-adjusting event Disclose if material

1 1 1 1 1

Max 5

(iii) Insurance Triangle has a contingent asset, in other words, an uncertain item of income. IAS 37 states that contingent assets are recognised only where their receipt is virtually certain. This is not the case here, as the insurance company has dismissed the claim made by Triangle, and as yet there is no legal opinion on the issue. If the contingent asset is assessed as probable of receipt, it should be disclosed in Triangle’s accounts. Otherwise it should be ignored. Marking guide Claim is a contingent asset Recognise where virtually certain Not virtually certain in this case Disclose if probable Ignore otherwise

128

Downloaded by isavic Alsina ([email protected])

1 1 1 1 1

Max 4

lOMoARcPSD|4664814

LSB_F7_Rev Kit:297mm x 210mm

28/10/09

13:37

Page 129

ANSWERS (iv) Sale and repurchase This transaction is an example of a sale and repurchase. Commercial substance prevails over legal form in the recording of such a transaction. If the substance of the transaction is a true sale, then it should be recorded as such. A true sale has occurred where the risks and rewards of ownership of the goods are transferred to the purchaser. Indicators that this is not the case here and this is not a true sale are: •

Triangle retains the goods on its premises



The sale was not at market value



Triangle may repurchase the goods at any time within 4 years at a less than market value price



Interest accrues on the cost of repurchase from the date of sale



Triangle pays the storage costs for the 4 years during which it has the option to repurchase (to be reimbursed by Factorall if the option is not taken)

Therefore this transaction should be accounted for as a financing arrangement as follows: The proceeds of $5million should be recorded as a loan, accruing interest at 10% per annum. The inventory should continue to be recorded as an asset in the accounts of Triangle. The costs of storage should form part of the cost of this inventory in accordance with IAS 2. Therefore the income statement for the year ended 31 March 2009 will include finance costs of $500,000 ($5m x 10%). The statement of financial position at that date will include inventory of $3.3 million ($3million + $300,000 storage costs) and a loan of $5.5million ($5million + $500,000 interest). Marking guide Commercial substance over legal form Sale recorded where transfer risks and rewards Application of risks and rewards to scenario Conclude this is financing arrangement Loan in SFP $5.5m Inventory in SFP $3.3m Finance costs $500,000

1 1 1 per point max 4 1 1 1 1

Max 8

25. Atkins (i) Consignment inventory

Marks

Atkins buys cars from a manufacturer to sell on to the public.The manufacturer transfers such cars to Atkins’ showroom to facilitate such sales, however Atkins only pays the manufacturer on the date that a car is sold to a customer (or after 6 months if sooner). The issue here is when Atkins should record the purchase of the cars (and the manufacturer should record the sale), and so whose inventory the cars are whilst they are in the showroom. Arguably the purchase (sale) could be recorded when the cars are physically transferred or when Atkins pays the manufacturer.

1

IAS 18 requires that a sale is recorded when the risks and rewards of ownership are transferred. Therefore an assessment of which entity has these risks and rewards whilst the cars are in Atkins’ showroom is required.

1

129

Downloaded by isavic Alsina ([email protected])

lOMoARcPSD|4664814

LSB_F7_Rev Kit:297mm x 210mm

28/10/09

13:37

Page 130

FINANCIAL REPORTING (INTERNATIONAL) If Atkins bears the risks and rewards then it should record the purchase of the cars and an associated liability to pay the manufacturer when the cars are delivered. If the manufacturer bears the risks and rewards then Atkins should not record the purchase until it has secured the onward sale (or after 6m if earlier). Assessment of risks and rewards •

The price paid by Atkins is fixed at the date of delivery meaning that Atkins bears the risk of a subsequent fall in the retail price, but also benefits from any subsequent increase in the retail price.



Atkins pays a ‘display charge’ to the manufacturer until the cars are paid for.This is, in effect, interest on the payable outstanding in Atkins’ accounts. Therefore Atkins bears the risk of slow moving goods (and so higher ‘display charges’.



Atkins’ right of return has never been exercised, indicating that this may not be a true benefit. i.e.Atkins may suffer a penalty such as reduced supply if it does exercise this right.



The manufacturer can recall the cars at any time.This is a benefit retained by the manufacturer.

Conclusion

1 per point to a max of 4

1

The above assessment indicates that Atkins bears the majority of risks and rewards of ownership, and therefore should record the purchase of the cars when they are transferred to the showroom. The display charge will accrue on a monthly basis.

2222 Max 6

(ii) Development land Atkins has purchased 5 plots of development land, subsequently selling 3 to Landbank with an option to repurchase them at any time within 3 years. The issue here is whether the substance of this transaction is a true sale, or a loan secured on the 3 plots of land.

1

An assessment of the facts indicates that the commercial substance is a loan made by Landbank to Atkins: •

The sale is for 25% less than market value indicating that it is not a true sale



Atkins can repurchase the land at any time over 3 years at the full market value as at October X2 rather than at the market value on the date of repurchase



Atkins will pay a premium on the repurchase which can be considered interest.



Landbank has a put option to require Atkins to repurchase the land after 3 years. Landbank is likely to exercise this if the market value of the land has fallen below the option price of $3.2million.

1 per point to maximum of 2

Land considered sold to Landbank Income statement of Atkins for the year ended 30 September 2009 Profit on sale of land ($2.4 m – (3/5 x $2.4m)) Statement of financial position of Atkins at 30 September 2009 Investment property (2/5 x $2m)

130

Downloaded by isavic Alsina ([email protected])

$000 1,200

800

1

1

lOMoARcPSD|4664814

LSB_F7_Rev Kit:297mm x 210mm

28/10/09

13:37

Page 131

ANSWERS Transaction considered financing arrangement Income statement of Atkins for the year ended 30 September 2009 Finance costs ($2.4m x 10%)

$000 240

Statement of financial position of Atkins at 30 September 2009 Investment property Loan (2.4m x 1.1)

2,000 2,640

2

1/2

1 1/2

26. CB (a)

Marks

Calculation of EPS and PE ratio Earnings per share Profits attributable to ordinary shareholders

$725,000

1

Weighted average number of ordinary shares in the period

1 Feb 08 – 28 Feb 08 1 March 08 – 31 Jan 09 Bonus fraction

Therefore EPS is

Share price before issue 222222222 TERP TERP 4 old shares @ $7.50 = 1 new share @ $6.50 = 2 5 $725,000 = 2222 3,694,349

PE ratio Price per share

=

Earnings per share (b)

Bonus fraction x 7.5/7.3

No 3,000,000 3,750,000

$30 $6.5 22 $36.5 19.6c

625 22 19.6c

Time X 1/12 X 11/12

No shares 256,849 3,437,500 2222 3,694,349

so TERP $36.5/5 = $7.30

= 31.88

1 1

1

2

Significance of PE ratios PE ratios provide indication of market confidence in a company. A calculated PE ratio is the multiple of earnings per share that an investor is willing to pay for each share, so a PE ratio of 10 reveals that an investor is willing to pay 10 times the current year EPS to buy a share. Therefore the investor is confident that the company will make future profits and perform well.

1 mark per point to max 4

PE ratios are of course a product of the share price in the market, and therefore it should be noted that any general market slump will affect this ratio. CB has a PE ratio of 31.88 compared to an average 28.4 for the business sector and 42.5 of its main competitor.This indicates that CB is viewed as one of the better performers within the sector, however the market has more confidence in its competitor in terms of future performance.

131

Downloaded by isavic Alsina ([email protected])

lOMoARcPSD|4664814

LSB_F7_Rev Kit:297mm x 210mm

28/10/09

13:37

Page 132

FINANCIAL REPORTING (INTERNATIONAL)

27. QRS

Marks

Accounting treatment of financial instruments Non-redeemable preferred shares IAS 32 requires that non-redeemable preferred shares are classified as equity.The nominal value of the shares issued should be recorded within share capital, and any premium within the share premium reserve.

1 1

The dividend should be reported in the statement of changes in equity. 1

Convertible bonds IAS 32 considers convertible bonds to be a hybrid instrument. It should therefore be classified part as debt and part as equity, to represent the conversion rights. The proceeds of the issue should be split between debt and equity as follows:

1

– The amount of debt is calculated as the present value of the bonds assuming there was no option to convert, and based on a market interest rate for a bond without conversion rights. 1

– Equity is the residual. The debt element is presented as a liability; the equity element is presented within reserves; the interest on the bond is all presented as interest in the income statement.

1

Split accounting for bonds:

1

DF 1/1.08 1/1.082 1/1.083 1/1.084 1/1.084

Interest year 1: 360,000 Interest year 2: 360,000 Interest year 3: 360,000 Interest year 4: 360,000 Repayment year 4: 6m Debt element Equity element therefore

PV$ 333.333 308,642 285,780 264,611 4,410,179 2222 5,602,545 397,455 2222 6,000,000

2 1

28. Harper (a)

Discontinued operations (i) Users of the financial statements are particularly interested in the future of an entity – what its profits may be, whether it is worthwhile investment and so on. The financial statements themselves are, however, historic, and therefore users must try to forecast future performance based on past events. The separate presentation of discontinued operations helps with this process by, in effect, identifying that portion of the profit or loss of an entity which will not continue in future years. Where a discontinued operation is loss making, future overall profits should increase; where a discontinued operation is profitable, future overall profits will decrease, and investors may question the strategy of divesting of a profitable operation. Thus, the disclosure of discontinued operations also helps investors to assess the stewardship of the management and how well they are running the company.

132

Downloaded by isavic Alsina ([email protected])

Marks 1 mark per point to a max 3

lOMoARcPSD|4664814

LSB_F7_Rev Kit:297mm x 210mm

28/10/09

13:37

Page 133

ANSWERS (ii) Operating profit in the absence of discontinued operations information

Operating profit (100 x 110%)

Gamma $m 110

Toga $m 110

Gamma $m

Toga $m

77 – – 22 77

99 110 – 22 209

1

Operating profit based on the information provided

Operating profit Continuing activities (70/90) x 110% Acquisitions (50 x 12/6m x 110%) Discontinued

1 1 1

(b) Earnings per share (i) Earnings per share is calculated by dividing the profit attributable to the ordinary shareholders of an entity by the weighted number of ordinary shares in issue throughout the period. An upwards trend in profits will be reflected in an upwards trend in EPS, provided that there are no new issues of shares during the period concerned.

1

1

If there is an issue of shares, the increased capital is likely to be utilised to increase profits and so there will likely be an upwards profit trend.The increased number of shares may, however mean that a conflicting downwards EPS trend is identified. From an individual shareholder perspective, EPS is a more important measure of performance, as EPS provides an indication of how much dividend per share may be paid in the future.

1

(ii) Basic EPS =

$50,m 200,000,000

Dilutive factors: Convertible loan stock Increase in profits (due to no interest): Increase in shares on conversion:

=

25c

8% x 200m x 75% = 12m $200m/$100 x 70 = 140m $12m = 8.5c 22 140m 8.5c is lower than basic EPS therefore this is a dilutive factor.

1 1

133

Downloaded by isavic Alsina ([email protected])

lOMoARcPSD|4664814

LSB_F7_Rev Kit:297mm x 210mm

28/10/09

13:37

Page 134

FINANCIAL REPORTING (INTERNATIONAL) Share options No effect on profits Increase in ‘free’ shares:

Diluted EPS

funds raised through options: Number of shares at MV: Therefore ‘free’ shares:

50m x $1.50 $75m / $2.50 50m – 30m

= $75m = 30m = 20m

Basic EPS

$50,m 22222 200,000,000

= 25c

$50,m 22222 200mm + 20m

= 23c

$50m + $12m 22222 220m + 140m

= 17c

Introduce effect of share options (most dilutive factor) Introduce effect of convertible loan stock

1

2

Therefore fully diluted EPS is 17c. (iii) Basic earnings per share is calculated based on actual profits for the year and the number of ordinary shares actually in issue throughout the year.

1 per point to a max of 4

A number of items in the statement of financial position may, however, lead to a future increase in the number of ordinary shares. An example of such an item is share options. The exercise of share options will increase the number of ordinary shares in issue and result in a lower EPS. It is therefore said to be a dilutive factor. In some cases an increase in the number of shares will also have an effect on profits. For example the conversion of loan stock into ordinary shares will increase the number of shares in issue and also increase profits through the reduction in interest payable. This may result in an overall decrease in EPS or it may result in an increase, depending on the proportionate increases in both shares and profits. If such a conversion results in decreased EPS, it is a dilutive factor; if it results in increased EPS, it is said to be anti-dilutive. Diluted EPS adjusts basic EPS for all dilutive factors. In other words, it takes into account all future potential dilutive issues of ordinary shares based on evidence in the current statement of financial position, and their impact on profits. It may therefore be considered a ‘worst case scenario’ EPS. Note, however, that diluted EPS does not take account of future profit forecasts or timescales for share issues, nor does it consider anti-dilutive factors. It is therefore not a prediction of future EPS. (c)

In many cases the tax charge reported in a company’s income statement is not equivalent to the tax rate multiplied by the profit before tax.There are a number of reasons for this: •

Tax is charged on taxable profits rather than the accounting profit before tax. Taxable profits are calculated by adjusting the accounting profit for certain expenses which are not tax allowable, for example depreciation, and income which is not taxable, for example dividends from companies in the same jurisdiction.

2



The year-end tax charge is an estimated amount, and any difference between this reported amount and the amount finally paid must be adjusted through the following year’s tax charge as an under or overprovision.

1



The tax charge in the income statement includes any movement in the provision for deferred tax.

1

The tax charge in the income statement is unlikely to be the same amount as tax paid in the statement of cash flows for the following reasons: •

For most companies, tax is paid a number of months after the year to which it relates.Therefore the amount reported as the tax charge in the income statement relates to the current year, whereas the amount reported as tax paid in the statement of cash flow for the same period is likely to be the previous year’s tax charge (with adjustment for any over or underprovision).

134

Downloaded by isavic Alsina ([email protected])

1

lOMoARcPSD|4664814

LSB_F7_Rev Kit:297mm x 210mm

28/10/09

13:37

Page 135

ANSWERS •

Deferred tax, which is included in the income statement tax charge, is not relevant to the statement of cash flows, as this is not tax currently payable.

1

29. FW Briefing paper Marks

From: AN Accountant This briefing paper analyses and interprets the effects of making a $500 million environmental provision on FW’s key ratios, taking into account the possible effects on the public perception of FW. 2008

2009 Before provision

Return on equity

24.7%

Return on net assets

17.7%

Gearing

82%

Operating profit margin

10.1%

Earnings per share

12.2c

1,670 222222 (4,954 + 5.656)/2 = 31.5% 2,080 222222 (9,016 + 10,066)/2 = 21.8% 4,410 22 5,656 = 78% 2,080 22 20,392 = 10.2% 1,002 22 6,000 = 16.7c

2009 After provision (see workings below for adjusted figures) 1,170 222222 (4,954 + 5,156)/2 = 23.1% 1,580 222222 (9,016 + 9,566)/2 = 17.0% 4,410 22 5,156 = 85.5% 1,580 22 20,392 = 7.7% 502 22 6,000 = 8.4c

1 1/2

1 1/2

1 1/2

1 1/2

1 1/2

Commentary on ratios Without exception, before considering the environmental provision, FW’s key ratios improved, in some cases significantly between 2008 and 2009. The most improvement was seen in return on equity and return on net assets. The introduction of the $500m provision into the accounts and subsequent recalculation of the key ratios, however results in a deterioration of all results compared to 2004. Whilst return on equity and return on net assets are only slightly below the 2004 level, the operating profit margin and earnings per share are significantly lower. Whilst many members of the public are unfamiliar with return on equity / net assets, earnings per share is a commonly used measure, and the fall in this from 12.2c to 8.4c is likely to have a negative impact on the public’s perception of FW. Similarly, profit margins are widely understood by most investors, and the fall from 10.1% to 7.7% will have a similar impact. Although the key ratios show deterioration, it should be remembered that FW, by creating the environmental provision, is demonstrating its commitment to ‘green’ issues.These are widely viewed as of increasing importance by the public, and therefore the company’s creation of a provision to restore damaged habitats and compensate displaced persons will show FW in a positive light.

1 for discussion of ratios without provision 2 for discussion of ratios after provision 2 for impact on public 1 1/2 for positive impact of ‘green’ actions

135

Downloaded by isavic Alsina ([email protected])

lOMoARcPSD|4664814

LSB_F7_Rev Kit:297mm x 210mm

28/10/09

13:37

Page 136

FINANCIAL REPORTING (INTERNATIONAL) Workings: adjustments

Net assets at 28.2.09 Share capital and reserves at 28.2.09 Operating profit Profit before tax Profit for the period

10,066 5,656 2,080 1,670 1,002

Provision (500) (500) (500) (500) (500)

Adjusted 9,566 5,156 1,580 1,170 502

30. LMN Consignment inventory

Marks

The transaction described is an example of consignment inventory, typical of the motor industry. In this type of transaction, goods are delivered from a manufacturer to a retailer and held on the retailer’s premises until sold.

1 for identifying transaction

Legally, the manufacturer retains title to the goods until such time as they are sold to a third party. The transaction is, however, accounted for according to the substance of the transaction: •

If the risks and rewards of ownership are deemed to have passed to the retailer at the date the manufacturer delivers the goods, then they are included as inventory in the retailer’s statement of financial position for the time that they remain on the retailer’s premises.



If the risks and rewards of ownership are retained by the manufacturer when the goods are delivered to the retailer, then the inventory remains in the manufacturer’s statement of financial position until it is sold on to a third party.

2 for explanation of accounting

The terms of LMN’s contract with IJK must be examined in order to assess which company holds the risks and rewards of ownership of the cars during the time that they are held on LMN’s premises: •

LMN is required to insure the vehicles whilst they remain on its premises.This is a risk of ownership and indicates that the inventory is an asset of LMN at delivery.



The price at which vehicles are supplied is determined at the date of delivery.Therefore IJK is unable to pass on any subsequent price changes, which indicates that the vehicles are inventory of LMN at delivery.



LMN is entitled to use any of the vehicles supplied to it for demonstration purposes and road testing.This is an example of a transferred reward and indicates that the vehicles are inventory of LMN at delivery.



A penalty is payable by LMN if the number of kilometres driven in any vehicle exceeds a certain level.The number of kilometres driven in test drives will clearly increase the longer that the vehicle goes unsold and therefore this may be viewed as an example of slow movement risk borne by LMN.This is a further indicator that the vehicles are inventory of LMN at delivery.



LMN has the right of return without penalty. It is unclear whether this right is exercised by LMN, but assuming that it is, the risk of return is borne by IJK and this is an indicator that the vehicles remain IJK’s asset.

1 per point for assessment of risks and rewards Max 5

The majority of the contract terms therefore indicate that the risks and rewards of ownership of the vehicles are transferred to LMN. The vehicles should therefore be recorded as their asset from delivery, with LMN booking a purchase and IJK booking a sale on the date of delivery.

136

Downloaded by isavic Alsina ([email protected])

2 for conclusion

lOMoARcPSD|4664814

LSB_F7_Rev Kit:297mm x 210mm

28/10/09

13:37

Page 137

ANSWERS

31. EFG (a)

(b)

Key ratios

Marks

Gross profit margin

429 x 100% 22 1,810 = 23.7%

1/2

Operating profit margin

193 x 100% 22 1,810 = 10.7%

1/2

Return on total capital employed

193 x 100% 22 (769 + 248) = 19%

1/2

Gearing

248 x 100% 22 769 = 32.5%

1/2

Key ratios after adjustment Gross profit margin Revenue

1/2

No adjustment required $000

Gross profit

Per income statement Add back extra depreciation on revaluation ($250,000/25 years) Inventory adjustment: Opening less closing inventory using AVCO valuation (197,000 – 201,000) (4,000) Opening less closing inventory using FIFO valuation (208,000 – 218,000) (10,000) 222 Therefore add back difference:

Revised gross profit margin

$000 429

1/2

10

1

6 222 445

1

1/2

445 x 100% 22 1,810 = 24.6%

Operating profit margin Revenue Operating profit

no adjustment required Per income statement Extra depreciation (see above) Inventory adjustment (see above) Add back bonus paid to skilled staff Fixed salaries to replace bonus

Revised operating profit margin

255 x 100% 22 1,810 = 14.1%

$000 193 10 6 96 (50) 222 255

1/2 1/2 1/2 1/2 1/2

1/2

137

Downloaded by isavic Alsina ([email protected])

lOMoARcPSD|4664814

LSB_F7_Rev Kit:297mm x 210mm

28/10/09

13:37

Page 138

FINANCIAL REPORTING (INTERNATIONAL) Return on total capital employed

Operating profit Capital employed

Revised ROCE

(see above) Per statement of financial position (769 + 248) Less: revaluation surplus Adjustments to retained profit Inventory b/f (208 – 197) Inventory adjustment (for year) Bonus Fixed salaries

$000 255 1,017 (200) 11 6 96 (50) 222 880

1/2 1/2

1 1/2 1/2 1/2

1/2

255 x 100% 22 880 = 29%

Tutorial note •

the preferred shares should be reclassified from equity to debt, however this does not affect overall capital employed, and therefore the adjustment is not made here.



The revaluation surplus would not exist if J applied EFG’s accounting policy.The $50,000 of the surplus transferred to retained earnings since the revaluation needs no adjustment, as this compensates for the lower retained earnings as a result of the revaluation policy.



The extra depreciation of $10,000 which forms an adjustment to operating profit is not adjusted for in capital employed as it has been transferred from the revaluation surplus to retained earnings by way of a reserves transfer (shown in the SOCE).



Retained earnings b/f must be adjusted for the difference in inventory valuation based on the AVCO and FIFO measurement methods.

Gearing

Long-term liabilities

Shareholders’ funds

Revised gearing

Per statement of financial position Reclassification of preferred shares Per statement of financial position Reclassification of preferred shares Less: revaluation surplus Adjustments to retained profit Inventory b/f (208 – 197) Inventory adjustment (for year) Bonus Fixed salaries

298 x 100% 22 582 = 51.2%

138

Downloaded by isavic Alsina ([email protected])

$000 248 50 222 298 769 (50) (200) 11 6 96 (50) 222 582

1/2 1/2

1/2 1/2 1/2

1

½

lOMoARcPSD|4664814

LSB_F7_Rev Kit:297mm x 210mm

28/10/09

13:37

Page 139

ANSWERS Advice to directors After adjustment to J’s results to bring its accounting policies into line with those of EFG, the key ratios are: 2

Gross profit margin Operating profit margin ROCE Gearing

24.6% 14.1% 29% 51.2%

The operating profit margin and ROCE exceed the acquisition criteria set by EFG, however the gross profit margin and gearing ratios fall short. The gross profit margin is only 0.4% lower than the required level of 25%, and would be easily addressed, particularly with the effect of post-acquisition synergies. Gearing, however, is more than double the acceptable level, and on this basis, the acquisition should not be pursued.

32. Breadline (a)

Marks

Subsidiary accounts Subsidiary companies are required to produce individual entity accounts for statutory purposes in most jurisdictions. These entity accounts are of limited use in the assessment of company performance largely due to related party transactions with the parent company and / or other group subsidiaries: •

The subsidiary is likely to have engaged in transactions that it would not have done as a standalone company, possibly sales to group companies or loans from group companies.



Related party transactions are often not conducted at arm’s length. Examples may include:

1

– Trading at non-commercial transfer prices which may artificially reduce the subsidiary’s company profit (in the case of a low transfer price for sales or a high transfer price for purchases) or increase them (in the case of a high transfer price for sales or a low transfer price for purchases)

1

– Loans which are made interest free, or with interest rates lower than those commercially available, so impacting profits.

1

– The parent company may pass back operating expenses to the subsidiary, so lowering profits.

1

– The parent company may provide management services free of charge to the subsidiary, so artificially inflating its profits.

1

– Certain functions are likely to be pooled at a group level and recharged to the subsidiary on a basis decided by management of the parent.

1 (max 5)

139

Downloaded by isavic Alsina ([email protected])

lOMoARcPSD|4664814

LSB_F7_Rev Kit:297mm x 210mm

28/10/09

13:37

Page 140

FINANCIAL REPORTING (INTERNATIONAL) (b)

REPORT To:

Chief Executive

From:

An Accountant

Date:

January 2009

Subject:

Financial position of Breadline

This report analyses the overall financial position of Breadline. The company is a customer of Judicious and a possible acquisition target for the Judicious Group. The 2009 statement of financial position of Breadline reports net assets of $3.7million, and increase of $1.2million since the previous year. The movements in net assets may be further analysed as follows: Non-current assets Non-current assets have increased from $2million to $4.12million. This is due to: •

The disposal of freehold premises with a previous carrying value of $1.25million (based on revalued amount) and acquisition of leasehold premises with a carrying value of $2.5million



Investment in plant resulting in the carrying value more than doubling from $750,000 to $1.62million.

This increase in non-current assets has adversely affected non-current asset turnover which has fallen from 3.25 to 2.06 times. It is possible that this will increase once again in the future as the full benefit of the new plant is felt and translates into further increases in revenue. As regards the premises, Breadline continues to trade from the same business address, suggesting that the freehold and leasehold premises are one and the same.This indicates that Breadline may have obtained financing in the form of a sale and leaseback transaction. Thus a one off profit of approximately $1.25million ($2.5m - $1.25m) is included in cost of sales, and Breadline would have benefited from cash proceeds of $2.5million. It should be noted, however that finance costs relating to the leased property are not evident in the statement of comprehensive income, nor is a lease liability reported in the SFP.This matter requires further investigation, and details of the following should be sought, if indeed this is a leaseback transaction: •

Interest rate implicit in the lease



Length of the lease.

As regards the additions to plant, this may be indicative of an increased market or market share, and so expansion of Breadline’s operations. The existence of increased market share should be investigated in order to justify the heavy investment in plant. Working capital Both inventory and receivables of Breadline have increased since 2008: Inventory has increased by 54% from $240,000 to $370,000 and receivables by 60% from $600,000 to $960,000. Both of these increases are in part justified by the 30% increase in revenues (from $6.5million to $8.5million) and the 50% increase in cost of sales as adjusted for the one-off profit ($4.81million to $7.2million). The increase in receivables in particular is disproportionate to the increase in revenue, as evidenced by an increase in receivables days from 34 to 41. This may indicate a poor credit control function or may be the result of offering increased credit terms to new customers in order to access new market share. The terms offered to all customers should be investigated in order to assess Breadline’s position in this respect. Inventory days remains relatively unchanged in 2009 at 19 days.The main concern here is whether this period of stock holding is appropriate for a bakery business. Fresh produce is unlikely to last for so 140

Downloaded by isavic Alsina ([email protected])

1 for format

1 per valid extended comment to a maximum of 10

1 for each matter for concern / investigatio n to a max of 5.

lOMoARcPSD|4664814

LSB_F7_Rev Kit:297mm x 210mm

28/10/09

13:37

Page 141

ANSWERS long, and if ‘old’ goods are being retailed, there will be a reputational impact which in turn will impact sales and profits.The exact nature of inventories should be investigated in order to ascertain whether this is indeed an issue. The payables balance of Breadline has increased by 75% from $590,000 to $1.03million.Within this amount is $100,000 owing to Judicious in X0 and $340,000 in 09. Some increase in payables is to be expected given the increased activity of Breadline resulting in a 50% increase in cost of sales (see above), however a 75% increase seems excessive. Calculation of payables’ days reveals an overall increase from 44 days to 63 days. IN respect of the Judicious balance, the increase is from 46 days to 103 days. The terms offered by our company are 30 days, and therefore this jump is a matter for some concern. Investigations should be undertaken as to why this is the case, and indeed the credit period offered by other suppliers should be investigated. There are clear indications of a cash crisis within Breadline, indicated in particular by an obvious inability to pay suppliers and the reduction of the bank balance from a credit balance of $250,000 to an overdraft of $220,000. Liquidity problems are further evidenced by a current ratio reduced from 1.85:1 to 1.06:1, and quick ratio reduced from 1.44:1 t0 0.78:1. Financing Breadline appears to have been financed during X1 from the following sources: •

The sale and leaseback transaction which realised approximately $2.5million



A share issue which raised $600,000



An issue of loan notes which raised $500,000



An overdraft of $220,000

The money raised has been used to purchase plant and pay a $900,000 dividend. As a result of the increase in loan notes and overdraft, gearing has increased from nil to 16%. Any increase in gearing brings increased risk, largely due to the increase in fixed interest costs and reduction in profit. At the current time profits are more than sufficient to cover interest, however the interest charge in the income statement does appear to be artificially low due to: •

The absence of finance lease interest



The non-commercial rate of interest on the loan note (6% below market rates).

It is assumed that the source of the low interest loan notes (and indeed the injection of capital) is Breadline’s parent company Wheatmaster. Wheatmaster is also the beneficiary of the large dividend, which brings about the question of why monies are being passed around the Wheatmaster Group in such a fashion. Certainly, if Judicious were to acquire Breadline, financing is likely to require restructuring. Conclusion On the basis of the information contained within the financial statements of Breadline, its financial position is relatively unhealthy, particularly in terms of liquidity. Furthermore there is clear manipulation of the accounts, presumably in order to make them appear attractive to would be purchasers of the company.The transference of cash around the Group through share issues/lending/ dividends masks the true position of Breadline and as such at this time an acquisition is not recommended.

141

Downloaded by isavic Alsina ([email protected])

lOMoARcPSD|4664814

LSB_F7_Rev Kit:297mm x 210mm

28/10/09

13:37

Page 142

FINANCIAL REPORTING (INTERNATIONAL) Appendix Financial position ratios 2009 500 + 220/(500+220 +3,700) 16%

2008 Nil

Current ratio

1090/590 1.85:1

1330/1250 1.06:1

Quick ratio

850/590 1.44:1

960/1250 0.78:1

Inventory days cost of sales in X1 adjusted to 7,200 (5,950 + (2,500 – 1,250))

370/7200 x 365 19 days

240/4,810 x 365 18 days

Receivables days

960/8,500 x 365 41 days

600/6,500 x 365 34 days

Payables days (based on adjusted cost of sales)

1030/7200 x 365 52 days

590/4,810 x 365 45 days

Payables days – Judicious

340/1200 x 365 103 days

100/800 x 365 46 days

Non-current asset turnover

8,500/4,120 2.06

6,500/2,000 3.25

Gearing

1 mark per relevant ratio to a maximum of 6

33. Comparator (a)

Problems of ratios The use of ratios to assess company performance is widespread, and in particular they are useful in identifying areas of the financial statements which require further investigation. In order to achieve a full analysis of a company’s position and performance, however, ratios should be used in conjunction with the full financial statements and non-financial information about the company. When using ratios regard should also be given to the following factors which may limit their usefulness: •

Many ratios can be calculated in different ways; comparison is only valid as long as the method of calculation is the same



Ratios mask the size of the companies being compared. Larger companies which can achieve economies of scale and operate globally will have very different ratios from small localised companies within the same industry.



Ratios are based on the financial statements, which in turn may be prepared under varying accounting policies. For example, a company which applies the revaluation model to PPE may report very different ratios from one which does not.



Estimates and judgements within the financial statements – such as depreciation methods – will also affect related ratios and may limit comparability.



Ratios are only truly useful when comparing one company over time or two very similar companies within the same industry.

142

Downloaded by isavic Alsina ([email protected])

1 per valid problem to a maximum of 7

lOMoARcPSD|4664814

LSB_F7_Rev Kit:297mm x 210mm

28/10/09

13:37

Page 143

ANSWERS •

Many ratios are based on year-end figures in the statement of financial position, for example receivables’ days. These are therefore affected by seasonality and possible window dressing.



Certain ratios are easily manipulated, for example the current ratio can be siginificantly altered by using cash to pay a liability immediately prior to the year end.

Additional problems of using ratios provided by interfirm comparison services are based upon the fact that the companies on which the ratios are based are not identified:

(b)



The size of the companies used may not be appropriate for comparison with an individual company



Year ends may vary, resulting in skewed results due to seasonality



The industry ranges may be broad and not correspond directly to the business of an individual company.

Ratios for Comparator

ROCE

Interfirm comparatives 22.1%

Revenue/capital employed

1.8 times

Gross profit margin

Comparator 34.6%

1/2

2,425 2222 335 + 300

3.8%

1/2

30%

555 22 2,425

22.9%

1/2

Net profit before tax margin

12.5%

186 22 2,425

7.7%

Current ratio

1.6:1

595 22 500

1.19:1

1/2

Quick ratio

0.9:1

595 – 275 2222 500

0.64:1

1/2

Inventory holding period

46 days

275/1,870 x 365

54 days

1/2

Accounts receivable collection period

45 days

320/2,425 x 365

48 days

1/2

Accounts payable payment period

55 days

350/1,870 x 365

68 days

1/2

Debt to equity

40%

300 22 335

89.5%

1/2

Dividend yield

6%

2.5%

1/2

Dividend cover

3 times

(90,000/600,000 shares) 22222222 6 96/90

1.07

1/2

186 + 34 2222 335 + 300

1/2

143

Downloaded by isavic Alsina ([email protected])

lOMoARcPSD|4664814

LSB_F7_Rev Kit:297mm x 210mm

28/10/09

13:37

Page 144

FINANCIAL REPORTING (INTERNATIONAL) (c)

REPORT By:

An Accountant

Subject:

Analysis of financial performance of Comparator

1 mark for format

This report analyses the financial performance of Comparator based on a comparison with sector averages. Profitability Comparator’s ROCE of 34.6% is significantly higher than the sector average of 22.1%. Its revenue is 3.8 times its capital employed compared to an industry average of 1.8 times.

1 mark per point to max 4 for profitability

Both of these ratios indicate that Comparator uses the resources at its disposal more efficiently than other companies in order to create revenue and profits. It should be noted, however, that these ratios are likely to be skewed by: •

the relatively low carrying value of Comparator’s non-current assets. These assets are clearly old with a written down value of just $540,000 on a cost of $3.6million.



the exceptional write off of inventory to profits (affecting ROCE only)



the fact that some companies included within the sector averages will adopt the revaluation model for non-current assets, so pushing ROCE down.

Comparator is clearly at the stage where it should be considering the replacement of its non-current assets; at such a time as this takes place, its ROCE and revenue/capital employed will reduce significantly, and may be in line with sector averages. The gross profit margin of Comparator is 22.9% compared to a sector average of 30%. Analysis of this figure is difficult without information on Comparator’s position within the industry, but the low figure may be due to: •

Comparator being a relatively small player which is unable to achieve the economies of scale that other companies can.



Comparator concentrating on a less premium product than the companies in the sector average calculations, and so intentionally achieving lower margins.

The net profit margin of 7.7% is significantly lower than the sector average of 12.5%. Part of the reason for this is likely to be a knock-on effect from the low gross margins. A further explanation is the one-off inventory write off.A recalculation of Comparator’s net profit margin gives 12.6% ((186 + 120)/2425). This recalculation suggests that the lower gross profit margin of Comparator is balanced by lower operating costs than sector average in order to achieve a consistent net margin. Working capital management Both Comparator’s current and quick ratios are below sector average suggesting potential liquidity and cash flow problems. This is further evidenced by high working capital days compared to sector average: •

inventory days of 54 compared to 46 sector average



receivables days of 48 compared to 45 average



payables days of 66 compared to 55 average

Whilst the high inventory days are not inappropriate for the product, it should be remembered that this is calculated after the write off of obsolete items. It may suggest that further items require writing off. Alternatively, Comparator may have increased year-end inventory due to an order received or upcoming sale event. Either way, associated with increased inventory are increased holding costs which will adversely affect profit.

144

Downloaded by isavic Alsina ([email protected])

1 per point to max of 3 for working capital

lOMoARcPSD|4664814

LSB_F7_Rev Kit:297mm x 210mm

28/10/09

13:37

Page 145

ANSWERS Receivables and payables days should be assessed compared to credit terms.Without this information, analysis is of limited use. If receivables do exceed terms given, this is indicative of poor credit control and possible bad debts; if payables exceed terms received, late payment is likely to result in stops to supply and a breakdown of relationships with suppliers. Solvency Comparator has a very high gearing ratio of 90% compared to 40% sector average.This is also evidenced by the interest cover of just 2.This places Comparator in a relatively risky position whereby raising further finance is difficult.This will become more of an issue as Comparator requires investment in non-current assets in the short term.

1 per point to max of 2 for solvency

Dividends The dividend yield of 2.5% is significantly lower than the sector average of 6%.This reduces the attractiveness of Comparator to investors.This, together with the gearing ratio makes the raising of finance through a share issue an unlikely option.

1 per point to max of 2 for dividends

Dividend cover of 1.07 is also low, indicating that Comparator pays out almost all of its profit for the year as a dividend.This is an unwise policy for a company which is already suffering from cash flow problems and with limited access to future funds. Conclusion Comparator appears to be in a relatively weak financial position based on sector averages. Its performance ratios are boosted only by the low carrying values of its non-current assets, and with regard to its long and short term financing, it may be argued that the company’s going concern status is in doubt.

34. Bigwood (a)

Statement of cash flows for Bigwood for the year ended 30 September 2009 $000 Profit before tax Finance costs Depreciation (W1) Loss on disposal of plant (1,200 + 50) Increase in inventory Increase in receivables Increase in payables Cash generated from operations Interest paid Income taxes paid (W2) Net cash from operating activities Cash flows from investing activities Purchase of PPE (W3) Costs of disposal of PPE

$000 700 300 3,800 1,250 (1,400) (50) 950 222 5,550 (300) (480) 222 4,770

(10,500) (50)

Net cash used in investing activities

Marks

1/2 1/2

1 1 1/2 1/2 1/2

1 1

1 1/2

222 (10,550)

145

Downloaded by isavic Alsina ([email protected])

lOMoARcPSD|4664814

LSB_F7_Rev Kit:297mm x 210mm

28/10/09

13:37

Page 146

FINANCIAL REPORTING (INTERNATIONAL) Cash flows from financing activities Issue of share capital ((5,000 + 1,000) – 3,000) Proceeds from long-term borrowings (3,000 – 1,000) Dividends paid

3,000 2,000 (600)

Net cash used in financing activities Net decrease in cash and cash equivalents Cash and cash equivalents at beginning of period Cash and cash equivalents at end of period

1 1 1

222 4,400 222 (1,380) 450 222 (930)

1/2 1/2

Workings (W1) Depreciation Balance b/f Disposal (3,000 – 1,200) Charge for year ( ) Balance c/f

$000 3,000 (1,800) 3,800 222 5,000

(W2) Income tax paid Payable b/f 450 IS tax charge Paid ( ) Payable c/f

$000 250 (480) 222 220

(W3) Purchase of PPE $000 9,500 (3,000) 10,500 222 17,000

Cost of PPE b/f Disposals Additions ( ) Cost of PPE c/f (b)

REPORT By:

An Accountant

Subject:

Performance and position of Bigwood

1 for format

This report analyses the performance and position of Bigwood.The analysis if based on income statements for the years ended 30 September 2008 and 2009; statements of financial positions at those dates, a statement of cash flows for the year ended 30 September 2009 and the ratios included in the appendix to this report. Performance Bigwood’s ROCE at 9.3% in 2009 is significantly lower than the 2008 equivalent figure of 33.9%. This drop initially indicates that Bigwood is not using the assets at its disposal as efficiently as in the previous year to create profits.The result is, however, skewed by the significant increase in noncurrent assets during the year from a carrying value of $6.5m to $12m.This increase represents the acquisition of 5 new stores (primarily for the sale of clothing) and the refurbishment of others. It is unclear as to when this investment took place during the year, but it is very possible that its effect on profits has not yet been felt in full, as evidenced by a fall in clothing sales per square metre from $445 to $333. In future years ROCE is likely to rise again as the new stores become established and generate increased revenues and profits. A further reason for the fall in ROCE is low profits in 2009. The overall gross profit margin has dropped from 20% to 16% and the net margin from 7.1% to 2%. 146

Downloaded by isavic Alsina ([email protected])

1 per valid explained point to a max of 4 for performance

lOMoARcPSD|4664814

LSB_F7_Rev Kit:297mm x 210mm

28/10/09

13:37

Page 147

ANSWERS The gross margin can be further analysed between clothing and food: •

The gross margin relating to clothing has halved from 18.6% to 9.4%



The margin relating to food has increased from 25% to 32%.

The fall in clothing margin is surprising given the involvement of a famous designer in creating the clothing items. Such involvement would usually result in a premium being charged. It may be that increased competition on the high street has meant that this is not possible, however the main reason for the fall in margins is the increased depreciation included within cost of sales relating to clothing as a result of the new stores and increased floor space, 92% of which relates to clothing. The increase in the margin relating to food may be explained by Bigwood achieving economies of scale on its supplies, or sourcing alternative suppliers who charge lower prices.Alternatively, Bigwood may have marketed its food as premium produce and thus been able to increase prices. Such marketing costs may contribute to 2009’s higher operating expenses. In any case, the decision to allocate the majority of the new selling space to clothing seems questionable given the higher margins in the food business.The reasons for this should be investigated. The overall fall in net margin is partly the result of the drop in gross margin. Other reasons include: •

Increased operating expenses (12% of revenues in 09 compared to 10% in 08). This is accounted for by the $1.25m loss on refurbishment of stores. If this exceptional item is stripped out, then operating expenses are $1.5m, which is just 7% of revenues



The increased interest expense related to $2million of new borrowings and the overdraft.

Interest cover has fallen from 25 times in 08 to 3.3 times in 09.This is due to both the increased interest expense and the fall in profits. 3.3 times interest cover is acceptable, although further falls in this ratio would be a matter for concern. Position As already discussed, the non-current asset base of Bigwood has increased considerably with the acquisition of new stores and refurbishment of others. This has been financed by way of: •

A new $2m loan



$3m raised through the issue of shares.

1 per valid explained point to a max of 4 for position

As a result of these movements, Bigwood’s gearing ratio has increased from 17% to 28%. On the face of it, and in light of the interest cover, this level of debt appears acceptable, however it should be assessed against the industry norm. The inventory holding period of Bigwood has increased from 39 to 68 days in respect of clothing and dropped from 17 to 15 days in respect of food. The increase in clothing inventory days is indicative of obsolete stock; the very high inventory days also suggest that Bigwood does not operate in high fashion (where new collections are launched weekly in some cases) and therefore the new designer may have created some pieces which are too cutting edge for its usual customers and so have not sold. A write down is likely to be required; this will adversely affect profits. The food inventory days has fallen, however remains high given the nature of the product.This level should be assessed against food shelf life to assess whether it is appropriate. Payables days is high at 59 in 09, and has increased from 50 in 08. This must be compared to credit facilities offered in order to come to a valid conclusion as to whether it is appropriate. Based on normal credit terms of 30 days, it is very high and could result in interest being charged, damaged supplier relationships, and in the worst case scenario, stopped supplies. The increase is also indicative of cash flow problems as further evidenced by the cash / overdraft balance.

147

Downloaded by isavic Alsina ([email protected])

lOMoARcPSD|4664814

LSB_F7_Rev Kit:297mm x 210mm

28/10/09

13:37

Page 148

FINANCIAL REPORTING (INTERNATIONAL) The current ratio of Bigwood is very low at 0.71:1 (0.77:1 in 08).A low ratio is acceptable given the nature of the business (retailing to the public means that the level of receivables is negligible), however Bigwood’s high payables and overdraft mean that the ratio is still lower than it should be. Cash position The business is clearly in a weak position as regards cash, with a healthy balance of $450,000 in 08 becoming an overdraft of almost $1m in 09.Analysis of the statement of cash flows identifies that this is almost exclusively due to the heavy investment in new stores. The cash problem is likely to be short term, as in due course these stores should generate cash and profits. In the meantime the company would benefit from formalising the overdraft into a loan agreement in order to reduce bank charges.

1 per valid explained point to a max of 2 for cash

Assuming that cash generated by operations remains static in the coming year (i.e. disregarding an expected growth), the $4.7million generated will be sufficient to repay the overdraft and the loan. Share price The share price of Bigwood has fallen significantly over the year from $6 to $3.This is in part due to the share issue but may also suggest a lack of market confidence in the company, possibly associated with the general economy or possibly associated with Bigwood’s financial position. Reasons for this should be further investigated.

1 for explanation of fall in MV

Conclusion Bigwood has recently undergone a massive expansion, increasing its selling space by 35%. As yet the effects of this have not been felt in terms of profit and cash generation. In future years the position should improve, although it is questionable whether the company is right to allocate the majority of the increased floor space to the lower margin clothing product.

1 for conclusion

Appendix

Gross profit margin (overall)

Share of total revenue Clothing Food Operating expenses as % of revenue

Sales per square foot Clothing Food

2009 3,750/23,000 16%

2008 3,900/19,600 20%

69% 31%

80% 20%

2,750/23,000 12%

1,900/19,600 10%

16m/48,000 $333 7m/6,000 $1,167

15.6m/35,000 $445 4m/5,000 $800

148

Downloaded by isavic Alsina ([email protected])

1 mark per relevant ratio to a max of 3

lOMoARcPSD|4664814

LSB_F7_Rev Kit:297mm x 210mm

28/10/09

13:37

Page 149

ANSWERS

35. Tabba (a)

Statement of cash flows for Tabba for the year ended 30 September 2009

Profit before tax Investment income Finance costs Depreciation (W1) Profit on disposal (12 – 7.4) Release of grant (W2) Increase in insurance claim receivable Increase in inventory Increase in receivables Increase in payables Cash generated from operations Interest paid Income taxes paid (W3) Net cash from operating activities Cash flows from investing activities Purchase of PPE (W4) Proceeds of sale of PPE Interest received Grant proceeds Net cash used in investing activities Cash flows from financing activities Redemption of loan Proceeds from long-term borrowings Finance lease payments (W5) Net cash used in financing activities Net increase in cash and cash equivalents Cash and cash equivalents at beginning of period Cash and cash equivalents at end of period

$000 50 (40) 260 2,200 (4,600) (250) (300) 222 (2,680) (700) (500) 1,100 222 (2,780) (260) (1,350) 222 (4,390)

Marks

1/2 1/2 1/2

1 1 1 1/2

1/2 1/2 1/2

1/2

1

(2,900) 12,000 40 950 222 10,090

See W4

(4,000) 800 (1,100) 222 (4,300) 222 1,400 (550) 850

1

1/2 1/2 1/2

1 See W5

1/2 1/2

149

Downloaded by isavic Alsina ([email protected])

lOMoARcPSD|4664814

LSB_F7_Rev Kit:297mm x 210mm

28/10/09

13:37

Page 150

FINANCIAL REPORTING (INTERNATIONAL) Workings (W1) Depreciation $000 $000 4,400 (1,200) 2,200 222 5,400

Balance b/f Disposal Charge ( ) c/f (W2) Government grant $000 Balance b/f (900 + 400) Awarded in year Amortised ( ) c/f (1,400 + 600)

$000 1,300 950 (250) 222 2,000

(W3) Income taxes Balance b/f (1,200 current + 500 deferred) Income statement Paid ( ) c/f (100 + 200)

$000 1,700 (50) (1,350) 222 300

(W4) PPE Balance b/f Disposal Additions by way of lease Additions ( ) c/f

$000 20,200 (8,600) 1,500 2,900 222 16,000

1/2

1

1/2

(W5) Finance lease Balance b/f (1,700 + 800) New lease Payment ( ) c/f (2,000 + 900) (b)

1/2

$000 2,500 1,500 1,100 222 2,900

Change in financial position of Tabba The net assets of Tabba remain similar at 30 September 2009 ($8.55m) as they were at the previous year end ($8.45m). Further analysis of the company’s financial statements reveal, however, significant changes in the elements of the financial position of the company. Non-current assets The carrying value of Tabba’s non current assets has fallen from $15.8million to $10.6million. $2.2million of this is accounted for by the depreciation charge for the year, and the remainder of the difference is due to the sale of the company’s factory.This initially indicates that Tabba’s capacity has reduced and that it is may face operational problems. In this case, however, the sale is part of a sale and operating lease back transaction. Such transactions are commonly used by companies wishing to realise cash, however retain the use of their assets. The reduction in non-current assets is therefore not a matter for concern in itself, although the need for the sale and leaseback in order to realise cash may be indicative of liquidity or solvency problems.

150

Downloaded by isavic Alsina ([email protected])

1/2

1

1/2

1 per valid explained point to a max of 8

lOMoARcPSD|4664814

LSB_F7_Rev Kit:297mm x 210mm

28/10/09

13:37

Page 151

ANSWERS Current assets and working capital The cash position has improved significantly compared to 08, when there was an overdraft of $550,000.The increase in cash to a positive balance of $850,000 is largely explained by the sale of the factory on the sale and leaseback transaction, although a grant of $950,000 and loan issue of $800,000 also contribute to the positive cash inflow. What is particularly worrying is that the company is not generating a positive cash flow from operations.This is a sustainable source of income, whereas financing transactions and grant receipts are not ongoing.This lack of generation of cash by the company’s operations indicates solvency problems in the near future. Indeed it is clear that in the current year the sale and leaseback transaction has been used to finance the cash outflows suffered by the company’s operations and those required to meet tax and interest obligations. The negative cash flow from operations can be explained in two main ways: •

The company makes very small profits in the first place, indeed if the profit on disposal of $4.6million, the increase in insurance claim of $300,000 and grant amortisation of $250,000 were stripped out,Tabba’s profit of $50m becomes a loss of $5.1m



Management of working capital appears inefficient, with increases in both inventories and receivables.

The increased inventories of Tabba at the 09 year end are explained by the write off at the year end of inventories in the previous year. Indeed if the $1.5m insurance claim were added to the 08 inventories, the 09 stocks are reduced on the previous year.This may be in line with reduced sales levels, however income statements for the two years are required in order to conclude on this matter. Trade receivables are 19% higher than the previous year at $3.1million.Again, without income statement information, the reasons for this are unclear, although it may be that the credit control function is not operating efficiently and a write off is required. Certainly, in order to improve their cash position,Tabba should be ensuring that its debts are paid on time. The payables of the company have increased significantly,Whilst this has a positive impact on the statement of cash flows, it is a further indicator of liquidity problems, and Tabba’s inability to meet its obligations. Liabilities Tabba has two main financing arrangements: •

6% loan notes



Finance leases (4 years remaining)

It is questionable in the light of the above analysis whether the obligations in respect of these items can be met when they fall due. The future cash status of Tabba is likely to be little improved; it is unlikely that a bank would finance a company in this position and the only expected future cash inflows are those of the insurance claim and tax repayment.These amounts are not substantial enough to absorb the operating cash losses and therefore unless Tabba sells more non-current assets it is likely to become insolvent in the near future and lose its going concern status.

151

Downloaded by isavic Alsina ([email protected])

lOMoARcPSD|4664814

LSB_F7_Rev Kit:297mm x 210mm

28/10/09

13:37

Page 152

FINANCIAL REPORTING (INTERNATIONAL)

36. Nedberg (a)

Statement of cash flows for Nedberg for the year ended 30 September 2009

Profit before tax Finance costs Depreciation Loss on disposal Amortisation of development expenditure (W1) Government grant release (W2) Goodwill impairment Increase in inventory Increase in receivables Increase in payables Cash generated from operations Interest paid (W3) Income taxes paid (W4) Net cash from operating activities Cash flows from investing activities Purchase of PPE Development expenditure Proceeds of sale of PPE (W5) Grant received Net cash used in investing activities Cash flows from financing activities Issue of share capital (W6) Proceeds from long-term borrowings (300 – 100) Dividends paid Net cash used in financing activities Net increase in cash and cash equivalents Cash and cash equivalents at beginning of period Cash and cash equivalents at end of period

152

Downloaded by isavic Alsina ([email protected])

$m 870 30 320 50 130 (90) 20 222 1,330 (480) (310) 145 222 685 (20) (130) 222 535

(250) (500) 20 50 222 (680)

450 200 (320) 222 330 185 (115) 222 70

Marks

1/2 1/2 1/2 1/2

1 1/2 1 1/2 1/2

1/2 1/2 1/2

1 1/2 2

1/2 1/2

2 1/2 1/2

3 1 1/2

1/2 1/2

lOMoARcPSD|4664814

LSB_F7_Rev Kit:297mm x 210mm

28/10/09

13:37

Page 153

ANSWERS Workings (W1) Development expenditure Balance b/f Capitalised Amortisation charge ( ) c/f

$m 100 500 (130) 222 470

(W2) Government grant Balance b/f (900 + 400) Awarded in year Amortised ( ) c/f (1,400 + 600)

$m 300 50 (90) 222 260

(W3) Loan interest Accrual b/f Income statement 30 Paid ( ) Accrual c/f

$m 5 (20) 222 15

(W4) Tax Balance b/f (160 current + 140 deferred) Income statement Paid ( ) c/f (130 + 310)

$m 300 270 (130) 222 440

(W5) PPE $000 1,830 200 250 (320) (70) 222 1,890

Balance b/f Revaluation Additions Depreciation Disposal CV c/f Proceeds of sale = 50 loss + 70 CV = $20m (W6) Share issue Revaluation reserve b/f Revaluation Transfer Bonus issue ( ) Revaluation reserve c/f

$m – 200 (10) 50 222 140

153

Downloaded by isavic Alsina ([email protected])

lOMoARcPSD|4664814

LSB_F7_Rev Kit:297mm x 210mm

28/10/09

13:37

Page 154

FINANCIAL REPORTING (INTERNATIONAL) Share capital Share premium $m $m 750 350 (50) (200) (250) 222 222 500 100

c/f Bonus issue Share issue ( ) b/f (b)

Total $m

450

Commentary on financial position of Nedberg Nedberg has an overall cash inflow of $330m.The main sources of the cash inflows are: •

$685m cash generated from operations



$450m proceeds of a share issue



$200m proceeds of a loan issue.

Of these inflows, the cash generated by operations is sustainable and will continue in future years, assuming that Nedberg continues to trade as present. The financing inflows are ‘one-off’ cash flows, and Nedberg can not raise finance in this was on an ongoing basis. Whilst the operating cash flows amply cover the ongoing cash outflows of interest, tax and a dividend, the ‘one-off ‘ cash inflows have been used to finance ‘one-off’ expenditure, namely the purchase of new plant and development expenditure. Both of these items of expenditure are expected to result in future benefits to Nedberg, in the form of revenues, profits and in turn cash inflows.They are therefore an efficient way to spend the proceeds of the financing activities. IN terms of cash from operations, an operational profit of $900m ($870m + $30m finance costs) equates to a cash flow of $685m. Where operational cash flow is less than operating profit, explanation should be sought, In this case the lower amount is largely due to the management of working capital and in particular the increases in receivables and inventory.Although these have a negative impact on cash flow, the increases may have valid business reasons, for example due to increased activity or providing extended credit to new customers or stocking up ahead of a big order. If these reasons are not in existence, the increases may be evidence of poor working capital management, which should be addressed. Finally the issue of the dividend payment should be considered.A dividend of $320m is paid in the same year that the company has raised $650m in finance. More efficient management of cash flows would have seen the dividend withheld for a year and less external financing sought.

154

Downloaded by isavic Alsina ([email protected])

1 per valid explained point to a maximum of 5

lOMoARcPSD|4664814

LSB_F7_Rev Kit:297mm x 210mm

28/10/09

13:37

Page 155

ANSWERS

37. Minster (a)

Marks

Statement of cash flows for Minster for the year ended 30 September 2009

Profit before tax Investment income Finance costs Depreciation Amortisation (180 – 135) Decrease in inventory Decrease in receivables Increase in amounts on construction contracts Decrease in payables Cash generated from operations Interest paid (40 – (8% x 150) unwinding) Income taxes paid (W1) Net cash from operating activities Cash flows from investing activities Purchase of PPE (W2) Purchase of software Purchase of investments (W3) Investment income (20 – 15 gain on investments) Net cash used in investing activities Cash flows from financing activities Issue of share capital Proceeds from long-term borrowings Dividends paid 5c x 2m (W4) Net cash used in financing activities Net increase in cash and cash equivalents Cash and cash equivalents at beginning of period (35-40) Cash and cash equivalents at end of period

$000 142 (20) 40 255 45 30 110 (25) (205) 222 372 (28) (54) 222 290

(410) (180) (10) 5 222 (595)

265 120 (100) 222 285 (20) (5) (25)

1/2 1/2 1/2 1/2 1/2 1/2 1/2 1/2 1/2

1 1

1 1/2 1/2

1 1

2 1/2

1

1 1/2

155

Downloaded by isavic Alsina ([email protected])

lOMoARcPSD|4664814

LSB_F7_Rev Kit:297mm x 210mm

28/10/09

13:37

Page 156

FINANCIAL REPORTING (INTERNATIONAL) Workings (W1) Income taxes $000 75 57 (54) 222 78

b/f (50 + 25) Income statements Cash paid ( ) c/f (60 + 18) (W2) PPE

$000 940 (255) 35 150 410 222 1,280

B/f Depreciation Revaluation Provision Cash additions ( ) c/f (W3) Investments

$000 125 15 10 222 150

b/f Gain Additions ( ) c/f (W4) Share issue Share capital $000 300 75 125 222 500

b/f Bonus issue 1 for 4 Cash issue ( ) c/f

Share premium $000 85 (75) 140 222 150

Total $000

265

No shares at 1 July 500/4 = 2m (b)

Commentary on financial performance and position of Minster Performance In the absence of a comparative statement of comprehensive income for the year ended 30 September 2008 or industry comparatives, commentary on Minster’s performance is difficult. At a very basic level, however, the company is profitable making 21% gross profits and 6% net profits. It also has a more than adequate interest cover of 4 (162/40). It should, however, be noted that the dividend of $100,000 exceeds the profit for the year of $85,000, meaning that Minster is using its reserves to pay a dividend. Position The statement of financial position reveals an increase in net assets from 2008 to 2009 of 21%.This overall increase can be explained by the following factors: •

PPE has increased $340,000 to $1,280.This is in part due to the $35,000 revaluation in the year, but more importantly due to the acquisition of a coal mine and related plant costing $410,000. This acquisition strengthens Minster’s position by increasing revenue-making capacity.

156

Downloaded by isavic Alsina ([email protected])

1 mark per valid point to a maximum of 10

lOMoARcPSD|4664814

LSB_F7_Rev Kit:297mm x 210mm

28/10/09

13:37

Page 157

ANSWERS •

The acquisition of software costing $180,000. Again this purchase provides access to future revenues which will become evident in the income statement of future years.



The acquisition of new investments held at fair value through profit or loss, and unrealised gain on existing investments.The unrealised gain indicates a good investment strategy on Minster’s part. Fair value through profit or loss investments are those investments held for trading; at any time, Minster could crystallise the gain by selling the investments to realise cash.



A reduction in both inventories and trade receivables. In the absence of historical income statement information, it is unclear whether this reduction is the result of a decrease in trading activities.Alternative explanations are more efficient working management or changes in stockholding policy / credit terms offered.



A deterioration in the net bank position with a net overdraft of $5,000 in 08 becoming a net overdraft of $25,000 in 09.The cash position is further explained by the statement of cash flows (see below)



The creation of an environmental provision related to the new mine has limited impact on the net assets of Minster due to its inclusion as both a provision and as part of the cost of the asset.



A decrease in trade payables of $205,000 indicating prompter payment of suppliers. If payment is earlier than credit terms this may indicate poor use of what is in effect a free credit facility.

The overall increase in net assets is financed by: •

A share issue raising $265,000, the success of which indicates market confidence in Minster



The issue of $120,000 9% loan notes which increases gearing from nil to 7%.Although this level of gearing is low and therefore acceptable, it does mean an increase in mandatory interest payments and so introduces risk to the business.

Cash position Minster reports a net cash outflow in 09 of $20,000. $385,000 has been raised through the share and loan issue, and $372,000 cash has been generated from operations.The net overall outflow is due to the very high expenditure on the new mine and related plant and the software.As mentioned above this use of cash will lead to future economic benefits and strengthens Minster’s future prospects. Cash generated by operations is healthy and well above the related profit figure. If Minster continues to generate at least similar amounts of cash in the year ahead, it will be in a position to pay off the loan and restore its bank balance to a positive position.

157

Downloaded by isavic Alsina ([email protected])

lOMoARcPSD|4664814

LSB_F7_Rev Kit:297mm x 210mm

28/10/09

13:37

Page 158

FINANCIAL REPORTING (INTERNATIONAL)

38. Rytetrend (a)

Statement of cash flows for Rytetrend for the year ended 31 March 2009

Profit before tax Equipment installation (part of PPE cost) Finance costs Depreciation (W2) Loss on disposal (W2) Decrease in inventory (3,270 – 2,650) Decrease in receivables (1,950 – 1,100) Increase in payables (2,850 – 1,980) Increase in warranty provision (500 – 150) Cash generated from operations Interest paid Income taxes paid (630 + 1,000 – 720) Net cash from operating activities Cash flows from investing activities Purchase of PPE (W1) Net cash used in investing activities Cash flows from financing activities Issue of share capital ((11,500 + 1,500) – 10,000) Proceeds from long-term borrowings Redemption of long-term borrowings Dividends paid Net cash used in financing activities Net increase in cash and cash equivalents Cash and cash equivalents at beginning of period Cash and cash equivalents at end of period

158

Downloaded by isavic Alsina ([email protected])

Marks

$000 3,400 300 460 7,350 700 620 850 870 350 222 14,900 (460) (910) 222 13,530

1/2 1/2 1/2

1 1 1/2 1/2 1/2 1/2

1 1

(15,550) 222 (15,550)

See W1

3,000 2,000 (4,000) (430) 222 570 222

1/2

1,450 400 (1,050)

1/2

1/2 1/2 1/2

1/2

lOMoARcPSD|4664814

LSB_F7_Rev Kit:297mm x 210mm

28/10/09

13:37

Page 159

ANSWERS Workings (W1) PPE Cost b/f Disposal New equipment Additions other than equipment ( ) Cost c/f Therefore total cash additions: New equipment net of trade in all’ce (8,000 – 500) Installation costs of new equipment (not capitalised) Other additions

$000 27,500 (6,000) 8,000 7,750 222 37,250

7500 300 7,750 222 15,550

1/2 1/2 1/2

(W2) Depreciation B/f Charge for the year ( ) Disposal (6,000 x 20% x 4 years) c/f Therefore loss on disposal: Proceeds (trade in allowance) CV at disposal (6,000 – 4,800) Loss (b)

$000 10,200 7,350 (4,800) 222 12,750

500 1,200 222 (700)

REPORT By:

AN Accountant

Subject:

Performance and position of Rytetrend

This report analyses the performance and position of Rytetrend for the years ended 2008 and 2009. It is based on the income statements and statements of financial position for these two years and a statement of cash flows for 2009.

1 for intro

Profitability Rytetrend’s profit for 2009 is $2.4million, an increase of $800,000, or 50% on the previous year. Further analysis of the income statement reveals that this increase is largely due to a 35% jump in sales revenue to $31.8 million.The investment in non-current assets, and particularly the mid-year purchase of new display equipment is likely to have contributed to this increase.

1 mark per point to a max of 4 for performance

The increased level of operations has, however, resulted in a corresponding increase in costs. Cost of sales has increased by 40% to $22.5 million. This is proportionately greater than the increase in revenue, as evidenced by a fall in gross margin from 32% to 29%. This disproportionate increase suggests that Rytetrend may have reduced its prices in order to achieve the volume growth in sales. The increase in costs will also be due to: •

the non-current asset investment which results in higher depreciation (it should be noted that a further $60,000 depreciation relating to the capitalised installation costs will increase cost of sales to $22.56m)



increased warranty costs associated with increased levels of sales.

159

Downloaded by isavic Alsina ([email protected])

lOMoARcPSD|4664814

LSB_F7_Rev Kit:297mm x 210mm

28/10/09

13:37

Page 160

FINANCIAL REPORTING (INTERNATIONAL) Operating expenses have increased by 18% to $5.44million, however it should be noted that $300,000 of staff costs included within this amount should have been capitalised as part of the cost of the new equipment.A revised figure for operating expenses is $5.14 million, meaning an increase of just 12% since 2008.This increase seems reasonable given the increased volume of activitiy. Interest costs for 2009 are in line with 2008, despite the restructuring of Rytetrend’s loan financing. The beneficial effect of the reduction of debt by half, and replacement of a 10% loan with a 6% loan is negated by interest costs relating to the $1.05million overdraft of $200,000. The company’s financing arrangements are discussed in more detail below. Cash flows It appears that the 50% increase in net profits is masking Rytetrend’s cash flow and liquidity issues. Overall the company has seen a net cash outflow in 2009 of $1.45 million, causing the $1.05million overdraft. This cash deficit can be explained by the heavy investment in non-current assets of more than $15.5million. The issue of shares and new debt are largely used to redeem old debt, meaning that just $1million of the proceeds of these issues contribute towards this investment.

1 mark per point to a max of 4 for cash flows

The remainder of the funding is provided by cash generated from operations and the overdraft. Whilst cash generated by operations is healthy, contributing $13.5 million cash to the business (after the payment of tax and interest), the company appears to have miscalculated the availability of funds for investment in non-current assets. Certainly, an overdraft is a very expensive way of financing a business, and Rytetrend should seek to formalise the overdraft into a cheaper loan agreement, secured on its non-current assets. Position As discussed above, there has been heavy investment in non-current assets, which is beneficial for Rytetrend in terms of improving and growing the business. It may be that the full effect of these additions has not been felt yet, and future revenues will continue to grow as a result of the investment.

1 mark per point to a max of 4 for position

Inventory and receivables have both decreased since 2008 (despite the increase in sales volume), whilst payables has increased.The fall in inventory and receivables are further indicators that Rytetrend has a liquidity problem: it is calling in debts sooner and avoiding tying up funds in inventory. This approach to working capital is likely to be unsustainable, with goodwill with customers being lost and the possibility of stock outs. Conclusion Rytetrend is a profitable company with a relatively healthy statement of financial position, particularly in respect of non-current assets. It must, however, address its financing structure through negotiation of a new loan in order to ease its apparent liquidity problems. If this can be achieved in the short term, the company has a good long-term prospect.

39. Update Marks

(a)

Problems of the historical cost convention Application of the historical cost convention means that items in the financial statements are carried at their cost when purchased.The impact of this can be very notable in the statement of financial position, less so in the income statement. In the statement of financial position, all assets, both current and non-current are measured at their historical cost.Whilst this is generally a relatively up-to-date measurement as regards current assets, it can be very outdated as regards non-current assets.

160

Downloaded by isavic Alsina ([email protected])

1 per valid point to a max of 6

lOMoARcPSD|4664814

LSB_F7_Rev Kit:297mm x 210mm

28/10/09

13:37

Page 161

ANSWERS For example, the historic cost of land purchased a number of years prior to the reporting date would bear no resemblance to the cost of a more recently purchased piece of similar land. Both however provide access to similar economic benefits. It is therefore misleading to users of the financial statements to report the pieces of land at different historic costs, and users may be lead to believe that the more recently purchased land, measured at the higher value will provide greater benefit, due to being a larger size or in a better position, when this is not the case. The position becomes more complicated when the asset is depreciated, as not only are non-current asset measurements in the SFP understated compared to current values, but depreciation is lower than it would be under current cost accounting, so inflating profits in the income statement. Other figures in the income statement are arguably less distorted, as they generally reflect transactions within the previous year, and over this period, prices are unlikely to change dramatically. In any case, the distortion of both SFP and income statement figures impacts key ratios, particularly those which compare SFP and IS figures, such as ROCE and asset turnover. Here, income statement values are relatively current, whereas asset values may be understated, so resulting in better ratios. Where comparison of two companies is undertaken, the problem is more apparent when one company’s assets were purchased more recently than another’s: the company will older assets will report better ratios than that with newer assets.Again, this is misleading as there may be no difference between the underlying performance of the companies. (b)

Different measurement bases. Historical cost basis

Depreciation charge (250,000 x 20%) Cost Depreciation (50,000 x 3 years) Carrying value at 31 March 2009 Current purchasing power basis Current purchasing power value of asset 216/180 x $250,000 Depreciation charge : CV based on 3 yrs depreciation ($300,000 x 80% x 80% x 80%) CV based on 2 years depreciation ($300,000 x 80% x 80%) Therefore, depreciation charge CV of asset at 31 March 2009 Current cost basis Current cost value of asset adjusted for productivity $320,000 x 420/480 Depreciation charge : CV based on 3 yrs depreciation ($280,000 x 80% x 80% x 80%) CV based on 2 years depreciation ($280,000 x 80% x 80%) Therefore, depreciation charge CV of asset at 31 March 2009

$ 50,000 250,000 (150,000) 222 100,000

1/2

1/2

$ 300,000

1/2

153,600 192,000 222 38,400

1

153,600

1

280,000

1/2

143,360 179,200 222 35,840

1

143,360 1

161

Downloaded by isavic Alsina ([email protected])

lOMoARcPSD|4664814

LSB_F7_Rev Kit:297mm x 210mm

28/10/09

13:37

Page 162

FINANCIAL REPORTING (INTERNATIONAL)

40. Appraisal (a)

Ratios for Reactive

Marks

ROCE

2008 28.1%

2009 32.4%

1

Net asset turnover

4 times

4,000 2222 1,160 – 480

5.9 times

1

Gross profit margin

17%

550 2222 4,000

13.8%

1

Net profit before tax margin

6.3%

200 2222 4,000

5%

1

Current ratio

1.6:1

610 2222 480

1.3:1

1

Closing inventory holding period

46 days

250 x 365 2222 3,450

26 days

1

Trade receivables collection period

45 days

360 x 365 2222 4,000 x 75%

44 days

1

Trade payables payment period

55 days

430 x 365 2222 3,450

45 days

1

Dividend yield

3.75%

90m/400m 2222 3.75

6%

1

150 2222 90

1.67 times

1

Dividend cover

(b)

2 times

200 + 20 2222 1,160 - 480

Analysis of position and performance of Reactive It should be noted that without full financial statements for the year ended 31 march 2008, analysis is limited and based primarily on ratios. Performance During the year, Reactive disposed of significant levels of non-current assets as part of a strategy to purchase rather than manufacture goods.This disposal has reduced the company’s asset base by approximately $80million. As a result, ROCE has increased from 28.1% to 32.3% and net asset turnover has increased from 4 times to 5.9 times. A recalculation of these ratios with adjustment for the disposed of plant gives ROCE of 23.7% (220 – 40 / (680 + 80)) and asset turnover of 5.3 times (4,000/(680 + 80)). The recalculated ROCE reveals that there is an underlying poor performance in 2009, although the recalculated asset turnover does suggest an increase in turnover compared to 2008. The increased turnover may be the direct result of the advertising campaign and rebate policy, whilst the The fall in the recalculated ROCE is further evidenced by the gross profit and net profit margin, which have both decreased, the latter in spite of the $40million exceptional gain on disposal of the plant.This reduction may be the result of : •

The costs of the rebates offered

162

Downloaded by isavic Alsina ([email protected])

1 per point to max 10

lOMoARcPSD|4664814

LSB_F7_Rev Kit:297mm x 210mm

28/10/09

13:37

Page 163

ANSWERS •

The advertising campaign costs



The changed cost base as a result of buying in goods rather than producing them in house.

Liquidity and working capital The current ratio has fallen slighty, and may now be considered to be too low. The possible liquidity issue is further highlighted by the existence of a $10million overdraft at 31 March 2009, despite the cash inflow in the year of $120m from the sale of the plant. IN terms of working capital, receivables’ days remains relatively constant, however there is a fall in both inventory and payables days.The inventory holding period has fallen from 46 days in 2008 to 26 days in 2009, and the payables period from 55 days in 2008 to 45 days in 2009. Both may be the result of a changed working capital policy, related to the new strategy of buying in goods. Dividends Dividend cover has fallen (based on the same level of dividend as 2008), giving a clear indication of lower absolute profits than 2008. Furthermore the increase in dividend yield can only be explained by a fall in the share price of the company, indicating a lack of market confidence in Reactive. Conclusion In conclusion, the sale of the plant and new strategy to buy in goods appear to have skewed the ratios to, in many cases, appear to have improved. The underlying performance of the company, however, appears to have weakened, as does its position, particularly in respect of cash and liquidity. (c)

Analysis of a not-for-profit organisation Not-for profit organisations include schools, hospitals and charities. As the name suggests, their main objective is not the creation of wealth for shareholders, but instead to fulfil their reason for existence.

1 mark per point to a max of 5

Therefore many of the ratios applicable to commercial organisations, such as ROCE or profit margins are not applicable to these not-for-profit entities. It should be noted, however, that some ratios, particularly those relating to efficiency of operations such as receivables’ days are still relevant, as are liquidity ratios. As an alternative to assessing these entities based on profitability, a generally accepted method is to assess them based on the 3 Es: economy, efficiency and effectiveness. This type of assessment concentrates not only on financial key performance indicators, but also on no-financial factors, for example: •

Exam pass rates for schools



Number of patients treated for hospitals



Number of activities funded for charities.

163

Downloaded by isavic Alsina ([email protected])

lOMoARcPSD|4664814

LSB_F7_Rev Kit:297mm x 210mm

28/10/09

13:37

Page 164

FINANCIAL REPORTING (INTERNATIONAL)

164

Downloaded by isavic Alsina ([email protected])

lOMoARcPSD|4664814

LSB_F7_Rev Kit:297mm x 210mm

28/10/09

13:37

Page 165

F7 Feedback and Review Form

Downloaded by isavic Alsina ([email protected])

lOMoARcPSD|4664814

LSB_F7_Rev Kit:297mm x 210mm

28/10/09

13:37

Page 166

FINANCIAL REPORTING (INTERNATIONAL)

166

Downloaded by isavic Alsina ([email protected])

lOMoARcPSD|4664814

LSB_F7_Rev Kit:297mm x 210mm

28/10/09

13:37

Page 167

FEEDBACK AND REVIEW FORM

F7

Please take the time to complete this feedback and review form about the study materials that you have used for your ACCA exams. We really appreciate your comments. YOUR DETAILS Name

:

Address

:

How did you use this material? □ □



Home study (only using books) Home study (books and InterActive videos) Classroom course

What made you buy this material? □ □ □ □ □ □ □

Saw information on LSBF website Saw information on InterActive website Saw advertisement Recommendation from friend/colleague Recommended by lecturer at college Used LSBF/InterActive materials before Other (please state)

STUDY MANUALS Please make an assessment about the quality of the videos were in the following areas:

Clarity of tutor explanations Engaging and interesting tutor Exam focus, hints and tips

Very useful

Useful

Not useful













□ □ □

Examples and exercises Overall Opinion

□ □ □

□ □ □

Would you use our materials again? □

Yes



No

INTERACTIVE VIDEOS (IF USED) Please make an assessment about the quality of the videos were in the following areas:

Clarity of tutor explanations Engaging and interesting tutor Exam focus, hints and tips Examples and exercises Overall opinion

Very useful □

Useful □

Not useful □

























Please return this form to: Paul Merison, Publications Manager, London School of Business and Finance, 8-9 Holborn, London EC1N 2LL.

167

Downloaded by isavic Alsina ([email protected])

lOMoARcPSD|4664814

LSB_F7_Rev Kit:297mm x 210mm

28/10/09

13:37

Page 168

FINANCIAL REPORTING (INTERNATIONAL) Please use this space to make any additional comments that you have about either the InterActive videos, study manuals or any other aspect of our service:

Thank you for taking your time to complete this form. Good luck in your forthcoming exams. If you would like to make any other general comments about this manual, please forward them to [email protected] or complete our electronic feedback on www.lsbf.org.uk/pbfeedback 168

Downloaded by isavic Alsina ([email protected])